Hechos Geométricos en el Triángulo (2015)

Angel Montesdeoca

(Última actualización: )

Hechos Geométricos en el Triángulo de:
(2013) (2014) (2016) (2017) (2018) (2019) (2020) (2021) (2022) (2023) (2024)

Cómo es el enlace a un Hecho Geométrico correspondiente a un día concreto:
http://amontes.webs.ull.es/otrashtm/HGT2015.htm#HGddmmaa
EJEMPLO: Jueves, 1 de enero del 2015
http://amontes.webs.ull.es/otrashtm/HGT2015.htm#HG010115

  • lunes, 28 de diciembre del 2015

    Isocúbicas cónico-pivotales cK(#Q,X2)


    Special Isocubics in the Triangle Plane. §8.1, J.P.Ehrmann and B.Gibert
    Otra descripción de las isocúbicas cónico-pivotales cK(#Q,X2)
    Isocúbicas cónico-pivotales cK(#Q,X2) y transversales isotómicas
    Transformación afín y triángulos cevianos


      Sean ABC un triángulo, A'B'C' el triángulo antimedial y h una recta variable que pasa por un punto fijo Q. El lugar geométrico de los centros Qh de las hipérbolas circunscritas H(Q,h) a A'B'C' y con una asíntota h, es la isocúbica cónico-pivotal cK(#Q,X2).
      El punto Qh también es el punto de intersección de la recta h con la parábola P(Q,h) circunscrita a ABC y cuyo eje tiene la dirección de h.

      cK(#Q,X2) es la isocúbica no pivotal nK(Q²,X2,Q), de polo , raíz el baricentro, que pasa por Q y por los vértices de triángulo ceviano de Q respecto al triángulo antimedial. Es unicursal con Q como punto doble.


      En coordenadas baricéntricas si Q(l:m:n) y h: (m-n)(mn+t)x+(n-l)(ln+t)y+(l-m)(lm+t)z=0 es una recta que gira alrededor de Q, la ecuación de la hipérbola, H(Q,h), circunscrita al triángulo antimedial con asíntota h es:

    (m-n)(-l^2+l(m+n)+m*n+2t)(-l^2(m+n)+l(m^2+n^2-2t)+(m+n)t)^2x^2 -
    2(l-m)(l-n)(m-n)(l^4(m+n)^2-2l^3(m^3+m^2n+m*n^2+n^3)+l^2(m^4-2m^3n-3m^2n^2-2m*n^3+n^4)
    +2l*m(m-n)^2n(m+n)+m^2(m-n)^2 n^2+(4l^3(m+n)-l^2(8m^2+9m*n+8n^2)
    +l(4m^3-9m^2n-9m*n^2+4n^3)+4m(m-n)^2n)t
    +(4l^2-13l(m+n)+4m^2-13m*n+4n^2)t^2-9t^3)y*z + ... =0.

      Los puntos suspensivos (...) indican permutación cíclica. Su centro Qh es:

    Qh = ((l^2(m+n)-l(m^2+n^2-2t)-(m+n)t)/((m-n)(m*n+t)):...:...).

      Y el lugar geométrico de estos puntos, cuando h gira alrededor de Q, es la cúbica nK(Q²,X2,Q):

    x(m²z²+n²y²) + y(n²x²+l²z²) + z(m²x²+l²y²) - 2(l m+l n+m n)x y z =0.

      Cuando h pasa por los vértices del triángulo antimedial, la hipérbola H(Q,h) degenera en el producto de las recta h y un lado del triángulo medial; así, los vértices de triángulo ceviano de Q, respecto al triángulo antimedial, están sobre la cúbica.

      La parábola P(Q,h) cicunscrita a ABC y con eje paralelo a la recta h es:

    (l^2 (m + n) - l (m^2 + n^2 - 2 t) - (m + n) t)^2yz+... = 0.

      Esta parábola interseca a la recta h en el centro Qh de la hipérbola H(Q,h).


      Las asíntotas de nK(Q²,X2,Q) son:

    (m+n)(2l+m+n)x - l^2y - l^2z = 0, -m^2x + (n+l)(2m+n+l)y - m^2z= 0, -n^2x -n^2y + (l+m)(2n+l+m)z = 0.

      Forman un triángulo homotético a ABC con centro en y razón ρ= 2(mn+lm+nl)/(l+m+n)^2.
      Como en toda cúbica las tangentes en tres puntos alineados vuelven a corta a la curva en tres puntos alineados, las asíntotas cortan a la cúbica en los puntos de la recta ("satellite line of the line at infinity"), paralela a la tripolar de Q²:

    m^2n^2(m+n)(2l+m+n)x + l^2n^2(l+n)(l+2m+n)y + l^2m^2(l+m)(l+m+2n)z = 0.

      En particular, si Q es el incentro "the satellite line of the line at infinity" de la cúbica nK(X6,X2,X1) es la tripolar de X1171 ("1st Saragossa point of X42"), que pasa por los centros X512, X1326, X2605, X3733.

    Cónica pivotal:

      La otra asíntota h' de la hipérbola H(Q,h) tiene la dirección del eje de la otra parábola P(G,h') circunscrita a ABC y que pasa por el centro de H(Q,h). La ecuación de h' es :

    h': (m-n)^2(mn+t)(-l^2+lm+mn+ln+2t)x + ... =0.

      La envolvente de estas rectas es la cónica (denominada cónica pivotal) de ecuación:

    C(Q): (m-n)^2x^2-2(l^2+mn+3l(m+n))y z + ... =0.

      Esta cónica está inscrita en los triángulos antimedial y preceviano de Q. Su centro Q0=(m+n:n+l:l+m), es complemento de Q. Su perspector Q', con respecto al triángulo antimedial, es el conjugado isotómico de Q respecto a este triángulo:

    Q' = (l^2-l(m+n)-m^2-mn-n^2 : m^2-m(n+l)-n^2-nl-l^2 : n^2-n(l+m)-l^2-lm-m^2).

      El punto de tangencia T, de cada asíntota h' con la cónica pivotal es:

    T = (l^3(m+n)^2 - l^2(m+n)(m^2-4mn+n^2-4t) + l(-m^3n+2m^2n^2-mn^3+8mnt+4t^2)+ 2(m+n)t^2)/(m-n) : ... : ...).



    Ejemplos de cúbicas nK(Q²,X2,Q)

     • nK(X6,X2,X1), contiene a los centros X1, X874, X875, X2284.
      Sus asíntotas forman un triángulo homotético a ABC mediante la homotecia de centro el el simediano y razón 2(ab+ac+bc)/(a+b+c)^2 = (r^2+4rR+s^2)/(2s^2).
      Su cónica pivotal pasa por X4427 y su centro es el punto de Spieker (incentro de triángulo medial, X10).

     • K015 = nK(X2,X2,X2), "Tucker nodal cubic", su cónica pivotal es la elipse circunscrita de Steiner.

     • K406= nK(X393,X2,X4).
     K406 tiene tres asíntotas paralelas a los lados de ABC, formando un triángulo homotético a ABC mediante la homotecia con centro en X393 y razón 2cotA cotB cotC cotω, donde ω es el ángulo de Brocard.
      La cónica pivotal, inscrita a los triángulos antimedial y anticeviano del ortocentro, tiene su centro en el circuncentro y pasa por los puntos donde la recta de Euler corta a la circunferencia circunscrita (X1113, X1114) y por X1897= (1/((b-c)SA):...:...).

     • nK(X32,X2,X6).
      Contiene a los centros X6, X880, X2284, X2395, X5468. La razón de homotecia entre el triángulo ABC y el formado por la asíntotas es (1/2)secω. El cento de la cónica pivotal es X141 (simediano del triángulo medial).

     • K052=nK(X4590,X2,X99).
     Contiene a los centros X99, X805, X877, X880, X892, X5468. Sus asíntotas concurren en X4590.
      La cónica pivotal tiene su centro en X115 (centro de la hipérbola de Kiepert) y pasa por X2, X315, X671, X6392, X7760.


  • domingo, 20 de diciembre del 2015

    Circunferencias tangentes a las circunferencias circunscrita e inscrita



    (Advanced Plane Geometry Dao Thanh Oai )

      Dado un triángulo ABC, sea Γae y Γai las circunferencias tangentes internamente a la circunferencia circunscrita, O(R), en A y tangentes a la circunferencia inscrita, I(r), externamente e internamente en Ae y Ai, respectivamente. Se definen Be, Ce y Bi, Ci cíclicamente.   Para construir las circunferencias Γae y Γai, tangentes en A a O(R) y a I(r), hacemos uso de la inversión de polo A que deja invariante a I(r).
      Las inversas de Γae y Γai, mediante esta inversión, son rectas tangentes a I(r) y paralelas a la tangente tA en A a O(R). Por lo, para resolver el problema, solo necesitamos trazar las dos rectas tangentes a I(r) paralelas a tA.
      Si Te y Ti son los puntos de tangencia de tales paralelas, los puntos Ae y Ai donde las rectas ATe y ATi vuelven a cortar a I(r), son los puntos de tangencia de la circunferencias Γae y Γai con I(r).

      Los triángulos y son perspectivos con centro de perspectividad X57.


      Denotamos por A'B'C' el triángulo delimitado por las tangentes a I(r) en Ae, Be, Ce y por A''B''C'' el triángulo delimitado por las tangentes a I(r) en Ai, Bi, Ci.

      Los triángulos A'B'C' y A"B"C" son perspectivos con centro de perspectividad X57. Sus vértices están sobre una misma cónica.


      Se denota por DEF el triángulo ceviano de un punto P. El lugar geométrico de los puntos P tales que los triángulos A'B'C' y DEF son perspectivos es la cúbica Φ, circunscrita a ABC de ecuación baricéntrica:


      Las coordenadas baricéntricas de A' son (a^2(b+c-a ) : b(a^2+c(c-b)-a(b+2c)) : c(a^2+b(b-c)-a(2b+c))).
      La cúbica Φ pasa por X1, X7, X57, X142, X354 y por los vértices del triángulo ceviano del punto de Gergonne (X7).

      Si el punto P recorre la cúbica Φ el centro de perspectividad Q de A'B'C' y DEF queda sobre la isocúbica pK(Ω,X57), con Ω=(a^3 (b+c-a)(a(b+c)-(b-c)^2):...:...).


      Pares {P,Q} con P sobre Φ y Q el centro de perspectividad de A'B'C' y DEF, sobre pK(Ω,X57):
      {X1, X55}, {X7, X57}, {X142, X8012}, {X354, X354}.
      Si P=X57, el centro de perspectividad Q es:

    (a^2(b+c-a)(a^3-3a^2(b+c)+ a(3b^2-2b c+3c^2)-(b-c)^2(b+c)): ... : ...).

    que tiene coordenadas trilineales exactas en el triángulo de ETC: (-2.5844587408030239, 2.8544107668115589, 2.8573610621777724). Es al interseccción de las rectas X55X220 y X57X218.
      La cúbica pK(Ω,X57) pasa, además, por los vértices del triángulo ceviano de X57, por los vértices de A'B'C', y por los puntos B'C∩C'B = (a((b-c)^2-a(b+c)) : b^2(a-b+c) : c^2(a+b-c)), C'A∩A'C y A'B∩B'A.

      Se denota por DEF el triángulo ceviano de un punto P. El lugar geométrico de los puntos P tales que los triángulos A"B"C" y DEF son perspectivos consta de la tripolar de X279 y de la cónica Ψ1, circunscrita a ABC que pasa por X7 y X57.


      Las coordenadas baricéntricas de A" son (a^2(a^2-(b-c)^2) : b(a-c)(-a^2+b^2+2a c-c^2) : c(b-a)(a^2-2a b+b^2-c^2)).
      La cónica Ψ1, de perspector X3669, pasa por los centros X7, X57, X269, X479, X1014, X1119, X1396, X1462, X2091, X8051.

      Si el punto P recorre la cónica Ψ1, el centro de perspectividad Q de A"B"C" y DEF queda sobre la cónica Ψ2, circunscrita a ABC y que pasa por X56 y X57.

      Cuando P recorre Ψ1 el centro de perspectividad Q de A"B"C" y DEF es el segundo punto de intersección de la recta PX57 con Ψ2.
      La cónica Ψ2 pasa por los vértices de A"B"C" y por los centros X56, X57, X738, X951, X1407, X1412, X1416, X1434, X1435, X1477, X2137, X2213, X6612, X7091, X7130, X7131, X7153.

      Si el punto P recorre la tripolar de X279, el centro de perspectividad Q de A"B"C" y DEF queda sobre la tripolar de X269.



    El triángulo A'B'C' es también perspectivo con los triángulos siguiente:
    1. A'B'C' perspectivo con ABC.
      Centro de perspectividad: X(354)
    2. A'B'C' perspectivo con el triángulo excentral.
      Centro de perspectividad: X(57)
    3. A'B'C' perspectivo con el triángulo de Feuerbach.
      Centro de perspectividad:

      U=((b+c-a) (a^3(b+c)^2 -a^2(b-c)^2(b+c)- a(b-c)^2(b^2+6b c+c^2)+ (b-c)^4(b+c)): ... : ...).

      que tiene coordenadas trilineales exactas en el triángulo de ETC: (2.5218174588907384, -10.902294391703143, 10.024491002829291)
    4. A'B'C' perspectivo con triángulo extangencial.
      Centro de perspectividad: X(55)
    5. A'B'C' perspectivo con el triángulo de Apolonio.
      Centro de perspectividad:

      W=( a^2(a-b-c) (a^4(b+c)^2-2a^2(b+c)^2(b^2+c^2)-4a b c(b^3+c^3)+(b^3-b^2c+b c^2-c^3)^2):...:...)

      que tiene coordenadas trilineales exactas en el triángulo de ETC: (3.1433391928115959, 2.7611853200434348, 0.27830270981049537)


  • martes, 15 de diciembre del 2015

    Problema 11860 de "The American Mathematical Monthly"



    (Problems and Solutions The American Mathematical Monthly Vol. 122, No. 8 (October 2015), pp. 801-808 )

    10860



      En un triángulo ABC, a, b, c son las longitudes de los lados opuestos a los vértices A, B, C, respectivamente. Dado un número real positivo t, la circunferencia A(ta), de centro A y radio ta, corta a la altura por A en dos puntos A', A'' (A'' en la semirrecta con origen en A y que contiene al pie de la altura por A). Similarmente, tomando las circunferencias B(tb) y C(tc), se definen los puntos B', B'' y C', C''.

      En coordenadas baricéntricas, referidas al triángulo ABC, la ecuación de la circunferencia A(ta) es:
    c^2 x y + b^2 x z + a^2 y z - (x + y + z) (-a^2 t^2 x + (c^2 - a^2 t^2) y + (b^2 - a^2 t^2) z) =0.

      Que interseca a la altura por el vértice A en A'=(ta^2+S:-tSC:-tSB) y A''=(ta^2-S:-tSC:-tSB), donde
    S es el doble del área de ABC, SA=(b^2+c^2-a^2)/2, ... (notación de Conway).

      Por permutación cíclica se obtiene: B'=(-tSC:tb^2+S:-tSA), B''=(-tSC:tb^2-S:-tSA), C'=(-tSB:-tSA:tc^2+S:), C''=(-tSB:-tSA:tc^2-S).

      Para obtener los valores de t tales que los triángulos A'B'C' y A''B''C'' sean equiláteros, tendremos que imponer que los vértices A' y A'' coincidan, respectivamente, con los vértices A'i y A''i de los triángulos equiláteros A'iB'C' y A''iB''C'', levantados interiormente sobre B'C' y B''C''.

      Considermos los siguientes triángulos:
          AaBbCc, obtenido como imagen de ABC, mediante la homotecia de centro su baricentro y razón 4.
          AeBeCe, triángulo de Kiepert, para θ=π/3. Es decir,
          AeBC, triángulo equilátero construido exteriormente sobre BC.
          AiBiCi, triángulo de Kiepert, para θ=-π/3. Es decir,
          AiBC, triángulo equilátero construido interiormente sobre BC.

      Los puntos A'i y A''i quedan, respectivamente, sobre las rectas AaAi y AaAe.

      La intersecciones de la altura por A con las rectas AaAi y y AaAe son, respectivamente, A1 y A2:

      Si resolvemos λ A' = A1 y λ A'' = A2, se obtienen los valores:

    t=1/√‍3   y   λ= 12 S ± 2 √‍3 SB.

      Concluimos que:

      Los triángulos A'B'C' y A''B''C'' son equiláteros si y solo si t=1/√‍3.
    Que son A1B1C1 y A2B2C2, respectivamente.




    OTRAS CONSIDERACIONES:

    •  El centro de los triángulos equiláteros A1B1C1 y A2B2C2 es el baricentro de ABC.

    •  La circunferncia circunscrita a A1B1C1 pasa por el centro X617 (anticomplemento del segundo punto isogónico).
      La circunferncia circunscrita a A2B2C2 pasa por el centro X616 (anticomplemento del primer punto isogónico).

    •  El centro radical Rt de las circunferencia A(ta), B(tb) y C(tc) está sobre la recta de Euler: ORt / RtH = -t^2/(1+t^2).

    •  Los triángulos AaBbCc y A2B2C2 son perspectivos (las rectas AiAaA2, BiBbB2 y CiCcC2 son concurrentes), con centro de perspectividad:

        Wi = ( (a^4+2 a^2 b^2-3 b^4+2 a^2 c^2+6 b^2 c^2-3 c^4)+2√‍3 (3 a^2-b^2-c^2)S : ... : ... ),

    que tiene coordenadas trilineales exactas en el triángulo de ETC: (-2.341488177186877, -0.4052508749808191, 5.001909630980428).

    •  Los triángulos y son perspectivos (las rectas AeAaA1, BeBbB1 y CeCcC1 son concurrentes), con centro de perspectividad:

        We = ( (a^4+2 a^2 b^2-3 b^4+2 a^2 c^2+6 b^2 c^2-3 c^4)-2√‍3 (3 a^2-b^2-c^2)S : ... : ... ),

    que tiene coordenadas trilineales exactas en el triángulo de ETC: (-73.01080236858014, 49.12946584317834, 3.325250760590207).

    NOTA (Peter Moses, 22/12/2015):
      En comunicación personal, Peter Moses (además de corregir una errata) hace constar que los puntos Wi y We son los anticomplementos de X627 y X628, respectivamente.
      Además, Wi=3 X2 - 4 X17 y We=3 X2 - 4 X18, están sobre la cuártica Q088 (Hilton quartic).

  • sábado, 12 de diciembre del 2015

    Triángulo determinado por tres rectas de Simson-Wallace

    a Pablo, por su "cumple"



    (Triangle determined by three Simson lines. Art of Problem Solving, Strobenz, Nov 23, 2015)

      Let ABC be a triangle and let P, Q, R be three points lying on the circumcircle of ABC. Prove that if the three Simson lines determined by P, Q and R intersect at X, Y, Z, then the triangle XYZ is similar to the triangle PQR.
      Dado un triángulo ABC y un punto P, sea DEF el triángulo circunceviano de P. Denotamos por A'B'C' el triángulo determinado por las rectas de Simson-Wallace de los puntos D, E y F.

      Si P(u:v:w), en coordenadas baricéntricas, las coordenadas de A' (las de B', C' se deducen por permutación cíclica) son:
     
       A'(a^2(2b^2u+a^2v+b^2v-c^2v)(2c^2u+a^2w-b^2w+c^2w) 
           (a^2b^2u^2-b^4u^2+a^2c^2u^2+2b^2c^2u^2 - 
            c^4u^2+a^4u v-a^2b^2u v+a^2c^2u v+a^4u w + 
            a^2b^2u w-a^2c^2u w+2a^4v w) :
        (a^2u+b^2u-c^2u+2a^2v) (a^2c^2u+b^2c^2u-c^4u+a^4w-a^2b^2w-a^2c^2w) 
           (-a^2b^2u^2+b^4u^2-b^2c^2u^2-a^4u v+
            a^2b^2u v+a^2c^2u v-2a^2b^2u w-a^4v w-a^2b^2v w+a^2c^2v w) : 
        -(-a^2b^2u+b^4u-b^2c^2u-a^4v+a^2b^2v+a^2c^2v)
           (-a^2u+b^2u-c^2u-2a^2w)(a^2c^2u^2+b^2c^2u^2-c^4u^2+2a^2c^2u v+ 
            a^4u w-a^2b^2u w-a^2c^2u w+a^4v w-a^2b^2v w+a^2c^2v w)).
       

      El triángulo A'B'C' degenera (las recta de Simson-Wallace son concurrentes) si el punto P recorre la cúbica K024.


    Shûzô TAKAMATSU, Shûsaku OGINO.- On Some Cubics connected with Simson Lines.
    Tohoku Math. J. 43, p 99-103 (1937).
      Theorem 2. (p.102)
      If a triangle A'B'C' is in perspective with the triangle ABC which has the same circumcircle with A'B'C' and if the three Simson lines of A',B',C' with respect to the triangle ABC meet at a point, the locus of the centre of the perspectivity is generally a cubic.
      The equation of the curve in trilinear coordinates is:
        sin A α(β^2+γ^2) + sin B β(γ^2+α^2) + sin C γ(α^2+β^2) = 0.

      Los triángulos DEF y A'B'C' son simétrico (en una simetría central) si y sólo si P está sobre la cúbica de McCay, K003.

      El lugar geométrico de los puntos P tales que los triángulos semejantes (no simétricos) DEF y A'B'C' sean perspectivos es una séxtica con puntos dobles en los vértices de ABC.

      Como en este caso los triángulos DEF y A'B'C' son semejantes y perspectivos, los centros de semejanza y de perspectividad están en la circunferencia circunscrita a ABC.

  • domingo, 29 de noviembre del 2015

    "The tripolar centroid and the Tucker nodal cubic"

    a Lolilla, por su "cumple"



      Dado un triángulo ABC y un punto P, se considera la cónica circunscrita c(P), de perspector P; su centro P0 es el cociente ceviano G/P, del baricentro y P. La cónica circunscrita c(P0) de perspector P0 tiene centro en P=G/P0. El cuarto punto de intersección (distinto de A, B, C) de c(P) y c(P0), tripolo de la recta PP0, lo denotamos por Q.
      Designamos por T y T0 los baricentros de cada terna de puntos en los que las polares de P y P0, respecto a las cónicas c(P) y c(P0), respectivamente, cortan a los lados de ABC. Como estas polares coinciden (en este caso) con las tripolares de P y P0 respecto a ABC, cada punto T y T0 es el "tripolar centroid" de P y P0, concepto introducido por Darij Grinberg.
      Sea R el punto de intersección de las polares de P y T respecto a c(P) y R0 el punto de intersección de las polares de P0 y T0 respecto a c(P0) (los triángulos PRT y P0R0T0 son autopolares respecto a las cónicas c(P) y c(P0), respectivamente).   En baricéntricas, las coordenadas de los puntos considerados son:

    P(u:v:w),   P0(u(v+w-u):v(u-v+w):w(u+v-w)),   Q(u/(v-w):v/(w-u):w/(u-v)),
    T(u(v-w)(v+w-2u):v(w-v)(u-2v+w):w(u-v)(u+v-2w)),
    R(u(2u^2-2u(v+w)-v^2+4vw-w^2):...:...),
    T0(u(v-w)(v+w-u)^2(2u^2-u(v+w)-(v-w)^2):...:...),
    R0(u(u-v-w)(2u^4-2u^3(v+w)-u^2(3v^2-8vw+3w^2)+4u(v-w)^2(v+w)-(v-w)^2(v^2+4vw+w^2)):...:...)

      Si P está sobre la cúbica nodal de Tucker, Ψ, del triángulo medial de ABC ("tripolar centroid" de la recta del infinito), los triángulos PRT y P0R0T0 son perspectivos.
      También P0 está en la cúbica y el centro de perspectividad P' es el tercer punto de intersección de la recta PP0 con la cúbica.

      La cúbica Ψ corta a la elipse imagen de la elipse inscrita de Steiner, mediante la homotecia de centro el baricentro y razón 3½, en los puntos de coordenadas coord y en los cuatro más que se obtienen por permutación cíclica. Pasa por los centros X2, X1645, X1646, X1647, X1648, X1649, X1650, X6544. Tangente a los lados en sus puntos medios y asíntotas paralelas a los lados.

      Cuando P recorre la cúbica Ψ, el punto Q está en la recta del infinito y su dirección es la de las asíntotas PT0 de c(P) y TP0 de c(P0); las otras asíntotas pasan por el baricentro.

      Cuando P recorre la cúbica Ψ, el cuadrivértice PP0RR0 está circunscrito a la elipse inscrita de Steiner. Los puntos de tangencia son los extremos T1 y T2 de su diámetro que pasa por P' (puntos medios de los segmentos PR y P0R0), el punto T3, sobre PP0, de intersección de RT y R0T0 (polares de P y P0 respecto a las cónicas c(P) y c(P0)) y, finalmente, el otro punto T4, sobre RR0, de intersección de la elipse con la recta por T3 paralela a la dirección de Q.

      Adicionalmente, el punto T1 está sobre c(P), T2 está sobre c(P0) y T3 sobre la cónica circunscrita c(P'), de perspector P', que además pasa por R, R0 y cuyo centro (sobre Ψ) es el punto medio P'0 de RR0.


      Mas sobre "The tripolar centroid and the Tucker nodal cubic" en:
    Results on the tucker cubic(Wilson Stothers, University of Glasgow).
    The tripolar centroid (Wilson Stothers, University of Glasgow).

    Más sobre la cúbica nodal de Tucker:
    Propiedades de la cúbica nodal de Tucker
    La cúbica nodal de Tucker como lugar geométrico
    Transformación afín y triángulos cevianos



  • miércoles, 18 de noviembre del 2015

    Hipérbolas de Apolonio relativas a la cónica inscrita de centro el simediano



      Dado un triángulo ABC, se consideran la cónica Ψ inscrita de centro el simediano, un punto M en la circunferencia circunscrita y la hipérbola de Apolonio H(M,Ψ) relativa a M y a la cónica Ψ.

      El lugar geométrico del centro de la hipérbola de Apolonio H(M,Ψ), cuando M varía sobre la circunferencia circunscrita, es una elipse Φ de centro X125 (centro de la hipérbola de Jerabek) que pasa por X468 (inverso en la circunferencia circunscrita del polo, X25, del eje órtico -tripolar del ortocentro- con respecto a la circunferencia circunscrita) y ejes paralelos a las direcciones de los conjugados isogonales de los puntos donde la recta de Euler corta a la circunferencia circunscrita.

    ( Mostrar/Ocultar figura )

      gtre2085.png

    Descargar fichero GeoGebra

      La ecuación baricéntrica de la elipse Φ es:
    
     (a^2-b^2-c^2)^2 D(a,b,c)x^2 + 2(a^2+b^2-c^2)(a^2-b^2+c^2) L(a,b,c)yz + ... = 0.
     
                 
        D(a,b,c)=  3a^12+2a^10(b^2+c^2)  
                  - a^8(11b^4-6b^2c^2+11c^4)  
                  - 4a^6(b^6+b^4c^2+b^2c^4+c^6) 
                  +  a^4(13b^8-28b^6c^2+46b^4c^4-28b^2c^6+13c^8)   
                  + 2a^2(b^2-c^2)^2(b^2+c^2)^3  
                  - (b^2-c^2)^4(5b^4-2b^2c^2+5c^4) 
             
        L(a,b,c) = 3a^12-12a^10(b^2+c^2)
                 + 11a^8(b^2+c^2)^2
                 +  8a^6(b^6-2b^4c^2-2b^2c^4+c^6)
                 -   a^4(b^2+c^2)^2(15b^4-26b^2c^2+15c^4) 
                 +  4a^2(b^2-c^2)^2(b^6+5b^4c^2+5b^2c^4+c^6)
                 + (b^2-c^2)^4(b^2+c^2)^2 
    
    
      El segundo punto de intersección de la recta de Euler con la elipse Φ es:
     
           ( (a^2+b^2-c^2)(a^2-b^2+c^2)(2a^14 - a^12(b^2+c^2) - 
                       4a^10(b^2+c^2)^2 + a^8(-11b^6+19b^4c^2+19b^2c^4-11c^6)+ 
                       2a^6(17b^8+8b^6c^2-34b^4c^4+8b^2c^6+17c^8)-
                        a^4(23b^10+51b^8c^2-58b^6c^4-58b^4c^6+51b^2c^8+23c^10) +
                      24a^2b^2c^2(b^4-c^4)^2 +
                      (b^2-c^2)^4(3b^6+13b^4c^2+13b^2c^4+3c^6) ):...:...),
                      
    
    que tiene coordenadas trilineales exactas en el triángulo de ETC: (6.26684470094504707, 5.38079268349056597, -2.97688954556064983).

  • martes, 17 de noviembre del 2015

    Hipérbolas de Apolonio relativas a la elipse circunscrita de Steiner



    Elipse circunscrita de Steiner es la elipse circunscrita al triángulo y con centro en el baricentro. Es la elipse circunscrita de área mínima.

    Hipérbola de Apolonio relativa a un punto M y a una cónica (con centro), es el lugar geométrico de los puntos de intersección de un diámetro variable de la cónica con la perpendicular a su diámetro conjugado trazada desde M. Pasa por los pies de las normales trazadas desde M a la cónica, por su centro y por el propio punto M. Sus asíntotas son paralelas a los ejes de la cónica.


      Sea ABC un triángulo, DEF el triángulo medial y (Ha) al hipérbola de Apolonio de D respecto a la elipse circunscrita de Steiner. Denotamos por A' el centro de (Ha); B' y C', se definen cíclicamente.
      Entones, AA', BB' y CC' son concurrentes.

    ( Mostrar/Ocultar figura )

      HipApolonioSteiner.png

      Considerando los diámetros de la elipse circunscrita de Steiner que pasa por B y C obtenemos cuatro puntos de la hipérbola (Ha):

    Ba = (SC, -2 SA, SC),  B'a = (-2 SB - SC, 2 (SA - SC), 4 SA + 2 SB - 3 SC)
    Ca = (-SB, -SB, 2 SA),  C'a = (SB + 2 SC, -4 SA + 3 SB - 2 SC, 2 (-SA + SB)).

      Ya tenemos suficientes puntos para determinar su ecuación:

    2 (b^2 - c^2) x^2 - 2 SC y^2 + 2 SB z^2 + 2 (b^2 - c^2) y z - (3 a^2 - b^2 - 3 c^2) z x + (3 a^2 - 3 b^2 - c^2) x y=0.

      El centro de esta hipérbola es:

    A' = (2 a^4 + (b^2 - c^2)^2 - a^2 (b^2 + c^2) :
    3 a^4 + 3 b^4 - 3 b^2 c^2 + 4 c^4 - a^2 (4 b^2 + 3 c^2) :
    3 a^4 + 4 b^4 - 3 b^2 c^2 + 3 c^4 - a^2 (3 b^2 + 4 c^2)).

      Procediendo cíclicamente, se obtienen los centros B' y C' de las correspondientes hipérbolas de Apolonio relativas a E y F y a la elipse de Steiner. El punto de intersección de las rectas AA', BB', CC' es:

    X = (1/(4a^4 - 3a^2 (b^2+c^2)+ 3b^4 - 4b^2 c^2 + 3c^4) : ... : ... ).

    que tiene coordenadas trilineales exactas en el triángulo de ETC: (3.7174000661276741, 1.7127306948297531, 0.73920473958177737).


      El punto de intersección de las hipérbolas (Ha), (Hb) y (Hc), a parte del baricentro y los puntos del infinito de los ejes de la elipse; son:

    A1 = (b^2-c^2, SC, -SB),   A1 = (-SC,c^2-a^2), SA),   A1 = (SB, -SA, a^2-b^2),

    que están en la tripolar de X253, centro de perspectividad de ABC y el triángulo pedal de X64.
      Los polos A0, B0, C0 de esta recta respecto a las tres hipérbolas de Apolonio son los complementos de A1, B1, C1.


    ∗  ∗  ∗  ∗  ∗  ∗  ∗  ∗  ∗  ∗ 




      UNA GENERALIZACIÓN:

      Sea ABC un triángulo, P un punto, PaPbPc el triángulo ceviano de P y c(P) la cónica circunscrita de perspector P. Denotamos por (HPa) la hipérbola de Apolonio de Pa respecto a c(P) y por A' su centro.

      Procediendo como en el caso particular de P el baricentro, para determinar puntos (HPa), se llega a la ecuación de ésta:

    2(c^2v^2-b^2w^2)x^2-2w(SB u-a^2w)y^2+2v(SC u-a^2v)z^2+2u(SB v-SC w)y z-2(b^2u w+SA u v-2SC v w)z x+2(c^2u v+SA u w-2SB v w)x y = 0.

    si las coordenadas baricéntricas de P son (u:v:w).

      Las ecuaciones de las hipérbolas, (HPb) y (HPc), de Apolonio relativas a los punto Pb, Pc y la cónica c(P), se obtienen por permutación cíclica.

      El centro de (HPa) es:
    
          A' = (-2a^4v w (-u+v+w)+(b^2-c^2)u(-c^2v(u-2w) + 
                b^2(u-2v)w) -  a^2(c^2v(u^2-2u v + 2(v-w)w) + b^2w(u^2-2u w + 2v(-v+w))) :
               -b^4u w(u+v+ w) -  v(a^4(u+2v)w + c^4u(3v+w) - 
                a^2c^2(3u v-4v^2+2u w+2v w)) +  b^2(a^2w(u^2+2u v+2v^2-u w) + 
               c^2u(3v^2+2v w +w^2-u(2v+w))) :
               -a^4v w(u+2w) +  a^2(b^2w(2u v+3u w+2v w-4w^2) + 
               c^2v(u^2-u v+2u w +2w^2)) -  u(c^4v(u+v+w) + b^4w(v+3w) - 
               b^2c^2(v^2+2v w+3w^2-u(v+2w))))
        
        

      El lugar geométrico de los puntos P tales que los centros de (HPa), (HPb) y (HPc) forman un triángulo perspectivo con ABC es una séptica, que pasa por los vértices de ABC (dobles), por el baricentro, por el simediano y por X523 (conjugado isogonal del foco de la parábola de Kiepert). Otros puntos de esta séptica son los puntos de intersección de los lados de ABC con las bisectrices y con las tripolares del incentro y ortocentro.

    ( Mostrar/Ocultar figura )

      HipApolonioSteinerg.png



  • jueves, 12 de noviembre del 2015

    X1176, "1st Saragossa Point of X66"



    Saragossa Points (X(1166)-X(1208), ETC)

     El nombre Saragossa hace referencia al rey que probó el teorema de Ceva, antes de que éste lo hiciera.

    Al-Mutamán (WikimediA) fue rey de la Taifa de Zaragoza de la dinastía hudí entre 1081 y 1085.
      En el año 1081, Almutamán contó con los servicios de las tropas mercenarias de El Cid (desterrado de Castilla por llevar a cabo razias en contra de los intereses de Alfonso VI de León y Castilla en territorios de Toledo), para resistir el empuje del rey aragonés, Sancho Ramírez.
      Fue un rey erudito, protector de las ciencias, de la filosofía y de las artes. Un ejemplo es su tratado "El Libro de la perfección y de las apariciones ópticas" (Kitab al-Istikmal) que trata los números irracionales, secciones cónicas, la cuadratura del segmento parabólico, volúmenes y áreas de varios cuerpos geométricos o el trazado de la tangente de una circunferencia, entre otros problemas matemáticos.
      A Al-Mutamán se debe la primera formulación conocida del Teorema de Giovanni Ceva, que no sería conocido en Europa hasta 1678 en la obra "De lineis rectis" del mencionado geómetra italiano.




      Sean ABC un triángulo, DEF su triángulo medial y D1E1F1 el triángulo medial de DEF. Se denota por (Ha) la hipérbola tangente a la altura en A, tangente en D a la mediatriz de BC y con una asíntota paralela a BC. Sean Ab y Ac, los puntos donde (Ha) vuelve a cortar a los lados AC y AB, respectivamente. La recta AbAc se denota por da y definimos cíclicamente db y dc.

      Las rectas da, db y dc delimitan un triángulo perspectivo con ABC, con centro de perspectividad X1176.

    ( Mostrar/Ocultar figura )

      X(1176).png


      En coordenadas baricéntricas la ecuación de la hipérbola es:

    - a^2 y^2+ a^2 z^2+(a^2 - b^2 + c^2) x y + (-a^2 - b^2 + c^2) x z =0.

      Los segundos puntos de intersección con los lados AC y AB son:

    Ab(a^2 : 0 : a^2 + b^2 - c^2)    Ac(a^2 : a^2 - b^2 + c^2 : 0).

      Los puntos Ab, Ac, Bc, Ba, Ca, Cb están en una misma cónica,
    b^2c^2(a^2-b^2+c^2)(a^2+b^2-c^2)x^2- a^2(b^2+ c^2-a^2)(a^4-b^4+3b^2c^2-c^4)y z + ... =0,
    con centro en el punto de coordenadas baricéntricas:
    (a^2 (-a^14 + 
       3 a^12 (b^2 + c^2) + (b^2 - c^2)^2 (b^2 + c^2)^3 (b^4 - 3 b^2 c^2 +c^4) - 
       a^10 (b^4 + 6 b^2 c^2 + c^4) - 
       a^8 (5 b^6 + 2 b^4 c^2 + 2 b^2 c^4 + 5 c^6) + 
       a^6 (5 b^8 + 6 b^6 c^2 - b^4 c^4 + 6 b^2 c^6 + 5 c^8) + 
       a^4 (b^10 + b^8 c^2 + 6 b^6 c^4 + 6 b^4 c^6 + b^2 c^8 + c^10) + 
       a^2 (-3 b^12 + 2 b^8 c^4 - 2 b^6 c^6 + 2 b^4 c^8 - 3 c^12)) : ... : ...),
    
    que tiene coordenadas trilineales exactas en el triángulo de ETC: (3.070964689812219, 1.909306637010982, 0.9014684917557517).

  • viernes, 6 de noviembre del 2015

    Un segmento en la recta de Euler como lugar geométrico



    Interesting Locus S (Art of Problem Solving, IvanGD)

     Three fixed points, A, B and C, are given such that triangle ABC is a scalene triangle. A variable real parameter λ is also given, and six points, P, Q, R, K, L and M, are defined by the following six vector equalities:

    AP= λAB, BQ= λBC, CR= λCA, BK= λBA, CL= λCB, AM= λAC.

     Let X be the circumcenter of triangle PQR, and Y the circumcenter of triangle KLM. The point S is defined as the midpoint of XY.
      For any fixed triple of points A, B and C, find the locus of S, as λ takes all possible real values.


      Planteamos el problema en la generalización siguiente:

      Sean ABC un triángulo y λ un número real. Consideramos los seis puntos P, Q, R y L, M, N, definidos como sigue:

    AP= λAB,  BQ= λBC,  CR= λCA,  BL= λBA,  CM= λCB,  AN= λAC.

      Sea Z un punto en la recta de Euler de ABC y tomamos Zd y Zl los mismos puntos en los triángulos PQR y LMN, respectivamente. Denotamos por S el punto medio de ZdZl.

      Cuando Z varía en la recta de Euler de ABC, los puntos Zd y Zl describen una cúbica con punto nodal en Z y la asíntota tiene como punto del infinito el punto Biham (X1499 en ETC), dirección de la perpendicular a la recta que pasa por el baricentro y simediano.

      Si OZ:ZH=t (O, H circuncentro y ortocentro de ABC) el punto S recorre el segmento UV, sobre la recta de Euler, de extremos en los puntos:
           OU = (1/(2+2t) OH,   OV = (-1+8t)/(6+6t) OH,   OV = (-1/3) OU.
    El punto U está sobre la cúbica y V es el punto de intersección de la asíntota con la recta de Euler.

    ( Mostrar/Ocultar figura )

      AoPSc6t48f6h1159233_interesting_locus_sEuler.png

      La asíntota de la cúbica (que pasa por V) y la tangente en U son perpendiculares al eje radical (recta X2X6) común a todas las circunferencias circunscritas a los triángulos PQR y LMN.



     Los triángulo PQR y LMN tienen el mismo baricentro que ABC y en el caso particular de que el punto Z sea el circuncentro (t=0), el punto U es X5, centro de la circunferencia de los nueve puntos.
      Ver también http://amontes.webs.ull.es/pdf/ejtr2436.pdf.

  • sábado, 31 de octubre del 2015

    Los centros del triángulo X3513 y X3514



    X3513 (Encyclopedia of Triangle Centers. Clark Kimberling)

      X(3513) and X(3514) are the limiting points (point-circles) of the coaxal system that includes the circumcircle and incircle. Their midpoint, X(3660), is the of the incircle and circumcircle. (Peter Moses, November 15, 2011)


      Sean ABC un triángulo, DEF el triángulo de contacto interior y D', E', F' los puntos de intersección con los lados BC, CA, AB de la tripolar del punto de Gergonne. Denotamos por Γa, Γb y Γc las circunferencias de diámetros DD', EE' y FF', respectivamente.

      Las tres circunferencias Γa, Γb y Γc son coaxiales, cortándose en los centros X3513 y X3514.

    ( Mostrar/Ocultar figura )

      X(3513)-X(3514).png

    Descargar fichero GeoGebra

      La ecuación baricéntrica de la circunferencia de Apolonio de Γb y Γc es:

      Las ecuaciones de las otras dos circunferencias de Apolonio, se deducen de ésta por permutación cíclica. Estas tres circunferencias son coaxiales de eje (pasa por el incentro y circuncentro):

    bc(a-b-c)(b-c)x + ca(b-c-a)(c-a)y + ab(c-a-b)(a-b)z = 0.

      Los dos puntos comunes de las tres circunferencias de Apolonio son (r y R son los radios de las circunferencias inscritas y circunscritas a ABC, respectivamente):

    ((2a-b-c)(a+b-c)(a-b+c)(a^2+(b-c)^2-2a(b+c)) ± 4(a+b+c)(2a^2-(b-c)^2-a(b+c))r(r^2+4rR)^(1/2) : ... : ... ),

    que corresponden a los centros del triángulo X3513 y X3514.

    NOTA: Peter Moses (comunicación personal) determina que el factor de proporcionalidad entre las coordenadas expresadas aquí y las que figuran en ETC, a((a-b+c)(a+b-c) ± 4r(r^2 + 4rR)½), es:

    h = 4sr(-r-4R ±3 (r(r+4R))½).


      Peter Moses da otra expresión para las coordenadas baricéntricas de X3513:

    (ra + (r (r + 4 R))½:...:...),

    donde ra es el radio de la circunferencia A-exinscrita a ABC.
      O bien, en coordenadas tripolares: (ra : rb : rc).

  • jueves, 29 de octubre del 2015

    Séxtica asociada a cónicas (4P11T)1


    (FORUM GEOM, Paris Pamfilos)

      Aplicamos a la geometría del triángulo, el caso de construcción de cónicas, dados cinco de sus elementos: tres puntos, la tangente en uno de ellos y la dirección de un punto del infinito.
      Paris Pamfilos.- A Gallery of Conics by Five Elements. Forum Geometricorum Volume 14 (2014) 295–348. §8.3

      Sean ABC un triángulo, P un punto en su plano y tA la tangente en A a la cónica, C(P), circunscrita a ABC de perspector P (es decir, cónica circunscrita con punto de Brianchon P).
      Denotamos por AP el centro y por Pa el perspector de la cónica (CA), circunscrita a ABC, tangente en A a tA y con un punto en el infinito dado por la dirección de la recta AP.
      Procediendo cíclicamente, se tienen los centros BP, CP y los perspectores Pb, Pc de las cónicas (CB) y (CC), respectivamente.

      El lugar geométrico de los puntos P tales que los puntos AP, BP, CP están alineados es la séxtica de ecuación baricéntrica:

    ( Mostrar/Ocultar figura )

      Sextic(4P_11T)_1.png

    Descargar fichero GeoGebra

      El baricentro, A, B, C son puntos dobles (aislados) de esta séxtica y tiene un par de asíntotas paralelas a cada lado de ABC. Pasa por las reflexiones de los vértices en el baricentro y es tangente en ellos a la elipse circunscrita de Steiner. Pasa por las imágenes de las reflexiones de los vértices en el baricentro mediante la homotecia de centro el baricentro y razón 4.

      La cónica (CA) puede ser construida, determinando un punto adicional, D, de ella. Con lo que estamos en la caso de construcción cónicas IPPPP (cónica: dados cinco puntos, uno en el infinito).
      Para construir el punto D, trazamos una recta BA' y aplicamos el Teorema de Pascal al caso límite de las cinco rectas tA, AP, paralela por C a AP, BC y BA' (tomadas en este orden).

      Una tratamiento analítico:

      Sea P(p:q:r) en coordenadas baricéntricas en el triángulo de referencia ABC. La cónica circunscrita, (CA), de perspector P tiene ecuación pyz+qzx+rxy=0, y la tangente en A es tA: ry+qz=0.
      Consideremos el haz de cónicas formado por las cónicas degeneradas BC·tA y AB·AC: x(ry+qz) + λyz=0.
      La ecuación de la cónica de este haz que pasa por el punto del infinito (q+r:-q:-r) de AP, se obtiene para el valor λ=2(q+r).

    (CA):  2(q+r)yz + qzx+ rxy = 0.

      Su centro es AP (-2(q+r)^2:q(q+3r):r(3q+r)).
      Procediendo cíclicamente se tienen los centros, BP(p(p+3r):-2(p+r)^2:r(3p+r)) y CP(p(p+3q):q(3p+q):-2(p+q)^2), de la correspondientes cónicas (CB) y (CC).
      Imponiendo las condición de que estos puntos estén alineados, resulta que P ha de estar en la séxtica de ecuación: 3 x^4 y^2 + 6 x^3 y^3 + 3 x^2 y^4 + 2 x^4 y z + 2 x y^4 z + 3 x^4 z^2 - 42 x^2 y^2 z^2 + 3 y^4 z^2 + 6 x^3 z^3 + 6 y^3 z^3 + 3 x^2 z^4 + 2 x y z^4 + 3 y^2 z^4 =0.
      Las asíntotas paralelas a BC son: (1±13½)x-y-z=0.


      El lugar geométrico de los puntos P tales que los perspectores de las cónicas (AB), (CB) y (CC) están alineados es la elipse circunscrita de Steiner y la recta del infinito.

      Si P recorre la elipse circunscrita de Steiner, la recta que une los perspectores de las tres cónicas pasa por el baricentro.
      Si P esta en el infinito, dicha recta es tangente a la elipse inscrita de Steiner y el punto de contacto es el cuadrado baricéntrico de P.

  • jueves, 22 de octubre del 2015

    Caracterización de las cúbicas K117 y K216


    (CTC, Bernard Gibert)

      Sea el centro X3142 (reflexión del punto de Longchamps en el ortocentro) del triángulo ABC. Para todo punto P se denota por Xa, Xb, Xc este mismo centro en los triángulos PBC, PCA, PAB, respectivamente.

      (Bernard Gibert. On two Remarkable Pencils of Cubics of the Triangle Plane)

      El lugar geométrico de los puntos P tales que los triángulos ABC y XaXbXc son perspectivos es la cúbica K117 y el lugar del centro de perspectividad, Q, es la cúbica K127.



      Sean DEF el triángulo ceviano de P, D1 y D2 los puntos de intersección de la recta BC con la circunferencia con centro en dicha recta y que pasa por E y F.
      Los dos pares de rectas que unen los puntos E, F con los puntos D1, D2 determina una recta da, perpendicular a BC (en un punto Ma).
      Si (u:v:w) son las coordenadas baricéntricas del punto P, la ecuación de la recta da es:

    (a^2(v-w)-(b^2-c^2)(v+w))x-2((c^2-b^2)u+a^2(u+w))y+2((b^2-c^2)u+a^2(u+v))z =0.


      Similarmente, se definen las rectas db y dc, procediendo cíclicamente sobre los lados de ABC.

    ( Mostrar/Ocultar figura )

      HG221015Diagonales.png

      El lugar geométrico de los puntos P tales que las rectas da, db y dc son concurrentes es la cúbica K117.
      El lugar del punto de concurrencia, T, es la cúbica K127.
      Cuando P recorre la cúbica K117, los cuatro puntos P, Q, T y X3142 están alineados.

    ( Mostrar/Ocultar figura )

      HG221015Ceviano.png

    Ver en GeoGebraTube

      Si P=(u:v:w) está sobre la cúbica K117 entonces el punto T de concurrencia de las rectas da, db y dc, sobre la cúbica K127, es:

    (a^4(5u^2 - 7v w + u(v+w)) - 2a^2u(b^2(5u+4v+3w) + c^2(5u+3v+4w)) + (b^2-c^2)(b^2(5u^2+7u v+9u w+3v w)-c^2(5u^2+9u v+7u w+3v w)) :...:...).

      Si P=X3146=( a^2SA-3SBSC:...:...), entonces el correspondiente punto T es:

    T3146 = (4a^10+8a^6b^2c^2-7a^8(b^2+c^2)+2a^4(b^2-c^2)^2(b^2+c^2)+4a^2(b^2-c^2)^2(b^4+c^4) -3(b^2-c^2)^4(b^2+c^2):...:...),

    que tiene coordenadas trilineales exactas en el triángulo de ETC: (0.516370022433727, 0.916295241354345, 2.76798161215427).
      Los vértices del triángulo pedal de T3146 son los puntos de corte de la cúbica K127 con los lados de ABC (aparte de los vértices).

      El lugar geométrico de los puntos P tales que las rectas da, db y dc determinan un triángulo A'B'C', no degenerado, perspectivo con ABC es la cúbica K216.

      O equivalentemente, si P está en la cúbica K126 los puntos

    Ma=BC∩da,    Mb=CA∩db,    Mc=AB∩dc

    están alineados.

      Como los triángulos ABC y A'B'C' son perspectivos y semejantes, el centro de semejanza, So, y el centro de perspectividad, S, están en la circunferencia circunscrita a ABC.
    ( Mostrar/Ocultar figura )
      HG221015Sister.png
    Descargar fichero GeoGebra

    K216 (Catalogue of Triangle Cubics. Bernard Gibert)
      Let A1, B1, C1 be the reflections of A, B, C in the sidelines of triangle ABC. For any point M, denote by Ma the intersection of the lines MA1 and BC. Define Mb and Mc similarly. These points Ma, Mb, Mc form a triangle perspective to ABC if and only if M lies on the Neuberg cubic. This property is equivalent to locus property 1 in the Neuberg cubic page.
      These points Ma, Mb, Mc are collinear if and only if M lies on K216. Hence, K216 is a special case of Grassmann cubic. See CL041.


      En el caso límite en el que P=A1 (reflexión de A en BC), la recta PA1 es la tangente en A1 a K216 y el centro de perspectividad de los triángulos ABC y A'B'C' es Sa (sobre la circunferencia circunscrita):

      Por permutación cíclica se obtienen los puntos Sb y Sc, cuando P es B1 y C1, respectivamente.

      El SaSbSc es el triángulo circunceviano de

    (1/(3a^2SA^3+SA^2(SB^2+9SB SC+SC^2)-SB^2SC^2):...:...),

    que tiene coordenadas trilineales exactas en el triángulo de ETC: (0.347473681183728, 6.62797050406776, -1.10830295068579).

    (Mostrar/Ocultar una descripción de este centro)

    La cúbica K216 (Neuberg cubic sister, nK(X6, X5, ?)) pasa por las reflexiones A1, B1, C1 de los vértices del triángulo de referencia ABC.
    La tangente en A1 a K216 y las rectas A1B1 y A1C1 cortan a BC, AC y AB en Aa, Ba y Ca, respectivamente.
    Las perpendiculares a BC, AC y AB en Aa, Ba y Ca, delimitan un triángulo perspectivo con ABC, con centro de perspectividad Sa (sobre la circunferencia circunscrita).
    Similarmente, se definen Sb y Sc. Entonces, SaSbSc es el triángulo circunceviano del punto con coordenadas baricéntricas:
    (1/(3a^2SA^3+SA^2(SB^2+9SB SC+SC^2)-SB^2SC^2):...:...)



      Sean A1*, B1*, C1* conjugados isogonales de los puntos A1, B1, C1, respectivamente, los cuales también están sobre K216.

      Los centros de perspectividad S de ABC y A'B'C' cuando P coincide con los puntos A1*, B1*, C1* son, respectivamente, los puntos antipodales de A, B, C en la circunferencia circunscrita.




      La tangente en A1 a la cúbica K216 corta a BC en Ma1. Denotamos por da1 la perpendicular por Ma1 a BC. Las rectas db1 y dc1 se definen de forma similar.

      El triángulo delimitado por las rectas da1, db1 y dc1 es perspectivo con ABC, con centro de perspectividad:

    (a^2 (a^10-3 a^8 b^2+2 a^6 b^4+2 a^4 b^6-3 a^2 b^8+b^10+a^8 c^2-2 a^4 b^4 c^2+b^8 c^2-7 a^6 c^4+a^4 b^2 c^4+a^2 b^4 c^4-7 b^6 c^4+5 a^4 c^6+5 b^4 c^6+2 a^2 c^8+2 b^2 c^8-2 c^10) (a^10+a^8 b^2-7 a^6 b^4+5 a^4 b^6+2 a^2 b^8-2 b^10-3 a^8 c^2+a^4 b^4 c^2+2 b^8 c^2+2 a^6 c^4-2 a^4 b^2 c^4+a^2 b^4 c^4+5 b^6 c^4+2 a^4 c^6-7 b^4 c^6-3 a^2 c^8+b^2 c^8+c^10):...:...),

    que tiene coordenadas trilineales exactas en el triángulo de ETC: (2.82196438798725, 9.48366816761158, -4.22739704397160).


    CASO TRIÁNGULO PEDAL

      Si DEF es el triángulo pedal de P:

      El lugar geométrico de los puntos P tales que las rectas da, db y dc son concurrentes es una cuártica tangente en el circuncentro al eje de Brocard, que pasa por los puntos isodinámicos y por X3416 ("X3-Vertex Conjugate of X6").

    ( Mostrar/Ocultar figura )
      HG221015Pedal.png
    Vertex Conjugates
      Let T be the vertex triangle of the circumcevian triangles, AUBUCU and AXBXCX, of U and X; viz., the sidelines of T are AUAX, BUBX, CUCX. Then T is perspective to ABC, and the perspector is the U-vertex conjugate of X (defined just before X3415 in ETC).

      X(3)-vertex conjugation maps the Euler line to a quartic.
    ( Mostrar/Ocultar figura )
      X3-Vertex Conjugate Brocard Axis.png


  • lunes, 19 de octubre del 2015

    Asunto: Eje de Brocard (continuación)

    a Kake, por su "cumple"



      Resumimos aquí resultados ya expuestos sobre una configuración asociada al punto de Lemoine, añadiendo otras propiedades.


      Sean ABC un triángulo y DEF el triángulo ceviano del punto de Lemoine (simediano, X6).
      Las rectas paralelas por D a AB y a AC, cortan a AC y a AB, respectivamente, en Ba y Ca. Análogamente, se definen los puntos Cb, Ab, Ac y Bc.
      Los cuadriláteros BCBaCa, CACbAb y ABAcBc son cíclicos y denotamos, respectivamente, por Γa, Γb y Γc sus circunferencias circunscritas.
    ( Mostrar/Ocultar figura )
      BrocardSuject3Circ.png

      Vamos a determinar los centros de las ocho circunferencias tangentes simultáneas a las circunferencias Γa, Γb y Γc. Así como describir ciertos puntos, sobre el eje de Brocard, relacionados con esta configuración.

    Los diez problemas de Apolonio (José María Guillén)

      Apolonio de Perga (262-190 a.C.), que es ampliamente conocido por su tratado sobre las cónicas, no lo es tanto por su tratado sobre Tangencias. En éste, Apolonio describe el problema que hoy se conoce como Problema de Apolonio y que tiene este enunciado:

    Dados tres objetos tales que cada uno de ellos puede ser un punto, una recta o una circunferencia, dibujar una circunferencia que sea tangente a cada uno de los tres elementos dados.

      Este problema da lugar a diez casos posibles. Los más sencillos (tres puntos y tres rectas) ya aparecen tratados en los Elementos de Euclides. Apolonio trató estos dos casos junto a estos otros seis (dos puntos y una recta; dos rectas y un punto; dos puntos y una circunferencia; dos circunferencias y un punto, dos circunferencias y una recta; un punto, una recta y una circunferencia) en el Libro I de las Tangencias, y los dos casos restantes (dos rectas y una circunferencia, y tres circunferencias) en el Libro II de las Tangencias. Aunque desgraciadamente estos libros se han perdido, a través de Pappus de Alejandría (s. IV d.C.) se sabe que Apolonio resolvió los nueve primeros, y hoy en día se cree que fue Isaac Newton el primer matemático que resolvió por medio de la regla y el compás el problema de encontrar la circunferencia tangente a otras tres circunferencias.



    Circunferencias de Apolonio X(1343) X(1342) X(39) Te Ti Wi We X(39) Ve Vi X(8160) X(8161) Re Ri

    Hacer clik en cada cuadrito ☐ de la figura para mostrar (en ventana aparte) la propiedad geométrica del punto correspondiente.

    Descargar fichero GeoGebra


      La circunferencia, Γe, tangente exteriormente a las circunferencias Γa, Γb y Γc, toca a éstas en Ae, Be y Ce, respectivamente.

      • Los triángulos ABC y AeBeCe son perspectivos, con centro de perspectividad en X1243 (centro exterior de homotecia de la circunferencia circunscrita y la primera circunferencia de Lemoine).


      La circunferencia, Γi, tangente interiormente a las circunferencias Γa, Γb y Γc, toca a éstas en Ai, Bi y Ci, respectivamente.

      • Los triángulos ABC y AiBiCi son perspectivos, con centro de perspectividad en X1242 (centro interior de homotecia de la circunferencia circunscrita y la primera circunferencia de Lemoine).

      • El centro de la circunferencia Γe, tangente exteriormente a las circunferencias Γa, Γb y Γc, es X8160.

      • El centro de la circunferencia Γi, tangente interiormente a las circunferencias Γa, Γb y Γc, es X8161.

      Denotamos los centros de las ocho circunferencias de Apolonio como sigue:
    Oe es centro de la circunferencia Γe, tangente exteriormente a las circunferencias Γa, Γb y Γc.
    Oi es centro de la circunferencia Γi, tangente interiormente a las circunferencias Γa, Γb y Γc.
    Oae es centro de la circunferencia Γae, tangente exteriormente a las circunferencias Γa e interiormente a Γb y Γc.
    Oai es centro de la circunferencia Γai, tangente interiormente a las circunferencias Γa y exteriormente a Γb y Γc.
      Análogamente se definen los centros Obe, Obi, Oce, Oci.

      • Las rectas OaeOai, ObeObi, OceOci concurren en X39 (punto medio de los puntos de Brocard y centro radical de las circunferencias Γa, Γb y Γc).

      • Las rectas AOae, BObe, COce concurren en un punto Te sobre el eje de Brocard.

      • Las rectas AOai, BObi, COci concurren en un punto Ti sobre el eje de Brocard. El punto medio de TeTi es X182, centro de la circunferencia de Brocard.

      Peter J. C. Moses, (comunicación personal, 14-Nov-2015) obtiene las coordenadas baricéntricas, si ω es el ángulo de Brocard:
               Ti = Tan[w/2]^2 X[3] + X[6]:: = a^2 (S + SA Tan[w/2]^2 Cot[w] )::
               Te = Cot[w/2]^2 X[3] + X[6]:: = a^2 (S + SA Cot[w/2]^2 Cot[w])::
    
    Te más cerca de X3 y Ti más cerca de X6. Tienen coordenadas trilineales exactas en el triángulo de ETC:
    Ti (6.6357576498369811187, 5.7833890613545006759, -3.4258776297242664965),
    Te (1.1395137394250342411, 1.6009246079754071902, 2.0064026428052352448).


      Establezcamos notaciones para los puntos de tangencia de las circunferencias:

    Abeb∩Γae,   Acec∩Γae,   Abib∩Γai,   Acic∩Γai,
    Bcec∩Γbe,   Baea∩Γbe,   Bcic∩Γbi,   Baia∩Γbi,
    Caea∩Γce,   Cbeb∩Γce,   Caia∩Γci,   Cbia∩Γci.

      • Las rectas AbeAce, BceBae y CaeCbe delimitan un triángulo perspectivo con ABC, cuyo centro de perspectividad We está en el eje de Brocard.

      • Las rectas AbiAci, BciBai y CaiCbi delimitan un triángulo perspectivo con ABC, cuyo centro de perspectividad Wi está en el eje de Brocard.

      Peter J. C. Moses, (comunicación personal, 14-Nov-2015)
     	   Wi = Tan[w]^4 / (1 - Sec[w]) X[3] + X[6] = a^2 (SA + S Cot[w]^2 (Cot[w] - Csc[w]))::
               We = Tan[w]^4 / (1 + Sec[w]) X[3] + X[6] = a^2 (SA + S Cot[w]^2 (Cot[w] + Csc[w]))::
    
      Tienen coordenadas trilineales exactas en el triángulo de ETC:
    Wi (-0.68718124850388126023, 0.21086860269055218923, 3.8118391024311727845),
    We (1.0265438934789327971, 1.5149581966088730617, 2.1180577795725749233).

      • El triángulo delimitado por las rectas BaeCae, CbeAbe y AceBce y el triángulo delimitado por las rectas BaiCai, CbiAbi y AciBci son perspectivos con ABC y tienen centro de perspectividad común: X39.

      • Las rectas BceCbe, CaeAce y AbeBae delimitan un triángulo perspectivo con ABC, cuyo centro de perspectividad Ve está en el eje de Brocard.

      • Las rectas BciCbi, CaiAci y AbiBai delimitan un triángulo perspectivo con ABC, cuyo centro de perspectividad Vi está en el eje de Brocard.

      Peter J. C. Moses, (comunicación personal, 15-Nov-2015)
     	   Vi = X[3] + (1 + Cot[w] Csc[w]) X[6] = a^2 (SA + S (Csc[w] + Tan[w]))::
               Ve = X[3] + (1 - Cot[w] Csc[w]) X[6] = a^2 (SA + S (Tan[w] - Csc[w]))::
    
      Tienen coordenadas trilineales exactas en el triángulo de ETC:
    Vi (1.4913107585596926366, 1.8686307999346269891, 1.6586997318482407400),
    Ve (0.23274128208754294570, 0.91089999627053574200, 2.9026223542951422299).

      • El centro radical, Re, de las circunferencias Γae, Γbe y Γce queda en el eje de Brocard.

      • El centro radical, Ri, de las circunferencias Γai, Γbi y Γci queda en el eje de Brocard.

      Peter J. C. Moses, (comunicación personal, 15-Nov-2015)
     	   Ri = (4+2Sec[w]+Tan[w]^2)X[3]+X[6]= a^2(2a^6+3a^4(b^2+c^2)-4a^2(b^2+c^2)^2-b^6-
      7b^2c^2(b^2+c^2)-c^6-2SA(a^2+b^2+c^2)^2sec[w])::
               Re = (4-2Sec[w]+Tan[w]^2)X[3]+X[6] = a^2(2a^6+3a^4(b^2+c^2)-4a^2(b^2+c^2)^2-b^6- 
       7b^2c^2(b^2+c^2)-c^6+2SA(a^2+b^2+c^2)^2 sec[w])::
    
      Tienen coordenadas trilineales exactas en el triángulo de ETC:
    Ri (5.9800811140007948580, 5.2844403522452551935, -2.7778316607242413499),
    Re (4.8543737469748340750, 4.4278133312766445035, -1.6652249383493751262).

    Recurso en GeoGebraTube



  • viernes, 9 de octubre del 2015

    Un centro y un par bicéntrico del triángulo

    Guaidil



      Sean ABC un triángulo y k un número real.
     Se consideran los puntos Ba y Ca sobre AC y AB, respectivamente, tales que la recta BaCa es paralela a BC y la distancia entre ellas es |kρa|, siendo ρa el radio de la circunferencia inscrita al triángulo ABaCa.
      Similarmente, se toman los puntos Cb y Ab, Ac y Bc.

      Los seis puntos Ba, Ca, Cb, Ab, Ac y Bc están en una misma cónica (Ck), para cada valor de k, de ecuación baricéntrica:
                bc(a+b+c)kx^2 - a(a^2+b^2+c^2+2(ab+ac+bc)+bck^2)yz + ... =0.

    ( Mostrar/Ocultar figura )
      rb745Ck.png
    Descargar fichero GeoGebra

      El lugar geométrico (Q) de los centros Wk de las cónicas (Ck) es una cuártica, que pasa por los centros del triángulo X2, X9, X3218,

    ( Mostrar/Ocultar figura )
      rb745quartic.png


      Las cuatro cónicas (Ck) degeneradas reales (en producto rectas) corresponden a los valores de k:

    ka = -(a+b+c)/a,   kb = -(a+b+c)/b,   kc = -(a+b+c)/c,   ks = -(a+b+c)/(abc)1/3.


      Los puntos singulares de cada cónica degenerada (punto de intersección de las rectas que la forman) tienen coordenadas baricéntricas, respectivamente:

    Sa =(bc-a2 : b(a-c) : c(a-b)),   Sb =(a(b-c) : ac-b2 : c(b-a)),   Sc =(a(c-b) : b(c-a) : ba-c2),

    S =(a-(abc)1/3 : b-(abc)1/3 : c-(abc)1/3).




      Los tres puntos Sa, Sa, Sa están alineados y el cuarto punto de intersección de la recta que los contiene con la cuártica (Q) es el centro del triángulo X3218, centro de la cónica (C-3).

      El punto S es un centro del triángulo que tiene coordenadas trilineales exactas en el triángulo de ETC: (-17.028943228905912, 0.44235839356617012, 11.193928622779759) y han sido incorporados (09/10/2015) a la Enciclopedia como X8183.
      Este punto está sobre la recta X1X2 y sobre la que pasa por los centros determinados por Francisco Javier García Capitán, (29/11/2016), X11065 y X11066.

      La cónica degenerada correspondiente al valor ka está formada por dos rectas que pasan por Sa y son paralelas a AB y AC. Análogamente con las cónicas degeneradas correspondientes a los valores kb y kc.

      Los tripolos de las dos rectas que forman la cónica degenerada (Cks), con punto singular S, forman un par bicéntrico de puntos (P,U), de coordenadas baricéntricas:

    P = ((ba2)1/3 : (cb2)1/3 : (ac2)1/3),   U = ((ca2)1/3 : (ab2)1/3 : (bc2)1/3).



      (Este par bicéntrico, P(124) = Montesdeoca trilinear poles (Dic. 2015), podría recibir el nombre de una flor: GUAIDIL)
    Note just before P(1) in BICENTRIC PAIRS OF POINTS:
      As the list of bicentric pairs continues to grow, it is convenient to borrow names from another field, one whose objects, like those of triangle geometry, please and proliferate. In keeping with the spirit of Hyacinthos message 7999, captioned "Another stellar (or flowered) transformation", names of flowers are selected for certain bicentric pairs, such as Acacia points for PU(43) . A special case is the flower name Hyacinth, which, according to The Language of Flowers - alphabetical by flower name, means the language of flowers. The symmedian point, X(6), also known as the Lemoine point in honor of Emile Michel Hyacinthe Lemoine, has bicentrics P(6) and U(6), which may be called the hyacinth points.


      La recta que pasa por los puntos de este par bicéntrico tiene la dirección del punto del infinito X513, conjugado isogonal del anticomplemento del punto de Feuerbach.
      El punto medio de P y U es (a(b+c)+2((abc)2/3+a(abc)1/3) : ... : ...), que tiene coordenadas trilineales exactas en el triángulo de ETC: (2.12885621927787, 1.74960264031873, 1.44685209125086).
      El producto baricéntrico de los puntos de este par bicéntrico es el incentro.
      El producto trilineal de los puntos de este par bicéntrico es el baricentro.

  • domingo, 4 de octubre del 2015

    Un problema de construcción en TriangulosCabri


    (TriangulosCabri, Fco. Javier García Capitán)

    Problema 745
     Dado un triángulo ABC, hallar una construcción con regla y compás de una paralela A´B´ a AB tal que la distancia entre AB y A´B´ sea el doble del radio de la circunferencia inscrita a A´B´C.

     García Capitán, F. (2015): Comunicación personal.


      Sean ABC un triángulo y un número real k, se consideran los puntos Ba en AC y Ca en AB, tales que la relación entre la distancia da, entre las rectas BaCa y BC, y el radio ρa de la circunferencia inscrita al triángulo ABaCa sea daa=k.

      Para determinar la recta BaCa, tal que se verifique la condición requerida, tomemos una recta variable YZ (Y sobre AB y Z sobre AC). Sea I0 el incentro del triángulo AYZ y consideremos las proyecciones ortogonales Z0 y D0 de Z y I0 sobre BC e YZ, respectivamente.
    ( Mostrar/Ocultar figura )
      rb745.png
    Descargar fichero GeoGebra

      El lugar geométrico del centro M de una homotecia que transforma el segmento ZZ0 en el segmento I0D0 (cuando Z varía) es la hipérbola (H) tangente a la bisectriz interior en A y al lado BC es C y con una asíntota paralela a la bisectriz interior en C.

      El lugar geométrico del punto Zk, tal que ZZk:I0Zk=k es una recta que pasa por A. Sea Mk la intersección de esta recta con la hipérbola (H); la recta paralela por Mk a la bisectriz interior en C corta a AC en el punto Ba buscado.

      El lugar geométrico del punto Z-k, tal que ZZk:I0Zk=-k es una recta que pasa por A. Sea M-k la intersección de esta recta con la hipérbola (H); la recta paralela por M-k a la bisectriz interior en C corta a AC en el punto B'a, que coincide con el punto Ba cuando se cambia k por -k.

    NOTA:   Todos los elementos geométricos que hemos considerado pueden ser construidos con regla y compás.
      Para la construcción de la hipérbola (H) podemos determinar cinco de sus puntos, tomando diferentes puntos Z en AC, o acudiendo a los casos de construcción de cónicas It_Pt_P o It_Pt_P2.


    Resultados analíticos en coordenadas baricéntricas:

      La ecuación de la hipérbola (H) es a(a+b+c)y^2 - b x(c y-b z) = 0.
      La recta lugar geométrico del punto Zk tal que ZZk:I0Zk=k es (a+b+c-c k)y + b k z = 0.
      El punto de intersección de ambas (aparte de A) es Mk = (k^2a:kb:a+b-c(1-k)).
      Con lo que se tiene que Ba=(ka:0:a+b+c) y Ca=(ka:a+b+c:0).

      Si D es el punto de contacto de la circunferencia inscrita a ABC con el lado BC, los puntos de tangencia Dk y D-k de las circunferencias inscritas a los triángulos ABaCa y AB'aC'a, con las rectas BaCa y B'aC'a, verifican:

    ADk : DkD = (a+b+c):(ak)         AD-k : D-kD = -(a+b+c): (ak)

      En el caso particular del Problema 745 de TriangulosCabri, un punto sobre la recta A'B' es el que divide al segmento CF en la razón s/c. Siendo c la longitud del lado AB, s el semiperímetro y F el punto de tangencia de la circunferencia inscrita a ABC con el lado AB.


    Ampliamos el estudio de esta configuración en la geometría del triángulo.



      Similarmente, se construyen los puntos Cb y Ab, Ac y Bc, C'b y A'b, A'c y B'c, procediendo cíclicamente sobre los lados de ABC.

      Sean los puntos A'=AbCb∩AcBc, B'=BcAc∩BaCa y C'=CaBa∩CbAb.

      Los triángulos ABC y A'B'C' son homotéticos y el lugar geométrico (cuando k varía) de su centro de homotecia T es la hipérbola circunscrita que pasa por el incentro y el baricentro. Su centro es X1015 ("exsimilicenter of Moses circle and incircle").

    ( Mostrar/Ocultar figura )
      rb745g.png

      Algunos pares (k,T): (0,X1), (∞,X2), (-2,X57), (-1,X81), (-3,X88), (-3/2,X89), ((a+b+c)2/(a2+b2+c2),X105).

      Los seis puntos Ba, Ca, Cb, Ab, Ac y Bc están en una misma cónica (Ck), para cada valor de k.

      La ecuación baricéntrica de la cónica (Ck) es:
          bc(a+b+c)kx^2 - a(a^2+b^2+c^2+2(ab+ac+bc)+bck^2)yz + ... =0.
      Las cuatro cónicas degeneradas reales (en producto rectas) de esta familia de cónicas corresponden a los valores de k:

    ka = -(a+b+c)/a,   kb = -(a+b+c)/b,   kc = -(a+b+c)/c,   ks = -(a+b+c)/(abc)1/3.


      El lugar geométrico de los centros W de las cónicas (Ck) es la cuártica, que pasa por los centros del triángulo X2, X9, X3218,

        a b c ((b-c)^2 x^4+(a-c)^2 y^4+(a-b)^2 z^4)+2 b c(a-b)(a-2 c) (b-c) x^3 y-2 a c(a-b) (b-2 c) (a-c) x y^3-2 b c(a-2 b) (a-c) (b-c) x^3 z+2 a c(2 a-b) (a-c) (b-c) y^3 z+2 a b(a-b) (a-c) (2 b-c) x z^3-2 a b(a-b) (2 a-c) (b-c) y z^3+a b c ((a^2+b^2-2 c^2-4 a b+2 a c+2 b c) x^2 y^2+(a^2-2 b^2+c^2+2 a b-4 a c+2 b c) x^2 z^2- (2 a^2-b^2-c^2-2 a b-2 a c+4 b c) y^2 z^2)-2 a b c (a^2+b^2+c^2-a b-a c-b c) x y z (x+y+z)=0.

      • Para k=0, (C0) es la cónica circunscrita de perspector el incentro y centro X9.
      • Cuando k → ∞, (C) es la elipse circunscrita de Steiner, de centro el baricentro.
      • Para k=-3, el centro de la cónica (C-3) es X3218 (anticomplemento de X908).
      Este es único valor de k para el que los incentros de los triángulos ABaCa, BCbAb. CAcBc están alineados, en la tripolar del incentro.
      Además, las tangentes trazadas desde cada uno de estos incentros a las otras dos circunferencias inscritas coinciden.
    ( Mostrar/Ocultar figura )
      rb745k-3.png

      La envolvente de las polares del incentro respecto a las cónicas (Ck) es la hipérbola con centro en X649, que pasa por X905 y sus puntos en el infinito son X513 y X514.

      La ecuación baricéntrica de esta hipérbola es bc(a-2b-2c)x^2-2a(bc+ca+ab)yz+...=0. Una de las asíntotas es la tripolar del incentro.

      Sea da la recta que pasa por los puntos de contacto con los lados AB y AC de la circunferencia Inscrita a ABaCa. Las rectas db y dc se definen cíclicamente.

      El triángulo delimitado por da, db y dc es perspectivo con ABC y el lugar geométrico del centro de perspectividad es la hipérbola de Feurbach.

    ( Mostrar/Ocultar figura )
      rb745Kiepert.png


    NOTATION AND COORDINATES in ETC:
      In order that every center should have its own name, in cases where no particular name arises from geometrical or historical considerations, the name of a star is used instead. For example, X(905) is POINT ACHERNAR (α Eridanus, the end of the river [Arabic]); X(908) is POINT ACUBENS (α Cancri, from the Arabic Az-Zubana, "the claws" of crab)


  • sábado, 3 de octubre del 2015

    Una propiedad geométrica de X194

    a Silvia, por su "cumple"



    X194 is the radical center of the Neuberg circles.

    Neuberg circles
     The A-Neuberg circle of ABC is the circle through A centered at the intersection of the A-cevian of the Tarry point and the perpendicular bisector of BC.

    Tarry point
      Is the point X98 of intersection, other than A, B, and C, of the circumcircle and Kiepert hyperbola.
    ( Mostrar/Ocultar figura )
      NeubergCircles.png


      Sean ABC un triángulo y DEF el triángulo ceviano del punto de Lemoine (simediano, X6).
      Las rectas paralelas por D a AB y a AC, cortan a AC y a AB, respectivamente, en Ba y Ca. Análogamente, se definen los puntos Cb, Ab, Ac y Bc.
      Los cuadriláteros BCBaCa, CACbAb y ABAcBc son cíclicos y denotamos, respectivamente, por Γa, Γb y Γc sus circunferencias circunscritas.

      Se considera la elipse (Cab) tangente en B al lado AB, que pasa por C, Ba y por el otro punto B'a donde la recta DBa vuelve a cortar a la circunferencia Γa.

      Así mismo, sea la elipse (Cac) tangente en C al lado AC, que pasa por B, Ca y por el otro punto C'a donde la recta DCa vuelve a cortar a la circunferencia Γa.

      Una cónica degenerada del haz de cónicas, determinado por (Cab) y (Cac), contiene a la recta BC y a otra recta, que denotamos por da (pasa por el pie de la simediana desde A). Procediendo cíclicamente se definen las rectas db y dc.
    ( Mostrar/Ocultar figura )
      HG191015.png

    Las tres rectas da, db y dc concurren en X194.

      Las ecuaciones baricéntricas de las cónicas (Cab), (Cac) y de la recta da son, respectivamente:

    (Cab): b^2x^2 - y(c^2x+a^2z)=0,       (Cac): c^2x^2 - z(b^2x+a^2y)=0,       da: (b^2-c^2)x - c^2y + b^2z = 0.

      Así, las coordenadas del punto de intersección de las rectas da, db y dc son:

    (b^2c^2 - a^2(b^2+c^2) : c^2a^2 - b^2(c^2+a^2) : a^2b^2 - c^2(a^2+b^2)).



  • viernes, 2 de octubre del 2015

    Asunto: Eje de Brocard



    (Brocard axis, Wolfram MathWorld)
     The Brocard axis is the line KO passing through the symmedian point K and circumcenter O of a triangle, where the segment OK is the Brocard diameter. The Brocard axis is perpendicular to the Lemoine axis and is the isogonal conjugate of the Kiepert hyperbola.


      Sean ABC un triángulo y DEF el triángulo ceviano de un punto P.
      Las rectas paralelas por D a AB y a AC, cortan a AC y a AB, respectivamente, en Ba y Ca. Análogamente, se definen los puntos Cb, Ab, Ac y Bc.
    Los cuadriláteros BCBaCa, CACbAb y ABAcBc son cíclicos si y solo si P=X6 es el simediano.

      A partir de ahora siempre DEF es el triángulo ceviano del simediano.
      Las circunferencias en las que estos cuadriláteros BCBaCa, CACbAb y ABAcBc están inscritos las denotamos por Γa, Γb y Γc, y sus respectivos centros por Oa, Ob y Oc.
      Los seis puntos Ba, Ca, Cb, Ab, Ac y Bc están en una misma cónica, cuyo centro tiene coordenadas baricéntricas:

    (a^8-a^6(b^2+c^2)-a^4(3b^4+2b^2c^2+3c^4)-3a^2b^2c^2(b^2+c^2)-b^2c^2(b^4-b^2c^2+c^4) : ... : ...),

    que tiene coordenadas trilineales exactas en el triángulo de ETC: (2.378616086553469, 2.216456917905695, 1.008371498794220).

      Sean Aa=BOc∩COb, Bb=COa∩AOc, Cc=AOb∩BOa.

      Las rectas AAa, BBb, CCc son paralelas al eje de Brocard.

      Las rectas AOa, BOb, COc concurren en el punto medio X182 del circuncentro y simediano.

      El centro radical de las circunferencias Γa, Γb y Γc es X39, punto medio de los puntos de Brocard.
    ( Mostrar/Ocultar figura )
      BrocardSuject.png

      Para hallar circunferencias tangentes a Γa, Γb y Γc, usamos un procedimiento debido a Adriaan van Roomen (1561-1615) que resolvió el problema utilizando la intersección de hipérbolas, pero esta solución no se basa únicamente en construcciones con regla y compás, por lo que puede considerarse menos elegante.
     (Paul Kunkel.- The tangency problem of Apollonius: three looks. BSHM Bulletin Volume 22 (2007), 34–46)

     Las circunferencias tangentes a dos de ellas tienen sus centros en una cónica de focos en los centros de las dos circunferencias. Concretando: los centros de las circunferencias tangentes a Γb y Γc están en la cónica (Ha) con centro Ha en el punto medio de los centros Ob y Oc de ambas circunferencias, que pasa por A, por el otro punto A' donde se cortan ambas circunferencias, por las reflexiones de A y A' en Ha, y por Aa=BOc∩COb (la reflexión de A' en Ha está en la circunferencia circunscrita a ABC).
     Tomando los otros dos pares de circunferencias Γc y Γa, Γa y Γb, se obtienen otras dos cónicas (Hb) y (Hc), lugares de los centros de circunferencias tangentes a cada par.
      Los dos puntos comunes Z1 y Z2 a las tres cónicas (Ha), (Hb) y (Hc) son los centros de dos circunferencias tangentes a las tres circunferencias Γa, Γb y Γc. (Para ver las ocho circunferencias de Apolonio tangentes a estas tres circunferencias pinchar aquí).
    ( Mostrar/Ocultar figura )
      BrocardSujectApolonio.png

      La primera coordenada de los puntos Z1 y Z2 (que están en el eje de Brocard y cuyo punto medio es el circuncentro) son (ω ángulo de Brocard, S doble del área de ABC):

    a^2(a^6(b^2+c^2)+2a^4(b^2+c^2)^2-a^2(4b^6+7b^4c^2+7b^2c^4+4c^6)+b^8-3b^6c^2-2b^4c^4-3b^2c^6+c^8 ± 4S^3(2a^2-b^2-c^2) csc ω)

    que tienen coordenadas trilineales exactas en el triángulo de ETC:
    Z1 = (9.530484849402319, 7.986183153420950, -6.286916862492733)
    Z2 = (4.034240938990372, 3.803718700041856, -0.8546365899632317).

      Estos centros del triángulo han sido incorporados (02/10/2015) a la Enciclopedia como X8160 y X8161.


      Los puntos de contacto de la circunferencia (Z1) con centro en Z1 y que toca a las tres circunferencias Γa, Γb y Γc abarcándolas, forman un triángulo T1aT1bT1c perspectivo con ABC, con centro de perspectividad en X1343 (centro exterior de semejanza de la circunferencia circunscrita y la circunferencia de Lemoine, Peter J. C. Moses, 4/2003).

    T1a = (-a^2(b^2+c^2)(a^2+b^2+c^2)S csc ω+a^2(b^2+c^2)(a^4-a^2 b^2+b^4-a^2c^2-b^2c^2+c^4) :
    -b^2S csc ω(a^4-2a^2b^2-a^2c^2-2b^2c^2+c^4)+b^2(2a^2b^4-b^6+a^4c^2+4a^2b^2c^2+2b^4c^2+a^2c^4) :
    -c^2(a^4-a^2b^2+b^4-2a^2c^2-2b^2c^2)S csc ω+c^2(a^4b^2+a^2b^4+4a^2b^2c^2+2a^2c^4+2b^2c^4-c^6)).

      Los puntos de contacto de la circunferencia (Z2) con centro en Z2 y que toca a las tres circunferencias Γa, Γb y Γc permaneciendo en el interior de ellas, forman un triángulo T2aT2bT2c perspectivo con ABC, con centro de perspectividad en X1342 (centro interior de semejanza de la circunferencia circunscrita y la circunferencia de Lemoine, Peter J. C. Moses, 4/2003).

    T2a = (a^2(b^2+c^2)(a^2+b^2+c^2)S csc ω+a^2(b^2+c^2)(a^4-a^2 b^2+b^4-a^2c^2-b^2c^2+c^4) :
    b^2S csc ω(a^4-2a^2b^2-a^2c^2-2b^2c^2+c^4)+b^2(2a^2b^4-b^6+a^4c^2+4a^2b^2c^2+2b^4c^2+a^2c^4) :
    c^2(a^4-a^2b^2+b^4-2a^2c^2-2b^2c^2)S csc ω+c^2(a^4b^2+a^2b^4+4a^2b^2c^2+2a^2c^4+2b^2c^4-c^6)).


      Las ecuaciones baricéntricas de las circunferencias (Z1) y (Z2), denominadas por Clark Kimberling "Outer Montesdeoca-Lemoine Circle" y "Inner Montesdeoca-Lemoine Circle", son respectivamente:

    a^2yz+b^2zx+c^2xy+ K1 (x+y+z)(b^2c^2x+a^2c^2y+a^2b^2z)=0,
    a^2yz+b^2zx+c^2xy+ K2 (x+y+z)(b^2c^2x+a^2c^2y+a^2b^2z)=0.



      Esta ecuaciones se pueden poner en una forma enormemente más simplificadas según la información adicional, aportada por Peter Moses (16/10/2015, Comunicación personal):

    a^2 y z + b^2 z x + c^2 x y ± Sin ω/(2 S)(x + y + z) (b^2 c^2 x + c^2 a^2 y + a^2 b^2 z) =0

    The Montesdeoca-Lemoine circles are members of the Schoute coaxal system, which contains the circumcircle, Brocard circle, Lemoine axis, ... X(8160) = a^2 (SA + S Cot[th]):: where th = ArcTan[-(2 + Cos[w]) Csc[w]] giving simple barys:
    X(8160) = a^2 (SA - (S Sin[w]) / (2 + Cos[w]))::
    X(8160) = (1 + 2 Sec[w]) X[3] - X[6].
    The outer Montesdeoca-Lemoine circle has A-power -b^2 c^2 Sin[w] / (2 S)
    It has radius R (2 Cos[w] + 1) / 2

    Similarly,
    X(8161) = a^2 (SA + S Cot[th]):: where th = ArcTan[(2 - Cos[w]) Csc[w]] giving simple barys:
    X(8161) = a^2 (SA + (S Sin[w]) / (2 - Cos[w]))::
    X(8161) = (1 - 2 Sec[w]) X[3] - X[6].
    The inner Montesdeoca-Lemoine circle has A-power b^2 c^2 Sin[w] / (2 S)
    It has radius R (2 Cos[w] - 1) / 2


  • lunes, 28 de septiembre del 2015

    Par de triángulos cevianos y centros del triángulo asociados


    (ADGEOM #2818, Dao Thanh Oai)

      Sean ABC un triángulo, y los triángulos cevianos de dos puntos P(u:v:w) y Q(p:q:r), en coordenadas baricéntricas.
      Denotamos por abc y acb las rectas paralelas por A a PbQc y a PcQb, respectivamente, (abc, acb son las rectas paralelas por A a los lados del cuadrivértice PbPcQbQc, que no coinciden con lados de los triángulos ABC, PaPbPc y QaQbQc).
      Similarmente se definen las paralelas por B, bca, bac y las paralelas por C, cab y cba.
      Los vértices (aparte de B y C) del cuadrilátero bacbcacabcba los denotamos por:
      A11=bac∩cab, A12=bac∩cba, A21=bca∩cab, A22=bca∩cba.
      Con un procedimiento similar, se definen los puntos B11, B12, B21, B22 y C11, C12, C21, C22.
    ( Mostrar/Ocultar figura )
      ADGEOM2818PQ.png


      • Los triángulos A11B11C11 y A22B22C22 son perspectivos (centro de perspectividad en el punto del infinito de PQ) y cada uno de ellos con ABC (centros de perspectividad, D1 y D2). Las rectas PQ y D1D2 son paralelas.
      • Los triángulos A11B12C21 y A22B21C12 son perspectivos (centro de perspectividad Ya) y cada uno de ellos con ABC (centros de perspectividad, A1 y A2). Los puntos A1, A2, Ya están alineados.
      • Los triángulos A21B11C12 y A12B22C21 son perspectivos (centro de perspectividad Yb) y cada uno de ellos con ABC (centros de perspectividad, B1 y B2). Los puntos B1, B2, Yb están alineados.
      • Los triángulos A12B21C11 y A21B12C22 son perspectivos (centro de perspectividad Yc) y cada uno de ellos con ABC (centros de perspectividad, C1 y C2). Los puntos C1, C2, Yc están alineados.
      • Las cuatro rectas D1D2, A1A2, B1B2 y C1C2 son concurrentes en un punto R.

      Las coordenadas del punto R son:

         (p u (q + r) (v + w) (q r u (u + v + w) + p v w (p + q + r ) + 2 q r v w) : ... : ... ).

      Algunos casos particulares de índices de ternas {P=X(i),Q=X(j),R=X(k)}: {1,75,1215}, {2,4,3520}, {2,8,4861}, {2,4240,402}, {2,5855,891}, {2,7539,7571}, {6, 76, 4074}.
      Para los triángulos cevianos del incentro y baricentro el punto R es:

    (a(b+c)(a^2+a(b+c)+5b c) : ... : ...),

    que tiene coordenadas trilineales exactas en el triángulo de ETC: (2.355851486426486, 1.734678041254997, 1.352417459611003).
      Aunque no tiene sentido la construcción de R, cuando P=Q, por su definición algebraica se tiene: {P,P,R="crosspoint" de P y el baricentro}.

      • El triángulo YaYbYc es perspectivo con ABC en un punto Y.

      Las coordenadas del punto Y son:

              (pu(q+r)(v+w)/(u(q+r)-p(v+w)) : ... : ...).

      Algunos casos particulares de índices de ternas {P=X(i),Q=X(j),Y=X(k)}:
    {1,2,4674}, {1,511,5360}, {1,513,512}, {1,514,661}, {1,522,4041}, {1,523,4705}, {1,876,4010}, {1,900,4730}, {1,1022,4145}, {1,3667,4729}, {1,3900,4524}, {1,4777,4770}, {1,4778,4822}, {1,4977,4983}, {2,4,74}, {2,7,1156}, {2,8,1320}, {2,69,895}, {2,542,542}, {2,752,752}, {2,888,888}, {2,2775,2775}, {2,2807,2807}, {2,2839,2839}, {2,3308,3308}, {2,4778,4778}, {2,5466,690}, {2,5468,690}, {2,5855,891}, {2,6366,6366}, {2,6548,900}.
      {P,Q=conjugado isotómico de P, Y}: {4,69,248}, {7,8,294}, {30,1494,3163}, {99,523,115}, {190,1086},{512,670,1084}, {513,668,1015}, {514,1086}, {518,2481,6184}, {519,903,4370}, {522,664,1146}, {524,671,2482}, {2479,?,2455}, {2480,?,2454}.

      • Fijado un punto Q=(p:q:r), los puntos Ya, Yb, Yc están alineados si solo si P queda sobre la cúbica:
          pqr(x(y^2+z^2)+y(z^2+x^2)+z(x^2+y^2) + (p^2(q+r)+q^2(r+p)+r^2(p+q)+4pqr)xyz =0.

      Estas cúbicas tiene asíntotas paralelas a los lados de ABC ((p+q)(p+r)(q+r)x + pqry + pqrz=0, ...). El centro de homotecia de ABC y el triángulo que ellas determinan es el baricentro.
    ( Mostrar/Ocultar figura )
      ADGEOM2818cubica.png


  • miércoles, 23 de septiembre del 2015

    "Conic through six Circumcenters"


    (AoPS, TelvCohl)

     Given a triangle ▵ABC with circumcenter O and a point P . Let ▵DEF be the circumcevian triangle of P wrt. ▵ABC. Let A_1, A_2 be the intersection of BC with FD, DE, respectively (define B_1, B_2, C_1, C_2 similarly). Let O_1, O_2, O_3, O_4, O_5, O_6 be the circumcenter of ▵AC_1F, ▵BC_2F, ▵BA_1D, ▵CA_2D, ▵CB_1E, ▵AB_2E, respectively. Let C_T be the conic with center T passing through A_1, A_2, B_1, B_2, C_1, C_2. Let C_S be the conic with center S passing through O_1, O_2, O_3, O_4, O_5, O_6. Let (T_1, T_2), (S_1, S_2) be the focus of C_T, C_S, respectively. Prove that S is the midpoint of OT and T_1T_2 ⊥ S_1S_2.
    ( Mostrar/Ocultar figura )
      d60861777a75daa501911b9b2446445c3e7a78.png


      Sean ABC un triángulo, P(u:v:w) un punto, en coordenadas baricéntricas, y DEF el triángulo circunceviano de P.
    La ecuación de la cónica (CT) que pasa por los puntos A1=BC∩FD, A2=BC∩DE, B1=CA∩DE, B2=CA∩EF, C1=AB∩EF, C2=AB∩FD es:
      El centro de esta cónica es:
       T (a^4u v w - a^2u(2(c^2 v^2+b^2 w^2)+(b^2+c^2)v w) - 2b^2c^2u^2(v+w) : ... : ...).

      El centro S de la cónica (CS) que pasa por los puntos O1, O2, O3, O4, O5, O6 (circuncentros de los triángulos AC1F, BC2F, BA1D, CA2D, CB1E, AB2E, respectivamente) tiene primera coordenada baricéntrica:

      -a^8 u v w
      +a^6 (c^2 v+b^2 w) (3 u v+3 u w+2 v w)
      +a^4 (3 b^2 c^2 u^2 v-4 b^2 c^2 u v^2-4 c^4 u v^2+3 b^2 c^2 u^2 w-3 b^4 u v w-2 b^2 c^2 u v w-3 c^4 u v w-2 b^2 c^2 v^2 w-2 c^4 v^2 w-4 b^4 u w^2-4 b^2 c^2 u w^2-2 b^4 v w^2-2 b^2 c^2 v w^2)
      -a^2 u (v+w) (4 b^4 c^2 u+4 b^2 c^4 u-b^4 c^2 v+2 b^2 c^4 v-c^6 v-b^6 w+2 b^4 c^2 w-b^2 c^4 w)
      + b^2 (b-c)^2 c^2 (b+c)^2 u^2 (v+w)

      Si el punto P recorre la tripolar del simediano (X6), la cónica (CT) degenera en un producto de rectas que se cortan en X6 y la cónica (CS) degenera en un producto de rectas que se cortan en X182.
      Si el punto P recorre la quíntica circular, con puntos dobles en los vértices de ABC, de ecuación

    2 b^2 c^4 x^3 y^2+2 a^2 c^4 x^2 y^3-a^2 b^2 c^2 x^3 y z+2 b^4 c^2 x^3 y z+2 b^2 c^4 x^3 y z-a^4 c^2 x^2 y^2 z+4 a^2 b^2 c^2 x^2 y^2 z-b^4 c^2 x^2 y^2 z+2 a^2 c^4 x^2 y^2 z+2 b^2 c^4 x^2 y^2 z-c^6 x^2 y^2 z+2 a^4 c^2 x y^3 z-a^2 b^2 c^2 x y^3 z+2 a^2 c^4 x y^3 z+2 b^4 c^2 x^3 z^2-a^4 b^2 x^2 y z^2+2 a^2 b^4 x^2 y z^2-b^6 x^2 y z^2+4 a^2 b^2 c^2 x^2 y z^2+2 b^4 c^2 x^2 y z^2-b^2 c^4 x^2 y z^2-a^6 x y^2 z^2+2 a^4 b^2 x y^2 z^2-a^2 b^4 x y^2 z^2+2 a^4 c^2 x y^2 z^2+4 a^2 b^2 c^2 x y^2 z^2-a^2 c^4 x y^2 z^2+2 a^4 c^2 y^3 z^2+2 a^2 b^4 x^2 z^3+2 a^4 b^2 x y z^3+2 a^2 b^4 x y z^3-a^2 b^2 c^2 x y z^3+2 a^4 b^2 y^2 z^3 =0,

    los puntos O1, O2, O3, O4, O5, O6 están alineados con P.

    ( Mostrar/Ocultar figura )
      AoPSc6t48f6h1144334.png


    Observaciones adicionales:

      • Si P es el incentro, el centro de la cónica (CT) es X1001 (punto medio de X1 y X9).
      • Si P es el circuncentro, el centro de ambas cónicas (CT) y (CS) es el circuncentro.
      • Las cónicas (CT) y (CS) tienen centros X6 y X182, respectivamente si P es el ortocentro.
      • Si P=X6, la cónica (CT) tiene ecuación:
        2 b^4 c^4 x^2+2 a^4 c^4 y^2+2 a^4 b^4 z^2-5 a^2 b^2 c^4 x y-5 a^2 b^4 c^2 x z-5 a^4 b^2 c^2 y z=0,
    y su centro es X5024.
    X5024 en ETC
      Let A'B'C' be the circumcevian triangle of the symmedian point (Lemoine point), X(6). The sidelines BC, CA, AB meet the sidelines of B'C', C'A', A'B' in 9 points, of which 6 do not lie on the trilinear polar of K; barycentrics for the 6 points are 0 : b^2 : 2c^2, 0 : 2b^2 : c^2, 2a^2 : 0 : c^2, a^2 : 0 : 2c^2, a^2 : 2b^2 : 0, 2a^2 : b^2 : 0. The 6 points lie on a conic with center X(5024) and equation
    2(b^4c^4x^2 + c^4a^4y^2 + a^4b^4z^2) -5a^2b^2c^2(a^2yz + b^2zx + c^2xy) = 0.

      • Si P=X523 (conjugado isogonal del foco de la parábola de Kiepert), el centro de la cónica (CS) es el baricentro.
      • Si P es X2574 o X2575 (conjugados isogonales de los puntos en los que la recta de Euler corta a la circunferencia circunscrita), el centro de la cónica (CS) es el circuncentro.
      • Si P es X1499, el centro de la cónica (CS) es el X7618.

    ∗  ∗  ∗  ∗  ∗  ∗  ∗  ∗  ∗  ∗ 



      Las circunferencias circunscritas a los triángulos AFC1 y BFC2 se vuelven a cortar en F'.
      Las circunferencias circunscritas a los triángulos BFC2 y BDA1 se vuelven a cortar en B'.
      Las circunferencias circunscritas a los triángulos BDA1 y CDA2 se vuelven a cortar en D'.
      Las circunferencias circunscritas a los triángulos CDA2 y CEB1 se vuelven a cortar en C'.
      Las circunferencias circunscritas a los triángulos CEB1 y AEB2 se vuelven a cortar en E'.
      Las circunferencias circunscritas a los triángulos AEB2 y AFC1 se vuelven a cortar en A'.

      Entonces, los seis puntos A', B', C', D', E' y F' están en una misma circunferencia Γ. Además, las rectas A'D', B'E', C'F' concurren en P.

    ( Mostrar/Ocultar figura )
      AoPScircle.png

      Las rectas AA', BB', CC', DD', EE', FF' concurren en el punto R de coordenadas baricéntricas:

          R=(u (c^2 v^2+b^2 w^2+(-a^2+b^2+c^2)v w)/(c^2v+b^2 w) : .. : ...).

      Algunos pares {i,j} → {P,R}={Xi,Xj}:
    {1,21},{4,54},{6,574},{64,1204},{187,574},{512,32},{513,56},{523,4},{924,1147},{2574,3},{2575,3},{3413,6177},{3414,6178},{3513,55},{3514,55},{3566,6337},{3900,7367}.

    CASOS PARTICULARES de circunferencias Γ por A', B', C', D', E', F':

      • Si P es el incentro, Γ es la circunferencia de diámetro X80X6265; su centro es X5 y su ecuación:
    c^2xy+b^2xz+a^2yz+(x+y+z)((b+c)(a^2-b^2+bc-c^2)x-(a+c)(a^2-b^2-ac+c^2)y-((a+b)(a^2-ab+b^2-c^2)z)/(2(a+b+c))=0.
      • Si P es el baricentro, Γ es la circunferencia de ecuación: que pasa por X67 y centro es el punto:

    (a^8 (b^2+c^2)-2 b^2 c^2 (b^2-c^2)^2 (b^2+c^2)+a^6 (b^4-4 b^2 c^2+c^4)-a^4 (b^6-6 b^4 c^2-6 b^2 c^4+c^6)-a^2 (b^8-3 b^6 c^2-4 b^4 c^4-3 b^2 c^6+c^8) : ... : ...),

    que tiene coordenadas trilineales exactas en el triángulo de ETC: (0.9605392001865930, -1.144992027768490, 3.990025485661060).
      • Si P es el circuncentro, Γ es la circunferencia circunscrita a ABC.
      • Si P es el ortocentro, Γ es la circunferencia de centro X6102 (19th Hatzipolakis-Montesdeoca Point) y que pasa por X265, X7722, su ecuación es:
      • Si P es el simediano, Γ es la circunferencia de ecuación: que pasa por X671 (punto de intersección de la hipérbola de Kiepert y la elipse circunscrita de Steiner) y centro es el punto:

    (a^2 (a^4 (b^2+c^2)-a^2 (3 b^4+7 b^2 c^2+3 c^4)+2b^6+2c^6-b^4 c^2-b^2 c^4) : ... : ...),

    que tiene coordenadas trilineales exactas en el triángulo de ETC: (-0.1748147694481922, 0.6007629779554242, 3.305435390760559).

    NOTA

      César Lozada (ADGEOM #2819) publica que la circunferencia Γ pasa por el conjugado antigonal del conjugado isogonal de P y su centro Q tiene primera coordenada baricéntrica:

    a^2 b^4 c^4 u^3 v^2-b^6 c^4 u^3 v^2+a^2 b^2 c^6 u^3 v^2+2 b^4 c^6 u^3 v^2-b^2 c^8 u^3 v^2+a^4 b^2 c^4 u^2 v^3-a^2 b^4 c^4 u^2 v^3+a^4 c^6 u^2 v^3+2 a^2 b^2 c^6 u^2 v^3-a^2 c^8 u^2 v^3-a^4 b^4 c^2 u^3 v w+2 a^2 b^6 c^2 u^3 v w-b^8 c^2 u^3 v w-a^4 b^2 c^4 u^3 v w+b^6 c^4 u^3 v w+2 a^2 b^2 c^6 u^3 v w+b^4 c^6 u^3 v w-b^2 c^8 u^3 v w-a^6 b^2 c^2 u^2 v^2 w+2 a^4 b^4 c^2 u^2 v^2 w-a^2 b^6 c^2 u^2 v^2 w+2 a^4 b^2 c^4 u^2 v^2 w+2 a^2 b^4 c^4 u^2 v^2 w-a^2 b^2 c^6 u^2 v^2 w+2 a^6 c^4 u v^3 w+2 a^4 b^2 c^4 u v^3 w-2 a^4 c^6 u v^3 w+a^2 b^6 c^2 u^3 w^2-b^8 c^2 u^3 w^2+a^2 b^4 c^4 u^3 w^2+2 b^6 c^4 u^3 w^2-b^4 c^6 u^3 w^2-a^6 b^2 c^2 u^2 v w^2+2 a^4 b^4 c^2 u^2 v w^2-a^2 b^6 c^2 u^2 v w^2+2 a^4 b^2 c^4 u^2 v w^2+2 a^2 b^4 c^4 u^2 v w^2-a^2 b^2 c^6 u^2 v w^2+a^8 c^2 v^3 w^2-a^6 b^2 c^2 v^3 w^2-a^6 c^4 v^3 w^2+a^4 b^6 u^2 w^3-a^2 b^8 u^2 w^3+a^4 b^4 c^2 u^2 w^3+2 a^2 b^6 c^2 u^2 w^3-a^2 b^4 c^4 u^2 w^3+2 a^6 b^4 u v w^3-2 a^4 b^6 u v w^3+2 a^4 b^4 c^2 u v w^3+a^8 b^2 v^2 w^3-a^6 b^4 v^2 w^3-a^6 b^2 c^2 v^2 w^3.

  • martes, 22 de septiembre del 2015

    Centros del triángulo asociados a cuadrivértices


      Sean ABC un triángulo y A'B'C' el triángulo de contacto interior. La circunferencia que pasa por B, C y es tangente a la circunferencia inscrita vuelve, a cortar a los lados AC y AB en los puntos Ba y Ca.
      Los puntos diagonales del cuadrivértice B'C'CaBa son A, A1, A2 (A1 en el interior de ABC).
    ( Mostrar/Ocultar figura )
      HG220915.png


      Sea a1 la diagonal AA1 del cuadrivértice B'C'CaBa. Definimos b1 y c1 cíclicamente. Las tres rectas a1, b1 y c1 concurren en el centro X269, de coordenadas baricéntricas:

    ( a/(-a+b+c)^2:b/(a-b+c)^2:c/(a+b-c)^2).

    ( Mostrar/Ocultar figura )
      HG220915a1.png


    A2 es el punto diagonal del cuadrivértice B'C'CaBa que queda en el exterior de ABC. Se definen los puntos diagonales B2 y C2, cíclicamente.
      Las rectas a2=AA2, b2=BB2 y c2=CC2 delimitan un triángulo A''B''C''.

      Los triángulos ABC y A''B''C'' son perspectivos, con centro de perspectividad X269.
      El triángulo de contacto interior A'B'C' es perspectivo con A''B''C'', con eje de perspectividad la recta que pasa por A2, B2, C2 y centro de perspectividad con coordenadas baricéntricas:

    Q ( (a (a^3+a^2 (b+c)-(b-c)^2 (b+c)-a (b+c)^2))/(-a+b+c)^2 : ... : ...)

    que tiene coordenadas trilineales exactas en el triángulo de ETC: (-0.07978231049636353, -0.1737153340003383, 3.797751548752166).
    ( Mostrar/Ocultar figura )
      HG220915a2.png


      Sea P1a el baricentro del cuadrivértice B'C'CaBa (QA-P1: Quadrangle Centroid, EQF)), Definimos P1b y P1c cíclicamente. El triángulo P1aP1bP1c es perspectivo con ABC y el centro de perspectividad tiene coordenadas baricéntricas:

    P1 ( a^2(a^2+b^2+c^2-2 a (b+c)-6 b c) : ... : ...)

    que tiene coordenadas trilineales exactas en el triángulo de ETC: (1.386660855692907, 1.602816812609743, 1.891024755165523).
    ( Mostrar/Ocultar figura )
      HG220915P1.png




  • lunes, 21 de septiembre del 2015

    Triángulo perspectivo con el triángulo de contacto interior


    Art of Problem Solving

      1)  ABC is a triangle.
      2)  The incircle (I) of ABC meet BC,CA,AB at D,E,F respectively.
      3)  ρ denotes a circle passing throw B,C that is tangent to (I).
      4)  ρ∩AC=S, ρ∩AB=T
      5)  J denotes the A-excenter of AST.
      6)  SJ∩EF=K, TJ∩EF=L.
      7)  O is the circumcenter of JKL .
      8)  M is midpoint of JO .
      Claim:
    The triangle formed by reflections of M with respect to AI,BI,CI is perspective with DEF.

      La circunferencia inscrita a ABC toca a los lados en los puntos (coordenadas baricéntricas):
        D (0:a+b-c:a-b+c), E (a+b-c:0:-a+b+c), F (a-b+c:-a+b+c:0).
      DEF es el triángulo contacto interior.

      La circunferencia (ρ) que pasa por B y C y es tangente a la circunferencia inscrita, toca a ésta en el punto R, que se determina hallando el eje radical ((-a+b+c)^2 x+(a-b+c)^2 y+(a+b-c)^2 z=0) de las circunferencias inscrita y circunscrita, e intersecándolo con la recta BC.

          R (0:(a+b-c)^2:-(a-b+c)^2).
      La segunda tangente desde R a la circunferencia inscrita interseca a ésta en el punto D' de tangencia con la circunferencia (ρ).
        D' (-4a^2(a+b-c)(a-b+c) : (a-b-c)(a+b-c)^3 : (a-b-c)(a-b+c)^3).

      La ecuación de la circunferencia (ρ) es:
    c^2 x y+b^2 x z+a^2 y z-(x+y+z)((a^3-3a^2(b+c)-(b-c)^2(b+c)+a(3b^2+2b c+3c^2)x)/(8a)=0.

      Esta circunferencia corta a los lados AC y AB, respectivamente, en los puntos:
    S (a^3-3 a^2 (b+c)-(b-c)^2 (b+c)+a (-5 b^2+2 b c+3 c^2) : 0 : -a^3+3 a^2 (b+c)+(b-c)^2 (b+c)-a (3 b^2+2 b c+3 c^2)),
    T (a^3-3 a^2 (b+c)-(b-c)^2 (b+c)+a (3 b^2+2 b c-5 c^2) : -a^3+3 a^2 (b+c)+(b-c)^2 (b+c)-a (3 b^2+2 b c+3 c^2) : 0).

      El A-excentro J del triángulo AST es la intersección de la bisectriz interior en A (cy-bz=0) con la bisectriz exterior en T,
    (a-b-c) (a^2+(b-c)^2-2 a (b+c)) x+(a-b+c) (a^2+b^2-c^2-2 a (b+2 c)) y+(a^3+a^2 (5 b-3 c)-(b-c)^2 (b+c)+a (-5 b^2+2 b c+3 c^2)) z=0.

       J (-2 a (b-c)^2+a^2 (b+c)+(b-c)^2 (b+c),-b (a^2+(b-c)^2-2 a (b+c)),-c (a^2+(b-c)^2-2 a (b+c))):

      Los puntos K=EF∩JS y L=EF∩JT tienen coordenadas:
    K (2 a c (-b + c) + (b - c)^2 (b + c) - a^2 (b + 3 c) : b (a^2 - b^2 - 2 a c + c^2) : c (3 a^2 + b^2 - c^2 - 2 a (2 b + c))),
    L (2 a b (b-c)+(b-c)^2 (b+c)-a^2 (3 b+c) : b (3 a^2-b^2+c^2-2 a (b+2 c)) : c (a^2-2 a b+b^2-c^2)).

      El centro de la circunferencia circunscrita a LKL es:
    Oa (-a^3 (b+c)+a (b-c)^2 (b+c)+(b^2-c^2)^2-a^2 (b^2+6 b c+c^2) : -b (-a^3+a^2 (-b+c)+(b-c) (b+c)^2+a (b^2+6 b c+c^2)) : -c (-a^3+a^2 (b-c)-(b-c) (b+c)^2+a (b^2+6 b c+c^2))).

      El punto medio del segmento JOa es:
    M (a (6 a b c+a^2 (b+c)-(b-c)^2 (b+c)) : b (-a^3+a^2 c+c (b^2-c^2)+a (b^2+6 b c+c^2)) : c (-a^3+a^2 b-b^3+b c^2+a (b^2+6 b c+c^2))).

      Los puntos simétricos de M respecto a las bisectrices interiores de ABC son:

    A1 (-a^3 (b+c)+a (b-c)^2 (b+c)-(b^2-c^2)^2+a^2 (b^2-8 b c+c^2) : b (a^3-a^2 b+b^3-b c^2-a (b^2+6 b c+c^2)) : c (a^3-a^2 c-b^2 c+c^3-a (b^2+6 b c+c^2))),
        A2 (a (a^3-a^2 b+b^3-b c^2-a (b^2+6 b c+c^2)) : -a^4+a^3 b-(b-c)^2 c (b+c)-a b (b^2+8 b c+c^2)+a^2 (b^2-b c+2 c^2) : -c (6 a b c+a^2 (b+c)-(b-c)^2 (b+c))),
        A3 (a (a^3-a^2 c-b^2 c+c^3-a (b^2+6 b c+c^2)) : b (-6 a b c-a^2 (b+c)+(b-c)^2 (b+c)) : -a^4+a^3 c-b (b-c)^2 (b+c)+a^2 (2 b^2-b c+c^2)-a c (b^2+8 b c+c^2)).

      Los triángulos ABC y A1A2A3 son perspectivos con centro de perspectividad:

    Q (a/ (-a^2 b+b^3-a^2 c-6 a b c-b^2 c-b c^2+c^3): ... : ...)

    que tiene coordenadas trilineales exactas en el triángulo de ETC: (1.985869133396268, 1.916652117958390, 1.397196492522601).
    ( Mostrar/Ocultar figura )
      perspectorAoSP.png


      Los triángulos DEF y A1A2A3 son perspectivos con centro de perspectividad:

    P (a (a^2-b^2-3 b c-c^2) (a^3 b-a^2 b^2-a b^3+b^4+a^3 c+8 a^2 b c+a b^2 c-a^2 c^2+a b c^2-2 b^2 c^2-a c^3+c^4): ... : ...)

    que tiene coordenadas trilineales exactas en el triángulo de ETC: (2.199262982600095, 2.268236144827733, 1.055302697365136).
    ( Mostrar/Ocultar figura )
      perspectorAoSPa.png

         NOTA:

      Si procedemos cíclicamente sobre los vértices de ABC, se obtienen triángulos B1B2B3 y C1C2C3, se tiene A1=B3=C2, A2=B1=C3, A3=B2=C1.


      Sean los centros de circunferencias Ob y Oc, obtenidos de forma similar a Oa.

      El triángulo OaObOc es perspectivo a ABC, con centro de perspectividad X3333 (Punto de Pohoata, en honor al matemático rumano Cosmin Pohoata).
          (a((a + b - c) (a - b + c) (a + b + c)+4abc) : ... : ... )


    X(3333) of ETC: Pohoata Point

      Let I be the incenter, X(1), and let KA be the symmedian point of triangle IBC; define KB, KC cyclically. Let X be the midpoint of segment AI,and define Y, Z cyclically. Then the triangles KAKBKC and XYZ are perspective, and their perspector is X(3333). Contributed by Cosmin Pohoata, April 4, 2008.
    ( Mostrar/Ocultar figura )
      X(3333)a.png



    Journal of Computer-Generated Euclidean Geometry 2009 No 5 : The Pohoata Point

    Theorem 1. The Pohoata Point is the External Center of Similitude of the Hexyl Circle and the Nine-Point Circle of the Intouch Triangle.
    ( Mostrar/Ocultar figura )
      X(3333).png


  • lunes, 7 de septiembre del 2015

    Mas sobre la cúbica PK-transformada de la circunferencia circunscrita



      Sean ABC un triángulo, P un punto y C(P) es la cónica circunscrita a ABC de perspector P.
    AaBbCc es el triángulo circunceviano del centro Ia de la circunferencia A-exinscrita.
    A0 es el segundo punto de intersección de la bisectriz interior por A con C(P).
    B0 es el segundo punto de intersección de la bisectriz exterior por B con C(P).
    C0 es el segundo punto de intersección de la bisectriz exterior por C con C(P).

      La cónica que pasa por Aa, B, C, B0, C0 vuelve a cortar a la bisectriz interior por A en A1.
      La cónica que pasa por Ba, C, A, C0, A0 vuelve a cortar a la bisectriz exterior por B en B1.
      La cónica que pasa por Ca, A, B, A0, B0 vuelve a cortar a la bisectriz exterior por C en C1.

      Los seis puntos Aa, Ba, Ca, A1, B1, C1 es están en una cónica (Ca) (Caso particular de ADGEOM #2730, Dao Thanh Oai).
    ( Mostrar/Ocultar figura )
      ADGEOM2730.png

      Sean Q y Qa los cuartos puntos de intersección de la circunferencia circunscrita con las cónicas C(P) y (Ca), entonces:

    Los puntos Ia, Q y Qa están alineados.


      Denotemos por Pa el segundo punto de intersección de la recta QQa con la cónica (Ca); los puntos Pb y Pc, sobre las cónicas (Cb) y (Cc), se definen cíclicamente. Entonces:

      Los puntos Pa Pb y Pc están en la recta del infinito si y sólo si el punto P está sobre la cúbica imagen de la circunferencia circunscrita mediante la PK-transformación.



  • jueves, 03 de septiembre del 2015

    Cónica asociada a puntos isoconjugados sobre la cúbica de Darboux


      Sean ABC un triángulo, P un punto y Q su conjugado isogonal.
      La recta por A perpendicular a AP corta a BQ en Qab y a CQ en Qac. AP corta a BC en Pa, PaQab corta a AC en Ab y PaQac corta a AB en Ac. Similarmente, se definen los puntos Bc, Ba, Ca, Cb sobre los correspondientes lados de ABC.
    ( Mostrar/Ocultar figura )
      OIMProblema266b.png

      Las rectas AbAc, BcBa, CaCB concurren en Q.
    Si P y Q son puntos isoconjugados sobre la cúbica de Darboux, entonces los puntos Ab, Ac, Bc, Ba, Ca, Cb están en una misma cónica.


    Casos particulares:

     • Si P es el incentro, las coordenadas baricéntricas del centro de la cónica son:

    (a(a-5(b+c)) : b(b-5(c+a)) : c(c-5(a+b)))

    que tiene coordenadas trilineales exactas en el triángulo de ETC: (2.23329327565285, 1.84676174717447, 1.33138637586997).
      Este resultado lo podemos enunciar así:

      Sean ABC un triángulo, DEF el triángulo ceviano del incentro y IaIbIc el triángulo excentral. Los puntos Ab=AC∩DIb, Ac=AB∩DIc, Bc=BA∩EIc, Ba=BC∩EIa, Ca=CB∩FIa, Cb=CA∩FIb están en la cónica de ecuación baricéntrica:
          2b^2c^2x^2+2a^2c^2y^2+2a^2b^2z^2-5a^2bcyz-5ab^2cxz-5abc^2xy=0

    ( Mostrar/Ocultar figura )
      OIMProblema266X1.png

     • Si P es el circuncentro, las coordenadas baricéntricas del centro de la cónica es:

    ((a^4-(b^2-c^2)^2)^2 (a^4-3 b^4-2 b^2 c^2-3 c^4+2 a^2 (b^2+c^2)) : ... : ...)

    que tiene coordenadas trilineales exactas en el triángulo de ETC: (-2.50324398113203, -2.71498061804897, 6.67560982415614).

     • Si P es el ortocentro, las coordenadas baricéntricas del centro de la cónica es:

    (a^2 (-a^10+5 a^8 (b^2+c^2)-10 a^6 (b^2+c^2)^2+(b^2-c^2)^2 (b^6-5 b^4 c^2-5 b^2 c^4+c^6)+2 a^4 (5 b^6+9 b^4 c^2+9 b^2 c^4+5 c^6)+a^2 (-5 b^8+4 b^6 c^2-14 b^4 c^4+4 b^2 c^6-5 c^8)) : ... : ...)

    que tiene coordenadas trilineales exactas en el triángulo de ETC: (3.00519298462766, 1.63557492607827, 1.12133123248666).

  • miércoles, 02 de septiembre del 2015

    Imagen de X204 mediante la inversa de la transformación Mimosa


    Notes just before X(1707)

      The Mimosa transform M(X) of a point X = x : y : z (barycentric) is defined by
    M(X) = a(-a^2SA/x + b^2SB/y + c^2SC/z) : b(a^2SA/x - b^2SB/y + c^2SC/z) : c(a^2SA/x + b^2SB/y - c^2SC/z).
    and the inverse Mimosa transform M-1(X), by
    M-1(X) = a SA/ (bz+cy) : b SB/ (cx+az) : c SC/(ay+bx).

      The Zosma transform of a point X = x : y : z is the isogonal conjugate of the inverse Mimosa transform of X, given by barycentrics
    a (bz+cy)/SA : b(cx+az)/SB : c(ay+bx)/SC .

      As with other ETC names, Mimosa and Zosma are names of two stars.

      Beta Crucis, also known as Mimosa or Becrux, is the second brightest star in the constellation Crux (after Alpha Crucis or Acrux) and is the 19th brightest star in the night sky. In the time of the ancient Greeks and Romans it was visible north of 40° due to the precession of equinoxes, and these civilizations regarded it as part of the constellation Centaurus. Its modern name, Mimosa, which is derived from the Latin word for "actor", may come from the flower of the same name

      Delta Leonis (δ Leo) is a star in the zodiac constellation of Leo. It has the traditional names Zosma (or Zozma) and Duhr. Rare spellings include Zozca, Zosca, Zubra, and Dhur. The name Zosma means girdle in ancient Greek, referring to the star's location in its constellation, on the hip of the lion.


      Sea ABC un triángulo y X20 el punto de De Longchamps (reflexión del ortocentro en el circuncentro). Las circunferencias de diámetros AX20, BX20, CX20 vuelven a cortar a la circunferencia circunscrita a ABC en los puntos A1, B1, C1, respectivamente. Denotamos por ta, tb, tc las tangentes en A1, B1, C1 a estas tres circunferencias.
    ( Mostrar/Ocultar figura )
      INVERSE MIMOSA TRANSFORM OF X(204).png
    Descargar fichero GeoGebra

      Las tangentes ta, tb, tc delimitan un triángulo A'B'C' perspectivo con ABC. El centro de perspectividad es la imagen mediante la inversa de la transformación Mimosa de X204 (cociente ceviano del incentro y el punto de Clawson, i.e. el centro de perspectividad del triángulo ceviano de X1 y del triángulo anticeviano de X19).


      Coordenadas baricéntricas del centro de perspectividad de ABC y A'B'C':

    (a^2/(a^6+a^2(b^2-c^2)^2-2(b^2-c^2)^2(b^2+c^2)) : ... : ... ),

    que tiene coordenadas trilineales exactas en el triángulo de ETC: (-7.606380614628, -7.651832717298, 12.44872433910), y es el conjugado isogonal de X1853 (Zosma transformado de X204).

      Una propiedad adicional de esta configuración es: Si D, L son los A-vértices de los triángulos pedales de X20 y de su conjugado isogonal (X64), respectivamente, la circunferencia circunscrita a A1DL es tangente en A1 a la circunferencia de diámetro AX20.

  • lunes, 31 de agosto del 2015

    Centro del triángulo en IMO


    IMO 2015 Tailandia

    Problem 3. Let ABC be an acute triangle with AB>AC. Let Γ be its circumcircle, H its orthocenter, and F the foot of the altitude from A. Let M be the midpoint of BC. Let Q be the point on Γ such that ∠HQA=90°, and let K be the point on Γ such that ∠HKQ=90°. Assume that the ponts A, B, C, K and Q are all different, and lie on Γ in this order.

    Prove that the circumcircles of triangles KQH and FKM are tangent to each other.


      Sea ABC un triángulo de ortocentro H. La circunferencia de diámetro AH vuelve a cortar a la circunferencia circunscrita en el punto A1. La circunferencia de diámetro A1H vuelve a cortar a la circunferencia circunscrita en el punto A2. Se denota por ta la tangente a esta última circunferencia en A2. Similarmente, procediendo cíclicamente, las tangentes tb y tc.
    ( Mostrar/Ocultar figura )
      IMO2015TailandiaProblem3.png
    Descargar fichero GeoGebra

      Las tangentes ta, tb y tc delimitan un triángulo perspectivo con ABC.


    el centro de perspectividad tiene primera coordenada baricéntrica:

    (a^4-2 a^2 b^2+b^4+2 a^2 c^2-c^4) (a^4+2 a^2 b^2-b^4-2 a^2 c^2+c^4)/((a^2-b^2-c^2)(3 a^12-2 a^10 b^2-15 a^8 b^4+20 a^6 b^6+5 a^4 b^8-18 a^2 b^10+7 b^12-2 a^10 c^2+30 a^8 b^2 c^2-20 a^6 b^4 c^2-56 a^4 b^6 c^2+62 a^2 b^8 c^2-14 b^10 c^2-15 a^8 c^4-20 a^6 b^2 c^4+102 a^4 b^4 c^4-44 a^2 b^6 c^4-7 b^8 c^4+20 a^6 c^6-56 a^4 b^2 c^6-44 a^2 b^4 c^6+28 b^6 c^6+5 a^4 c^8+62 a^2 b^2 c^8-7 b^4 c^8-18 a^2 c^10-14 b^2 c^10+7 c^12)),


    con coordenadas trilineales exactas en el triángulo de ETC: (0.834446774988, -0.0356537774527, 3.28021858553)

  • sábado, 29 de agosto del 2015

    Caracterización de X110


    (ADGEOM #2703)

    ETC #110
    Some characterizations of X(110)

    • The focus of Kiepert parabola (tangent to the sides of the triangle, the line at infinity and the line -the perspectrix of a reference triangle and its tangential triangle-).

    • The center of the Stammler hyperbola (rectangular hyperbola that passes through the excenters, and the vertices of the tangential triangle).

    • The fourth intersection of the circumcircle with the MacBeath circumconic (the isogonal transformation of the orthic axis -the perspectrix of the orthic triangle and reference triangle-).

    • The fourth intersection of the circumcircle with the Johnson circumconic (circumconic centered at the nine-point center).

    • Euler reflection point. If the Euler line is reflected in every side of triangle ABC, then the three reflections concur in X(110).

    • Trilinear pole of the Brocard axis (the line KO passing through the symmedian point K and circumcenter O of a triangle).

    • The J-point (*) of Lemoine line. The Lemoine line cross the sides at P, Q, R respectively. The J-point of Lemoine's line can be located as follows. From A draw a line parallel to PQR cutting the circle at U. Then UP crosses the circle again at J.

    • The O-point (*) of Euler line. Draw AU parallel to the Euler line cutting the circle at U; then draw UO perpendicular to BC cutting the circle at O-point of Euler line.

    • The O'-point (*) of the orthocenter on Euler line. Any point S be taken on the given line that cross the sides at P, Q, R respectively, and if AS, BS, CS meet the circumcircle at U', V', W', then U'P, V'Q, W'R are concurrent at a O'-point of S on given line.

    • The O'-point (*) of the symmedian point on Brocard axis.

    • The O'-point (*) of the centroid on the line joining the symmedian point to the centroid.

    • The O'-point (*) of the inverse point of the incenter on the line joining that point to the incenter.

    • Inverse of the centroid in the Brocard circle (circle having the line segment connecting the circumcenter O and symmedian point K of a triangle ABC as its diameter).

    • Circumcircle-antipode of X(74). The point X(74) is the fourth intersection of the circumcircle with the isogonal transformation of Euler line (Jerabek hyperbola).

    Theorem(*).- The O-point of any straight line is diametrically opposite to the point of intersection of the isogonal transformation of the given line with the circumcircle.

    • (**) Let P be a point, other than X(4), on Euler line. Let A' be the reflection of P in BC, and define B', C' cyclically. The circumcircles of AA'B', BC'A', and CA'B' concur in X(110).

    • (**) Let Q be a point on line X(2)X(6) other than X(2). Let A'B'C' be the cevian triangle of Q. Let A" be the {B,C}-harmonic conjugate of A' (or equivalently, A" = BC∩B'C'), and define B", C" cyclically. The circumcircles of AB"C", BC"A", CA"B" concur in X(110).

    • (**) Let La, Lb, Lc be the lines through A, B, C, resp. parallel to the Euler line. Let Ma, Mb, Mc be the reflections of BC, CA, AB in La, Lb, Lc, resp. Let A' = Mb∩Mc, and define B', C' cyclically. Triangle A'B'C' is inversely similar to, and 3 times the size of, ABC. Let A"B"C" be the reflection of A'B'C' in the Euler line. The triangle A"B"C" is homothetic to ABC, with center of homothety X(125) and centroid X(110). (See Hyacinthos #16741/16782, Sep 2008.)

    • (**) Let A', B', C' be the intersections of the Lemoine axis and lines BC, CA, AB, resp. The circumcircles of AB'C', BC'A', CA'B' concur in X(110).

    • (**) Let A', B', C' be the intersections of line X(36)X(238) and lines BC, CA, AB, resp. The circumcircles of AB'C', BC'A', CA'B' concur in X(110).

    • (**) Let A', B', C' be the intersections of the Euler line and lines BC, CA, AB, resp. Let Oa, Ob, Oc be the circumcenters of AB'C', BC'A', CA'B', resp. The lines AOa, BOb, COc concur in X(110).

    • (**) Let Na be the reflection of X(5) in the perpendicular bisector of BC, and define Nb, Nc cyclically. X(110) = X(2070) of NaNbNc.

    _______________
    (*) J. W. Clawson, "Points on the circumcircle," American Mathematical Monthly 32 (1925) 169-174
    (**) Seven constructions from Randy Hutson, January 29, 2015


      Sean ABC un triángulo y P un punto.
      La perpendicular a BC desde P corta a CA, AB en Ab, Ac, respectivamente. Sea da la mediatriz del segmento AbAc. Similarmente definimos las rectas db y dc. Denotamos por A'B'C' el triángulo acotado por las rectas da, db y dc.

      La circunferencia de los nueve puntos de A'B'C' es tangente a la circunferencia circunscrita a ABC si solo si el punto P está sobre la recta de Euler o sobre la recta que une el baricentro con X64 o sobre una hipérbola con asíntotas paralelas a las rectas anteriores y que pasa por X186.
      Si el punto P está sobre la recta de Euler, el punto de contacto de las dos circunferencias es X110.

    ( Mostrar/Ocultar figura )
      ADGEOM2697X110.png


  • miércoles, 19 de agosto del 2015

    X6146 como centro ortológico de dos pares de triángulos


    (Anopolis #2774, Antreas Hatzipolakis)

    ETC #6146
    Hatzipolakis-Moses-Triangle-to-ABC Orthology Center

     Let A'B'C' be the orthic triangle of ABC. Let Ab be the orthogonal projection of A' onto line AB, and define Bc and Ca cyclically. Let Ac be the orthogonal projection of A' onto line AC, and define Ba and Cb cyclically. Let A'b be the reflection of A in Ab, and let A'c be the reflection of A in Ac . Let Na be the nine-point center of the triangle AA'bA'c, and define Nb and Nc cyclically. The triangle NaNbNc, here named the Hatzipolakis-Moses triangle, is orthologic to ABC and perpective to ABC.
     The ABC-to-NaNbNc center of orthology is X(6145), and the NaNbNc-to-ABC center of orthology is X(6146). The perspector of ABC and NaNbNc is X(54). (Based on notes , Antreas Hatzipolakis and Peter Moses, November 19, 2014; see Hyacinthos #22774)

      The centre of perspective X(54) and the orthologic center X(6145) and X(6146) are on the same line perpendicular to the axis of perspectivity. (Sondat's theorem, V. Thebault, Perspective and Orthologic Triangles and Tetrahedrons. The American Mathematical Monthly vol. 59, No. 1, Jan. 1952, pp. 24 - 28, specifically, on page 25)
    ( Mostrar/Ocultar figura )
      Hyacinthos22771.png
    Descargar fichero GeoGebra


      Sea ABC un triángulo, Ab y Ac las proyecciones ortogonales de A sobre las cevianas del circuncentro por B y C, respectivamente, Ma es el punto medio de AbAc.
      Similarmente se definen los puntos Mb y Mc.

      Los puntos A2 y A3 son las reflexiones de A en las cevianas del circuncentro por B y C, respectivamente, M1 es el punto medio de A2A3.
      Similarmente se definen los puntos M2 y M3.

      Los triángulos MaMbMc y M1M2M3 son ortológicos. El centro de ortología de MaMbMc respecto a M1M2M3 es X6146.
    El centro de ortología de M1M2M3 respecto a MaMbMc es el punto V, centro del triángulo de primera coordenada baricéntrica:

    -2a^10 + 5a^8(b^2+c^2) - 2a^6(2b^4+b^2c^2+2c^4) + 2a^4(b^2-c^2)^2(b^2+c^2) - 2a^2(b^8-b^6c^2-b^2c^6+c^8) + (b^2-c^2)^4(b^2+c^2),

    con número de búsqueda en ETC: 11.66401239277167617436532056

    ( Mostrar/Ocultar figura )
      Anopolis2774IIIX3.png



     •  Si, en vez de tomar las cevianas del circuncentro tomamos las bisectrices, El centro de ortología de MaMbMc respecto a M1M2M3 es X942 ( Hyacinthos 23074).
    El centro de ortología de M1M2M3 respecto a MaMbMc es X1071 (X(1071) appears in Hyacinthos message #3849, Paul Yiu, Sept. 19, 2001).

      En este caso los triángulos MaMbMc y M1M2M3 son además perspectivos, con centro de perspectividad el punto de Gergonne, que está alineado con sus centros ortológicos (teorema de Sondat).
    ( Mostrar/Ocultar figura )
      Anopolis2774IIIX1.png

     • En general, los triángulos MaMbMc y M1M2M3 son "bilógicos" (ortológicos y perspectivos) si sólo si se toman cevianas de un punto P sobre la séptica Q030, que pasa los centros X(1), X(4), X(13), X(14), X(74), X(80).

  • martes, 18 de agosto del 2015

    Centros ortológicos deducidos de reflexiones en las alturas


    (Anopolis #2774, Antreas Hatzipolakis)

      Sea ABC un triángulo, A2 y A3 las reflexiones de A en las alturas por B y C, respectivamente, M1 es el punto medio de A2A3.
      Similarmente se definen los puntos M2 y M3.

      Los triángulos ABC y M1M2M3 son ortológicos. El centro de ortología de ABC respecto a M1M2M3 es X64 (conjugado isogonal del punto de De Longchamps). El centro de ortología de M1M2M3 respecto a ABC es X185 (punto de Nagel del triángulo órtico, cuando ABC es agudo).

    ( Mostrar/Ocultar figura )
      Anopolis2774aX4.png



     •  Si, en vez de tomar las alturas, se toman las cevianas de X74 (conjugado isogonal del punto del infinito de la recta de Euler), el centro de ortología de ABC respecto a M1M2M3 es X64.
     Una construcción de X64 aparece en Emile Lemoine, "Quelques questions se rapportant à l'étude des antiparallèles des côtes d'un triangle", Bulletin de la S. M. F., tome 14 (1886), p. 107-128, concretamente, en la página 111. Este artículo está disponible en Numdam

    (dans page 110)
      ABC est un triangle. En B je mène une perpendiculaire à AB qui coupe AC en ia; en C une perpendiculaire à AC qui coupe AB en ic. La droite ibic se denota por (Ia). De même, les droite (Ib) et (Ic) sont définies.
      Nous appellerons Ia, Ib, Ic les sommets du triangle formé par les droites (Ia), (Ib), (Ic), Ia étant l'intersection des droites (Ib), (Ic),...
      Le deux triangles ABC, IaIbIc sont homologiques; le centre d'homologie a pour coordonnées
    1/(cosA-cosB cosC), 1/(cosB-cosC cosA), 1/(cosC-cosA cosB)
    ( Mostrar/Ocultar figura )
      X(64).png

      Y el centro de ortología de M1M2M3 respecto a ABC es el centro del triángulo de primera coordenada baricéntrica:

    4a^10 - 4a^8(b^2+c^2) + a^6(-11b^4+26b^2c^2-11c^4) + 17a^4(b^2-c^2)^2(b^2+c^2) - a^2(b^2-c^2)^2(5b^4+18b^2c^2+5c^4) - (b^2-c^2)^4(b^2+c^2),

    con número de búsqueda en ETC: 21.8240539268997687556671035
    ( Mostrar/Ocultar figura )
      Anopolis2774aX74.png



     •  Si se toman las cevianas de X80 (reflexión del incentro en el punto de Feuerbach), el centro de ortología de ABC respecto a M1M2M3 es X1 (incentro).
    El centro de ortología de M1M2M3 respecto a ABC es el centro del triángulo de primera coordenada baricéntrica:

    a(a^5(b+c) - a^4(b^2+c^2) + a^3(-2b^3+b^2c+b c^2-2c^3) + a^2(2b^4+b^3c-4b^2c^2+bc^3+2c^4) + a(b-c)^2(b^3+c^3) - (b^2-c^2)^2(b^2+b c+c^2)),

    con número de búsqueda en ETC: 5.709370540773524750351740641
    ( Mostrar/Ocultar figura )
      Anopolis2774aX80.png


  • lunes, 17 de agosto del 2015

    Puntos isodinámicos y cuadriláteros cíclicos

    a Clara, por su "cumple"



      Sean ABC un triángulo (a=BC, b=CA, c=AB) y P un punto en su plano. La circunferencia P(a), de centro P y radio a, corta a la recta AP en los puntos Pa y P'a; la circunferencia P(b), de centro P y radio b, corta a la recta BP en los puntos Pb y P'b; la circunferencia P(c), de centro P y radio c, corta a la recta CP en los puntos Pc y P'c.

      Si denotamos por da la distancia de A a P, PPa : PaA = -a : a + da y PP'a : P'aA = -a : a - da. Similarmente para Pb, P'b, Pc, P'c.

      Los puntos isodinámicos (X15 y X16) son los únicos puntos para los cuales los cuadriláteros BCPbPc, CAPcPa, ABPaPb, BCP'bP'c, CAP'cP'a, ABP'aP'b son cíclicos.



      El lugar geométrico de los puntos P tales que los cuadriláteros BCPbPc y BCP'bP'c son cíclicos es la circunferencia circunscrita a AX15X16.
    ( Mostrar/Ocultar figura )
      HG170815a.png
    Descargar fichero GeoGebra



      Sean ABC un triángulo y P un punto, la circunferencia de centro en A y radio a corta a la recta AP en los puntos A1 y A2 (A2 en la semirrecta AP). Similarmente, se definen los puntos B1, B2, C1, C2.

      Los cuadriláteros BCB1C1, CAC1A1, ABA1B1 son cíclicos si P es el punto que tiene coordenadas trilineales exactas respecto al triángulo de lados a=6, b=9 y c=13: (4.7237955610083, 4.0664258171145, -1.35476672732948)

    ( Mostrar/Ocultar figura )
      HG170815b1.png

      Los cuadriláteros BCB2C2, CAC2A2, ABA2B2 son cíclicos si P es el punto que tiene coordenadas trilineales exactas respecto al triángulo de lados a=6, b=9 y c=13: (31.918183326295, 30.096561337426, -31.926885671523)

    ( Mostrar/Ocultar figura )
      HG170815b2.png


  • domingo, 16 de agosto del 2015

    Los centros del triángulo X84 y X946 como centros ortológicos


    (Anopolis #2774, Antreas Hatzipolakis)

      Sea ABC un triángulo, Ab y Ac las proyecciones ortogonales de A sobre las bisectrices por B y C, respectivamente, A' es el punto medio de AbAc.
      Similarmente se definen los puntos B' y C'.

      Los triángulos ABC y A'B'C' son ortológicos. El centro de ortología de A'B'C' respecto a ABC es X84. El centro de ortología de ABC respecto a A'B'C' es X946.

    ( Mostrar/Ocultar figura )
      Anopolis2774X1.png



      Si en vez de tomar las bisectrices se toman las cevianas de X80 (reflexión del incentro en el punto de Feuerbach), el centro de ortología de ABC respecto a A'B'C' es X84.
    ( Mostrar/Ocultar figura )
      Anopolis2774X80.png


  • domingo, 2 de agosto del 2015

    PK(P) de puntos sobre la circunferencia circunscrita

    Especial aniversario



    Preamble to ETC #2081 (Bernard Gibert)
    PK Transform

     On October 1, 2003, Bernard Gibert described the transform:
      Let X be a point, then PK(X) is the point of intersection of the trilinear polar of X and the trilinear polar of X* (X* denote the isogonal conjugate of X).
      PK(X) is the perspector of the circumconic through X and X*.

      Examples:
    PK(X(99)) = X(2086)
    PK(X(100)) = X(2087)
    PK(X(110)) = X(2088)


      Sean P y Q dos puntos y δ una recta variable que pasa por Q y corta a BC, CA, AB en los puntos Da, Db, Dc, respectivamente.
      La paralela por Db a PC y la paralela por Dc a PB se cortan en el punto A'.
      Si P=(p:q:r) y Q=(u:v:w), en coordenadas baricéntricas:

      El lugar geométrico de los puntos D de intersección de las rectas δ y PA', cuando δ gira alrededor de Q, es la cúbica circunscrita cu(P,Q) a ABC que pasa por P y Q (doble), de ecuación:

    cu(P,Q):
    ( Mostrar/Ocultar figura )
      ADGEOM2646.png
    Descargar fichero GeoGebra
    NOTA: Si se definen los puntos B' y C' de manera cíclica, se tiene que P, A', B', C' están alineados (ADGEOM #2646, Dao Thanh Oai).

      ALGUNAS CÚBICAS PARTICULARES:

    cu(X6,X2): K280=spK(X6, X597).
    cu(X2,X6): K281= spK(X2, X597).
    cu(X4,X2): K295.
    cu(X4,X3): K009= spK(X4, X5), Lemoine cubic.
    cu(X3,X4): K028= spK(X3, X5), Musselman (third) cubic.

    ∗  ∗  ∗  ∗  ∗  ∗  ∗  ∗  ∗  ∗ 



      El lugar geométrico de los puntos D de intersección de las rectas δ y PA', cuando δ varía, paralela a la dirección del punto del infinito Qi, es la hipérbola circunscrita co(P,Qi) a ABC que pasa por P y sus asíntotas tienen las direcciones de Qi y de la recta AA', de ecuación:

    co(P,Qi):
    ( Mostrar/Ocultar figura )
      PKpropiedadc(PQi).png
    Descargar fichero GeoGebra
    NOTA: Si el punto Qi se mueve en la recta del infinito, el perspector de co(P,Qi) está sobre la tripolar de P.

    ∗  ∗  ∗  ∗  ∗  ∗  ∗  ∗  ∗  ∗ 



      Si P está sobre la circunferencia circunscrita a ABC y Qi=P* es su conjugado isogonal (en la recta del infinito), el perspector de la hipérbola co(P,Qi) es PK(P).

    ( Mostrar/Ocultar figura )
      PKpropiedad.png
    Descargar fichero GeoGebra

      La imagen de la circunferencia circunscrita mediante la transformación PK es la cúbica unicursal, con punto singular aislado el simediano, que pasa por los vértices de los triángulos cevianos del incentro y simediano, por los puntos de intersección de los lados de ABC con la tripolar del incentro, y a través de los centros del triángulo X2086, X2087 y X2088.
      Su ecuación baricéntrica es:
    b^4c^4x^3 + a^4c^4y^3 + a^4b^4z^3 - a^2b^2c^2x(c^2y^2+b^2z^2) - a^2b^2c^2y(c^2x^2+a^2z^2) - a^2b^2c^2z(b^2x^2+a^2y^2) + a^2b^2c^2(a^2+b^2+c^2)xyz=0.



  • miércoles, 24 de junio del 2015

    Circuncentros de triángulos sobre la recta de Euler. Una quíntica


    (Anopolis #2708, Antreas Hatzipolakis)

      Sea ABC un triángulo, P un punto y A'B'C' el triángulo antipedal de P.
      Denotamos por N1, N2, N3 los centros de las circunferencias de los nueve puntos de los triángulos A'BC, B'CA, C'AB, respectivamente.
      Y por M1, M2, M3 los puntos medios de los segmentos AN1, BN2, CN3, respectivamente.

      El lugar geométrico de los puntos P tal que el circuncentro del triángulo M1M2M3 está en la recta de Euler de ABC es la quíntica de ecuación baricéntrica:

    ( Mostrar/Ocultar figura )
      Anopolis2707.png

      Esta quíntica pasa por los vértices de ABC (puntos dobles), por los excentros, por los vértices del triángulo ceviano de X265 y por los centros X1, X4, X5, X30, X1113, X1114, X1138.
      Si P es el incentro o el ortocentro, el circuncentro de M1M2M3 es X140.

      Las tangentes en los excentros concurren en X339, punto sobre la asíntota que es paralela a la recta de Euler

  • miércoles, 17 de junio del 2015

    Los centros X6374 y X6376 sobre una cuártica



    ETC #3221 (C. Kimberling)
    Symbolic substitution (a,b,c) ↦ (bc,ca,ab)

     Symbolic substitutions are introduced in the following article:
    C. Kimberling, "Symbolic substitutions in the transfigured plane of a triangle," Aequationes Mathematicae 73 (2007) 156-171.

      Examples:
    X(6374) = SS(a ↦ bc) OF X(3)
    X(6376) = SS(a ↦ bc) OF X(9)
    X(6377) = SS(a ↦ bc) OF X(11)


      Sean ABC un triángulo y t un número real.
      Denotamos por Ab la imagen de A en la homotecia h(B,tc) (de centro B y razón tc) y por Ac la imagen de A en la homotecia h(C,tb).
      Similarmente, se definen Bc, Ba, Ca, Cb.

      Los puntos Ab, Ac, Bc, Ba, Ca, Cb están en una misma cónica Γt.
      El lugar geométrico del centro de Γt, cuando t varía, es una cuártica circunscrita a ABC, con puntos dobles en los puntos medios de sus lados.


    ( Mostrar/Ocultar figura )
      HG170615.png
    Descargar fichero GeoGebra

      La ecuación baricéntrica de la cónica Γt es:   La primera coordenada del centro W de Γt es:

    a^2t(b+c-b^2t-c^2t) + a(b(bt-1)+c(ct-1))) + bc(bt-1)(ct-1)(bc(bt-1)(ct-1).

      El lugar geométrico de W, cuando t varía, es la cuártica:   Esta cuártica pasa por los vértices de ABC, por los puntos medios de los lados (dobles) y por los centros del triángulo X6374 y X6376. Estos puntos se obtienen cuando t ↦ ∞ y cuando t ↦0, respectivamente.
      Cuando t ↦0, Γ0 es la cónica circunscrita de perspector X75: bcyz + cazx + abxy = 0, que pasa por A, B, C, X(668), X(789), X(799), X(811), X(1978), X(4554), X(4583), X(4593), X(4602).

      Las rectas AbAc, BcBa, CaCb delimitan un triángulo A'B'C' perspectivo con ABC. El lugar geométrico los centros de perspectividad es la cónica circunscrita de centro X6377.

    ( Mostrar/Ocultar figura )
      HG170615a.png
    Descargar fichero GeoGebra
      Las rectas AA', BB', CC' concurren en el punto:

    Z = (1/(a-(ab-bc+ca)t) : 1/(b-(bc-ca+ab)t) : 1/(c-(ca-ab+bc)t)).

      El lugar geométrico que describe Z es la cónica:

    (b-c)(bc-a(b+c))yz + (c-a)(ca-b(c+a))zx + (a-b)(ab-c(a+b))xy = 0.

      Esta cónica tiene centro en X6377 y pasa por los puntos X(75), X(192), X(257), X(330), X(2998), X(3212), X(3226), X(6376), X(7233).

      El cuarto punto de intersección de esta cónica con la circunferencia circunscrita es:

    (1/(a^2(b-c)^2(b+c) - a(b^4+c^4) + bc(b^3+c^3)) : .... : ... ),

    con número de búsqueda en ETC: -0.24303748881040055714859319

  • jueves, 11 de junio del 2015

    Hipérbola equilátera, lugar geométrico

    ( TriangulosCabri #737, Ricardo Barroso)

      Dados tres puntos B, C y P, hallar el lugar geométrico del punto A tal que la circunferencia de los nueve puntos del triángulo ABC pasa por P.



      Los pies de las perpendiculares desde B y C a los lados AB y AC están en la circunferencia de Euler de ABC y quedan sobre la circunferencia Γa de diámetro BC.

      Para determinar el lugar geométrico de A, tomamos una recta por m que pasa por B y una homóloga n por C, imagen de una proyectividad que vamos a describir.
    ( Mostrar/Ocultar figura )
      rb737.png
    Descargar fichero GeoGebra

      Tomamos un punto F sobre la circunferencia Γa y sea m la recta BF.

      Como la circunferencia de Euler pasa por Ma (punto medio de BC) y debe pasar por P, tomamos la circunferencia ΓF que pasa por F, Ma y P. Esta circunferencia vuelve a cortar a la circunferencia Γa en un punto E.

      La correspondencia FE es una proyectividad sobre la circunferencia Γa, pues proyecta F sobre E desde el punto fijo (en la recta PMa) por donde pasan los ejes radicales de las dos circunferencias Γa y ΓF.

      Llamamos n a la recta CE.

      La correspondencia mn es una proyectividad entre los haces de rectas con bases en B y C. Luego, el punto A, intersección de m y n, describe una cónica que pasa por B y C.

      La cónica es una hipérbola, cuyos puntos del infinito con los de las rectas que unen B (o C) con los puntos de intersección de la recta PMa (circunferencia de diámetro infinito) con la circunferencia Γa. Luego las asíntotas son perpendiculares (hip. equilátera).

    El centro de la hipérbola equilátera es el punto P:

      Si m1=BP y F1=m1 ∩Γa, sea E1 el punto donde la circunferencia (PF1Ma) vuelve a cortar a Γa, n1=CE1 y A1=m1∩n1.

      Las rectas BE1 y CE1 son perpendiculares. Luego, A1B es un diámetro de la circunferencia (A1BE1), es decir, P es el punto medio de A1B.
     Lo mismo ocurriría si tomamos como n2=CP: A2=m2∩n2; P es el punto medio de A2C.
     Así, A1A2 es una cuerda paralela a BC. La polar de P es la recta del infinito. Por lo que P es el centro de la hipérbola.

  • miércoles, 10 de junio del 2015

    Construcción de triángulo


    (Anopolis #2628, Antreas Hatzipolakis)

      Sea ABC un triángulo y AA', AA" the a-mediana, a-simediana resp.
      Construir ABC si se da el triángulo AA'A".



    Anopolis #2629 (Luís Lopes)

      Let AA'=m_a, AH_a=h_a and AD_a=d_a where d_a is a-internal bisector.
    As one can construct both h_a and d_a, the problem is equivalent to construct ABC given h_a, m_a, d_a.
    And this is known.


    Anopolis #2638 (Telv Cohl)

      Let AX, AY be the bisectors of ∠A'AA'' (X and Y are on A'A''). Let the circle with diameter XY cuts AA'' at {A,T} and S be the point on AA' such that ST// A'A''. Then B, C are the intersection of A'A'' with (AST).
    ( Mostrar/Ocultar figura )
      Anopolis2638.png


      Utilizaremos el método de lugares geométricos y la intersección de una cónica con una recta.
    ( Mostrar/Ocultar figura )
      Anopolis2628.png
    Descargar fichero GeoGebra

      Sobre la recta A'A'' (donde van a estar los vértices B y C del triángulo a construir) tomamos un punto B1 y su simétrico B'1, respecto a A'. Al variar B1, el lugar geométrico del simediano K1 del triángulo AB1B'1 es una recta que pasa por A' y corta a la recta AA'' en el simediano K del triángulo buscado ABC.

      Tomando, en particular, el simediano K' del triángulo AA'''A'' (A''' la reflexión de A'' en A'), la recta lugar geométrico de los simedianos K' queda determinada por los puntos A y K'.

      Sea D1E1F1 el triángulo pedal de K1, respecto al triángulo AB1B'1. El lado E1F1 permanece paralelo a sí mismo. El lugar geométrico de los puntos E1 y F1, cuando B1 varía, es una elipse que pasa por A y tangente en A' a la recta paralela a E1F1. Esta elipse pasa, en particular, por los vértices E' y F' del triángulo pedal D'E'F' de K' respecto al triángulo AA'''A''. Con lo que tal elipse puede ser construida: corresponde al caso PPPtP de construcciones de cónicas.

      Vamos a utilizar la propiedad del simediano siguiente:

    "El simediano es el baricentro de su triángulo pedal".


      Sea D el pie de la perpendicular trazada desde K a la recta A'A'' y M el punto en que divide al segmento KD en -1:3.

      La recta por M paralela a E'F' corta a la elipse en los puntos E y F, siendo DEF el triángulo pedal de K, respecto al ABC. Así, las rectas AE y AF cortan a la recta A'A'' en los vértices C y B del triángulo buscado.

      Cuando la recta EF y la elipse no se cortan, NO hay solución. Esto ocurre cuando Va no está sobre el segmento HaMa (ver http://amontes.webs.ull.es/pdf/ejct2031.pdf).

  • martes, 9 de junio del 2015

    Cúbica K018 y triángulos perspectivos



      Sean ABC un triángulo, y un punto P (no situado en la recta del infinito). Los ejes radicales de la circunferencia de diámetro BC y de las circunferencias que pasan por P y el punto medio de BC, pasan por un punto fijo A'.
      Procediendo cíclicamente, sobre los vértices de ABC, se definen los puntos B' y C'.

      Los triángulos ABC y A'B'C' son perspectivos si y solo si el punto P está sobre la circunferencia de Euler o sobre la cúbica K018.


    ( Mostrar/Ocultar figura )
      HG090615.png
    Descargar fichero GeoGebra

      Si (u:v:w) son las coordenadas baricéntricas de P:
    A'=(-a^2 u (u + v + w) : a^2 (u + v) (u - v + w) - u (b^2 (u + v - w) + c^2 (u - v + w)) : a^2 (u + v - w) (u + w) - u (b^2 (u + v - w) + c^2 (u - v + w))).

      La condición algebraica para que los triángulos ABC y A'B'C' sean perspectivos es:
    (-a^2 u^2 + b^2 u^2 + c^2 u^2 - 2 c^2 u v + a^2 v^2 - b^2 v^2 + c^2 v^2 - 2 b^2 u w - 2 a^2 v w + a^2 w^2 + b^2 w^2 - c^2 w^2) (-a^2 c^2 u^2 v + c^4 u^2 v + b^2 c^2 u v^2 - c^4 u v^2 + a^2 b^2 u^2 w - b^4 u^2 w + a^4 v^2 w - a^2 b^2 v^2 w + b^4 u w^2 - b^2 c^2 u w^2 - a^4 v w^2 + a^2 c^2 v w^2)=0.

      El lugar geométrico de los centros de perspectividad de los triángulos ABC y A'B'C', cuando P recorre la circunferencia de Euler, es el eje órtico.

  • martes, 2 de junio del 2015

    Triángulos inversamente semejantes


    (Anopolis #2585, Antreas Hatzipolakis-Leonard Giugiuc)

      Sean ABC un triángulo, O, H el circuncentro y ortocentro, respectivamente. Se denota por Oa la reflexión de O en AH, por Ha la reflexión de H en Oa y por Ma el punto medio de OaHa. Similarmente se definen Mb y Mc.

      Los triángulos ABC y MaMbMc son inversamente semejantes.

    ( Mostrar/Ocultar figura )
      Anopolis2585.png
    Descargar fichero GeoGebra

      Las coordenadas baricéntricas de Ma son:

    (a^2(-4a^4+(b^2-c^2)^2+3a^2(b^2+c^2)) : a^6+a^4c^2-3(b^2-c^2)^3+a^2(2b^4+3b^2c^2-5c^4) : a^6+a^4b^2+3(b^2-c^2)^3+a^2(-5b^4+3b^2c^2+2c^4)).

      La matriz asociada a la aplicación afín que transforma ABC en MaMbMc es:
    a^2 b^2 c^2 (4 a^4-3 a^2 b^2-b^4-3 a^2 c^2+2 b^2 c^2-c^4) a^2 c^2 (3 a^6 - 2 a^4 b^2 - b^6 - 9 a^4 c^2 - 3 a^2 b^2 c^2 - b^4 c^2 + 9 a^2 c^4 + 5 b^2 c^4 - 3 c^6) a^2 b^2 (3 a^6 - 9 a^4 b^2 + 9 a^2 b^4 - 3 b^6 - 2 a^4 c^2 - 3 a^2 b^2 c^2 + 5 b^4 c^2 - b^2 c^4 - c^6)
    b^2 c^2 (-a^6 - 2 a^2 b^4 + 3 b^6 - a^4 c^2 - 3 a^2 b^2 c^2 - 9 b^4 c^2 + 5 a^2 c^4 + 9 b^2 c^4 - 3 c^6) a^2 b^2 c^2 (-a^4 - 3 a^2 b^2 + 4 b^4 + 2 a^2 c^2 - 3 b^2 c^2 - c^4) a^2 b^2 (-3 a^6 + 9 a^4 b^2 - 9 a^2 b^4 + 3 b^6 + 5 a^4 c^2 - 3 a^2 b^2 c^2 - 2 b^4 c^2 - a^2 c^4 - c^6)
    b^2 c^2 (-a^6 - a^4 b^2 + 5 a^2 b^4 - 3 b^6 - 3 a^2 b^2 c^2 + 9 b^4 c^2 - 2 a^2 c^4 - 9 b^2 c^4 + 3 c^6) a^2 c^2 (-3 a^6 + 5 a^4 b^2 - a^2 b^4 - b^6 + 9 a^4 c^2 - 3 a^2 b^2 c^2 - 9 a^2 c^4 - 2 b^2 c^4 + 3 c^6) a^2 b^2 c^2 (-a^4 + 2 a^2 b^2 - b^4 - 3 a^2 c^2 - 3 b^2 c^2 + 4 c^4)

      Se trata de una semejanza, pues transforma rectas perpendiculares en rectas perpendiculares. En efecto, las imágenes de los puntos del infinito de la recta px+qy+rz=0 y de su dirección perpendicular son, respectivamente:

    (f:g:h)=
    (a^2 (b^4 (-p + q) + c^4 (p - r) + a^2 (b^2 (p - q) + c^2 (-p + r))) : ... : ...),

    (f':g':h')=
    (a^2 (a^4 (b^2 (p - q) + c^2 (p - r)) + (b^2 - c^2) (b^4 (p - q) + c^4 (-p + r) + 2 b^2 c^2 (-q + r)) - a^2 (2 b^4 (p - q) + 2 c^4 (p - r) + b^2 c^2 (-2 p + q + r))) : ... : ...).

      Verificándose: SAff' + SBgg' + SChh' = 0.

      El centro de semejanza, que es el punto fijo (propio) de esta semejanza correspondiente a la raíz del polinomio característico,
    λ1 = 2 (a^6 b^2 c^2 - 2 a^4 b^4 c^2 + a^2 b^6 c^2 - 2 a^4 b^2 c^4 - 2 a^2 b^4 c^4 + a^2 b^2 c^6),
    es:

    S=(a^2 (9 a^8 - 24 a^6 (b^2 + c^2) + a^4 (18 b^4 + 37 b^2 c^2 + 18 c^4) - 15 a^2 b^2 c^2 (b^2 + c^2) - (b^2 - c^2)^2 (3 b^4 + 4 b^2 c^2 + 3 c^4)): ... : ...),

    con número de búsqueda en ETC: 3.997265584497438004548017190

      Este punto ha sido incorporado a ETC: X(7666) = Giugiuc Center Of Similitude.

  • martes, 19 de mayo del 2015

    Ortotransversales respecto a triángulos cevianos y pedales


    Forum Geometricorum Volume 3 (2003) 1–27 (Bernard Gibert)

      The perpendiculars drawn through P to the lines AP, BP, CP meet BC, CA, AB respectively at three collinear points called the orthotraces of P. These three points lie on a line called orthotransversal of P.

      Sean ABC un triángulo y un punto P (no en la recta del infinito), las ortotransversales de P respecto a ABC y al triángulo ceviano de P son paralelas si solo si P está en la quíntica de Euler-Morley (Q003).

      Este resultado es consecuencia de que, para todo P, estas dos rectas ortotransversales se cortan en la tripolar de P y de la propiedad 1: “Q003 is the locus of P such that the trilinear polar of P and the orthotransversal of P are parallel”.
    ( Mostrar/Ocultar figura )
      orthotransversal-cevianas.png
    Descargar fichero GeoGebra
    NOTA:
      Cuando P coincide con el incentro o los exincentros, las ortotransversales consideradas son simétricas respecto a P.
      Una demostración sintética de este hecho, para el caso en que P es el incentro, puede verse en Art Of Problem Solve (Luis González).

      Sean ABC un triángulo y un punto P (no en la recta del infinito), las ortotransversales de P respecto a ABC y al triángulo pedal de P son paralelas si solo si P está en la quíntica de ecuación baricéntrica:



      Las rectas ortotransversales de P(u:v:w) respecto a ABC y al triángulo pedal de P son, respectivamente:
        x/( u (-u SA + v SB + w SC) + a^2 v w) + …=0,
        v w (-a^4 u+(b-c)^2 (b+c)^2 u+ a^2 SA (v+w)) x + … =0.

      Estas rectas y la del infinito, x+y+z=0, son concurrentes si P(u:v:w) satisface a la ecuación de la quíntica del enunciado.

    ( Mostrar/Ocultar figura )
      orthotransversal-pedal.png
    Descargar fichero GeoGebra


      Propiedades geométricas de esta quíntica:

      • Pasa por los vértices de ABC, que son puntos dobles.
      • Pasa por los centros del triángulo X3, X4, X1113, X1114.
      • Pasa por las intersecciones de cada ceviana del circuncentro y la circunferencia con diámetro el correspondiente lado de ABC.
      • Tangente a la recta de Euler en el ortocentro.
      • Tangente a las alturas en sus pies.

      Otras propiedades (indicadas por Bernard Gibert):
      • Is a circular quintic (with focus X5) and contains the infinite points of the cubic K006, pK(X6,X4).
      • Is a member of the pencils of quintics generated by:
        – the union of K006 with the circumcircle (O) and the union of the sidelines, the line at infinity, the Euler line,
        – Q003 and the union of the altitudes with (O),
        – Q038 and the union of K059 with (O),
        – Q044 and the union of K329 with (O).

      Sean ABC un triángulo y un punto P (no en la recta del infinito), las ortotransversales de P respecto a ABC y al triángulo circunceviano de P son paralelas si solo si P está en la cubica pK(X6,X4) (K006).


    ( Mostrar/Ocultar figura )
      orthotransversal-circunceviano.png
    Descargar fichero GeoGebra


  • martes, 21 de abril del 2015

    Una séxtica isogonal


    (Anopolis #2502, Antreas Hatzipolakis)

      Sean ABC un triángulo, P un punto y PaPbPc el triángulo antipedal de P.
      Se denotan por AaAbAc, BaBbBc, CaCbCc los triángulos pedales de Pa, Pb, Pc, respectivamente.

      El triángulo A'B'C' delimitado por las rectas AbAc, BcBa, CaCb es no degenerado y perspectivo con ABC si y solo si P en la séxtica isogonal circunscrita a ABC de ecuación baricéntrica:

    a^6y^2z^2(y-z)(x+2(y+z))+
    a^4y z(c^2y(x^3+3x^2y-2z(y+z)^2+ 2x(2y^2-y z-3z^2))- b^2z(x^3+3x^2z-2y(y+z)^2+2x(2z^2-y z-3y^2)))+
    a^2x(-2b^2c^2y z(x-y)(x-z)(y-z)+ c^4y^2(2x^3+2x y(y+z)+x^2(4y+6z)- z(4y^2+3y z+z^2))+ b^4z^2(-2x^3-2x z(y+z)-2x^2(3y+2z)+ y(y^2+3y z+4z^2)))+
    x^2(c^6(x-y)y^2(2x+2y+z)- b^6(x-z)z^2(2x+y+2z)+ b^2c^4y(-2y^3-2x^2(y-2z)-6y^2z+z^3+ x(-4y^2-2y z+3z^2))+ b^4c^2z(-y^3+6y z^2+2z^3+x^2(-4y+2z)+ x(-3y^2+2y z+4z^2)))=0.

    ( Mostrar/Ocultar figura )
      Anopolis2502.png
    Descargar fichero GeoGebra

      Si P=(u:v:w), los vértices del triángulo pedal de Pa son:

    Aa = (0 : (b^2 - c^2) u - a^2 (u + 2 w) : (-b^2 + c^2) u - a^2 (u + 2 v)),
    Ab = (v ((-b^2 + c^2) u + a^2 (u + 2 w)) : 0 : -b^2 u (2 u + v + 2 w) - v (c^2 u + a^2 (u + 2 w))),
    Ac = (((b^2 - c^2) u + a^2 (u + 2 v)) w : -(b^2 u + a^2 (u + 2 v)) w - c^2 u (2 u + 2 v + w) : 0).

      La ecuación de la recta AbAc es:

    x+((((b^2-c^2) u+a^2 (u+2 v)) w)/((b^2 u+a^2 (u+2 v)) w+c^2 u (2 u+2 v+w)))y + ((v ((-b^2+c^2) u+a^2 (u+2 w)))/(b^2 u (2 u+v+2 w)+v (c^2 u+a^2 (u+2 w))))z = 0.

      Permutando cíclicamente se obtienen las ecuaciones de las rectas BcBa y CaCb. Estas tres rectas son concurrentes si el punto P está sobre una séxtica circunscrita a ABC.

    ∗  ∗  ∗  ∗  ∗  ∗  ∗  ∗  ∗  ∗ 


      El triángulo A'B'C' delimitado por las rectas AbAc, BcBa y CaCb es no degenerado y perspectivo con AaAbAc si P está en la séxtica del enunciado.
      Esta séxtica es isogonal y contiene a los centros X1, X20 y X64.
      Tiene los vértices de ABC como puntos dobles. Las tangentes en ellos son las tangentes a la circunferencia circunscrita y las rectas que pasan por X25 (punto no situado sobre la curva).
      Otros puntos de la séxtica son los antipodales de A, B y C. Las tangentes en ellos se cortan en X25.
    ( Mostrar/Ocultar figura )
      Anopolis2502tg.png
    Descargar fichero GeoGebra
      Pasa por los vértices del triángulo ceviano de X69 y los exincentros.

      Tres de sus asíntotas son paralelas a las alturas y se cortan en X64; las otras tres son paralelas a los lados de ABC y forman un triángulo perspectivo con ABC, con centro de perspectividad en X184.

  • miércoles, 15 de abril del 2015

    Centros ortológicos y homografía


    (Anopolis #2483, Antreas Hatzipolakis)

      Sean ABC un triángulo, P un punto, A'B'C' el triángulo pedal de P y A1B1C1, A2B2C2 los triángulos mediales de ABC, A'B'C', respectivamente.
      Sea A*, B*, C* las reflexiones de A1, B1, C1 en A2, B2, C2, respectivamente.

      Los triángulos ABC y A*B*C* son ortológicos para todo P.
      Las perpendiculares desde A*, B*, C* a los lados BC, CA, AB concurren en un punto Q.
      La correspondencia P ↦ Q es una homografía.

    ( Mostrar/Ocultar figura )
      Anopolis2483.png
    Descargar fichero GeoGebra

      Si P=(u:v:w), coordenadas baricéntricas, se tiene:

    A* = (c^2(a^2-c^2)v-b^4w+b^2(a^2w+c^2(4u+v+w)) : b^2(c^2(-u+v)+(-a^2+b^2)w) : c^2((-a^2+c^2)v+b^2(-u+w)) )

    Las coordenadas de B* y C* se obtienen a partir de las de A*, por permutación cíclica.

      El centro de ortología de A*B*C* respecto a ABC es:

    Q = (a^2 (a^6 (c^2 v + b^2 w) - 3 a^4 (b^2 + c^2) (c^2 v + b^2 w) + a^2 (3 c^6 v + b^4 c^2 (-2 u + v) + 3 b^6 w + b^2 c^4 (-2 u + w)) - (b^2 - c^2)^2 (c^4 v + b^4 w - b^2 c^2 (2 u + v + w))) : ... : ... ).

    O en coordenadas trilineales (César Lozada):

    Q = (sin(A)cos(B-C)u+sin(B)(-cos(A)+sin(A)sin(2C))v+sin(C)(-cos(A)+sin(A)sin(2B))w : ... : ... ).


      Pares de puntos {P=X(i),Q=X(j)} que son actualmente centros de la Enciclopedia de Clark Kimberling, {i,j}:
    {3, 4}, {4, 52}, {5, 5446}, {6, 1351}, {64, 382}, {110, 113}, {113, 1112}, {154, 381}, {155, 155}, {156, 5448}, {159, 1352}, {206, 5480}, {221, 1482}, {381, 3060}, {382, 5889}, {399, 110}, {511, 3564}, {512, 525}, {513, 521}, {516, 916}, {517, 912}, {523, 520}, {924, 523}, {1154, 539}, {1158, 1872}, {1351, 193}, {1352, 1843}, {1482, 3868}, {1498, 3}, {1503, 511}, {1510, 6368}, {1625, 2967}, {2192, 2095}, {2390, 519}, {2393, 524}, {2574, 2574}, {2575, 2575}, {2777, 5663}, {2778, 2771}, {2781, 542}, {2818, 952}, {2883, 389}, {2917, 6288}, {3197, 5779}, {3357, 3627}, {3556, 355}, {3566, 512}, {3827, 518}, {3852, 732}, {5878, 185}, {5887, 1829}, {5891, 428}, {5907,6756}, {6000,30}, {6001,517}, {6288,6152}, {6759,5}, {6760,1559}, {7387,68}.

      La correspondencia P ↦ Q es una homografía de matriz asociada:
    a^2SA(b^2SB+c^2SC+SA SB SC)a^2SB(SA^2 SB - c^2 SC^2)a^2SC (SA^2 SC - b^2SB^2)
    b^2SA(SASB^2-c^2 SC^2) b^2SB(a^2SA^2+c^2SC^2+SASBSC) b^2SC(SB^2 SC-a^2SA^2)
    c^2SA(SA SC^2-b^2SB^2)c^2SB(SB SC^2-a^2SA^2) c^2SC (a^2SA^2+b^2SB^2+SASBSC)

      Los puntos fijos de esta homografía, correspondientes a los valores propios λ1= 2S^2SASBSC, λ2,3 = S^2(SASBSC±2abcS*OH)/2, son X155, X2574 y X2575, estos dos últimos (en la recta del infinito) son los conjugados isogonales de los puntos en los que la recta de Euler corta a la circunferencia circunscrita.

  • viernes, 27 de marzo del 2015

    Foco de la parábola de Kiepert como punto fijo de una semejanza inversa



      Sean ABC un triángulo y DEF su triángulo órtico. Denotemos por Ba y Ca las reflexiones de B y C en D. Los puntos Cb, Ab, Ac, Bc se definen análogamente. A'B'C' es el triángulo delimitado por las rectas BcCb, CaAc , AbBa.

      Los triángulos ABC y A'B'C' se corresponde en una semejanza inversa con punto fijo en el foco de la parábola de Kiepert, X110.

    ( Mostrar/Ocultar figura )
      X(110).png
    Descargar fichero GeoGebra

      En coordenadas baricéntricas, las ecuaciones de la semejanza inversa son:

    λx' = a^2 (a^2 b^2 c^2 (a^2 -b^2 ) (a^2 -c^2 )x+2SB c^2 (a^2 -c^2 ) (a^4-b^4+c^4+a^2 (b^2-2 c^2)) y+2SC b^2 (a^2 -b^2 )(a^4+b^4-c^4+a^2 (c^2-2 b^2)) z))
    λy' = b^2 (2 SA c^2 (b^2-c^2) (-a^4+b^4+c^4+b^2 (a^2-2 c^2)) x+a^2 b^2 c^2 (-a^2+b^2)(b^2-c^2) y+2 SCa^2 (-a^2+b^2) (a^4+b^4-c^4+b^2 (-2 a^2+c^2)) z)
    λz' = c^2 (2 SAb^2 (-b^2+c^2) (-a^4+b^4+(a^2-2 b^2) c^2+c^4) x+2 SB a^2 (-a^2+c^2) (a^4-b^4+(-2 a^2+b^2) c^2+c^4)y+a^2 b^2 c^2 (-a^2+c^2) (-b^2+c^2) z).

      El otro punto de intersección de las circunferencias circunscritas a ABC y A'B'C' es X107. Ocurre además que los focos de las parábolas de Kiepert de estos triángulos coinciden.

  • jueves, 26 de marzo del 2015

    La cúbica K621 como lugar de perspectores



    (CTC K621, Bernard Gibert)

      Sea ABC un triángulo, DEF el triángulo ceviano de un punto P y HaHbHc el triángulo órtico (pedal del ortocentro H=X4). Denotamos por A' el punto de intersección del lado HbHc del triángulo órtico con la recta que une D con el punto medio del segmento AH. Los puntos B' y C' se definen similarmente.

    Los triángulos ABC y A'B'C' son perspectivos si y solo si P está en la cúbica pK(X393,X4×X317).


      La isocúbica pK(X393,X4×X317) pasa por los centros X2, X4, X254, X393, X2165 y X3542 y por los vértices del triángulo anticeviano del ortocentro.

      Si (u:v.w) son las coordenadas baricéntricas de P, el punto A' es:

    A' = (2 (a^2 (b^2 + c^2)-(b^2 - c^2)^2) (-(b^2 - c^2) (v - w) + a^2 (v + w)) :
          (a^2 - b^2 - c^2) (a^4 (v - w) + (b^2 - c^2)^2 (v - w) - 2 a^2 (c^2 (v - w) + b^2 (v + w))) :
          -(a^2 - b^2 - c^2) (a^4 (v - w) + (b^2 - c^2)^2 (v - w) + 2 a^2 (b^2 (-v + w) + c^2 (v + w))) ).
    ( Mostrar/Ocultar figura )
      K621.png

      Los centros de perspectividad de ABC y A'B'C' están en la cúbica K621=pK(X393, X2052).

    A los centros X2, X4, X254 , X393, X2165, X3542 les corresponden, respectivamente, X6, X4, X847, X2052, X2, X24.

  • lunes, 23 de marzo del 2015

    Propiedad de la cúbica K045

    a Marta, por su "cumple" (serían 39)



    (CTC K045, Bernard Gibert)

      Sea ABC un triángulo, DEF el triángulo ceviano de un punto P y HaHbHc el triángulo órtico (pedal del ortocentro). Denotamos por A' el pie de la perpendicular trazada desde D al lado HbHc del triángulo órtico. Los puntos B' y C' se definen similarmente.

    Los triángulos ABC y A'B'C' son perspectivos si y solo si P está en la cúbica K045=pK(X2,X264).



      Si (u:v.w) son las coordenadas baricéntricas de P, el punto A' es:

    ( (b^2-c^2)^2-a^2(b^2+c^2)) ((b^2-c^2)(v-w) - a^2(v+w)) :
          b^2(2a^2(b^2+c^2)v - (b^2-c^2)^2(v-w) - a^4(v+w)) :
          c^2(2a^2(b^2+c^2)w + (b^2-c^2)^2(v-w) - a^4(v+w) ).
    ( Mostrar/Ocultar figura )
      K045.png

      Los centros de perspectividad de ABC y A'B'C' están en la cúbica pK(X32,X24).

    A los centros X2, X3, X4, X69 , X264, X1993 les corresponden, respectivamente, X6, X3, X24, X2351, X25, X1609.

  • sábado, 21 de marzo del 2015

    Arbeolos y cúbicas asociadas


    (ADGEOM #2429, Emmanuel Antonio José García)

      Dado un triángulo ABC, sea DEF el triángulo ceviano de un punto P. Consideremos las semicircunferencias construidas en el sentido contrario de las agujas del reloj, sobre los segmentos BC, BD y DC. Tomemos los puntos Da, Db y Dc más altos de estas semicircunferencias. Las tres circunferencias de centros en Da, Db y Dc y que pasan por D, C y B, respectivamente, son concurrentes en un punto A', sobre la perpendicular a BC por D y tal que BC=DA'.
    ( Mostrar/Ocultar figura )
      ADGEOM2429.png
    Descargar fichero GeoGebra

    Similarmente, se determinan los puntos B' y C', procediendo cíclicamente sobre los lados de ABC.

    Los triángulos ABC y A'B'C' son perspectivos si y sólo sí P está sobre la cúbica C+ circunscrita de ecuación baricéntrica:

    ( Mostrar/Ocultar figura )
      ADGEOM2429F.png
    Descargar fichero GeoGebra

      Si P(w:v:w), la circunferencia de centro Da y que pasa por D es:
    a^2yz+b^2zx+c^2xy -(x+y+z) (((b^2+c^2) v w+S (v+w)^2+SA (v^2+w^2)) x+a^2 v w y+a^2 v w z)/(v+w)^2=0.

      La circunferencia de centro Db y que pasa por C es:
    a^2yz+b^2zx+c^2xy + (x+y+z)(((a^2-c^2)v-(SA+S)w)x+a^2vy)/(v+w)=0.

      Y la circunferencia de centro Dc y que pasa por B es:
    a^2yz+b^2zx+c^2xy + (x+y+z)(((a^2-b^2)w-(SA+S)v)x+a^2wz)/(v+w)=0.

      El punto de intersección de estas tres circunferencias es:

    A' =(4a^2S(v+w) : a^4v-2a^2((b^2+c^2)v+S(v+w))+(b^2-c^2)((b^2-c^2 )v-2S(v+w)) : a^4w-2a^2((b^2+c^2)w+S(v+w))+(b^2-c^2)((b^2-c^2)w+2S(v+w))).

      Las coordenadas de los puntos B' y C' se obtienen por permutación cíclica.

      La cúbica C+ pasa por los vértices de ABC, por los pies de las alturas y por los centros X2, X4, X193, X492, X1585.

      Los centros de perspectividad de ABC y A'B'C' para X2, X4, X492, X1585 son, respectivamente, X1131, X4, X485, X1321.


      Las tangentes a C+ en los vértices de ABC concurren en el punto, sobre la recta X2X6:
    T+ = ((SA+S)^2 : c^2 S+S^2+SA SB : b^2 S+S^2+SA SC).

    ∗  ∗  ∗  ∗  ∗  ∗  ∗  ∗  ∗  ∗ 

      Si consideremos las semicircunferencias construidas en el sentido de las agujas del reloj, sobre los segmentos BC, BD y DC, los puntos de intersección de las correspondientes tres circunferencias son A'', B'' y C'', simétricos de A', B' y C' respecto a los lados BC, CA y AB, respectivamente.

    Los triángulos ABC y A''B''C'' son perspectivos si y sólo sí P está sobre la cúbica C- circunscrita de ecuación baricéntrica:

    ( Mostrar/Ocultar figura )
      ADGEOM2429D.png
    Descargar fichero GeoGebra



      La cúbica C- pasa por los vértices de ABC, por los pies de las alturas y por los centros X2, X4, X193, X491, X1586.

      Los centros de perspectividad de ABC y A''B''C'' para X2, X4, X492, X1585 son, respectivamente, X1132, X4, X486, X1322.


      Las tangentes a C- en los vértices de ABC concurren en el punto, sobre la recta X2X6:
    T- = ((SA-S)^2 : -c^2 S+S^2+SA SB : -b^2 S+S^2+SA SC).

      El punto medio de T+T- es el centro de coordenadas:

    (a^4 - 3 a^2 (b^2 + c^2) + 2 (b^4 - b^2 c^2 + c^4) : ... : ...),

    con (6,9,13)-número de búsqueda en ETC: 234.213982093553888171950376

    ∗  ∗  ∗  ∗  ∗  ∗  ∗  ∗  ∗  ∗ 

    Imagen conjunta de las dos cúbicas:
    ( Mostrar/Ocultar figura )
      ADGEOM2429FD.png

    ADGEOM #2442 (Bernard Gibert)

      This cubics are pK(X1585,X4) and pK(X1586,X4) very similar to K070a,b.
      The loci of the perspectors are pK(X1131 x X1321, X1321) and pK(X1132 x X1322, X1322) resp.



  • lunes, 9 de marzo del 2015

    DC(X1113) y DC(X1114)


    Dual triangles, DC and CD Points (From the preamble to X(2979) in ETC)

      Let DEF be a triangle in the plane of triangle ABC. Let D' be the isogonal conjugate of the point of intersection of line EF and the line at infinity. Define E' and F' cyclically. D'E'F' is the dual triangle of DEF. The vertices of D'E'F' lie on the circumcircle, and D'E'F' is similar to DEF.
    ( Mostrar/Ocultar figura )
      trianguloDual.png
    Descargar fichero GeoGebra

      Suppose P is a point having cevian triangle DEF and dual triangle D'E'F'. Then there exists a point DC(P) whose circum-anticevian triangle (TCCT, p. 201) is D'E'F'. The mapping DC is given for P = (u : v : w) (barycentric) by
          DC(P)=( a^2/(u(v+w)) : b^2 /(v(w+u)) : c^2 /(w(u+v)) ).

      To construct DC(P) from P, let A' = AD'∩BC, and let A" be the {B,C}-harmonic conjugate of A'. Define B" and C" cyclically. The lines AA", BB", CC" concur in DC(P).
    ( Mostrar/Ocultar figura )
      DC(P)Ceviano.png
    Descargar fichero GeoGebra


     • El lugar geométrico de DC(P) cuando P recorre la circunferencia circunscrita al triángulo ABC es la cónica circunscrita de centro en el simediano.

      Esta cónica pasa por los vértices de ABC y por los centros:
    X110=DC(X99), X287=DC(X1297), X648=DC(X110), X651=DC(X100), X677=DC(X927), X895=DC(X2373), X1331=DC(X1305), X1332=DC(X?), X1797=DC(X2370), X1813=DC(X?), X1814=DC(X?), X1815=DC(X?), X2986=DC(X74), X2987=DC(X98), X2988=DC(X102), X2989=DC(X103), X2990=DC(X104), X2991=DC(X105), X4558=DC(X925), X4563=DC(X3565).

    DC-1(X1332)=(a/((b - c) (a^3 + b^3 + b^2 c + b c^2 + c^3 - a^2 (b + c) - a (b + c)^2)):...:...),
    con (6,9,13)-número de búsqueda en ETC 1.0260605042776496167820946164

    DC-1(X1813)=(1/((b - c) (b^5 - a^3 b c - b^3 c^2 - b^2 c^3 + c^5 + a^4 (b + c) + a b c (b + c)^2 - a^2 (2 b^3 + b^2 c + b c^2 + 2 c^3))):...:...),
    con (6,9,13)-número de búsqueda en ETC 1.2691774318095899947087466917

    DC-1(X1814)=(a/(b^5 + 2 a^3 b c - b^4 c - b c^4 + c^5 - a^4 (b + c)):...:...),
    con (6,9,13)-número de búsqueda en ETC 14.143672842741161860260393735

    DC-1(X1815)=(1/(-2 a^3 (b^2 - c^2)^2 + a^5 (b^2 + c^2) + a (b^2 - c^2)^2 (b^2 + c^2) - a^4 (b^3 + c^3) + 2 a^2 (b^5 - b^3 c^2 - b^2 c^3 + c^5) - (b - c)^2 (b^5 + 2 b^4 c + 5 b^3 c^2 + 5 b^2 c^3 + 2 b c^4 + c^5)):...:...),
    con (6,9,13)-número de búsqueda en ETC 13.41493102206036549146596589



     • El lugar geométrico de DC(P) cuando P recorre la recta de Euler del triángulo ABC es la cuártica de ecuación:
      Esta cuártica pasa por los vértices de ABC (dobles) y por los centros X2, X6, X275, X288, X1073, X2982, X2983, que son las imágenes DC(P) para P los centros X4, X2, X3, X5, X20, X21, X27, respectivamente.
    ( Mostrar/Ocultar figura )
      DC(X1113-X1114).png
    Descargar fichero GeoGebra

    Los otros dos puntos de intersección de la cuártica y cónica citadas son DC(X1113) y DC(X1114), cuyas primeras coordenadas baricéntricas son, respectivamente:

    a/((b c (2 a^4-(b^2-c^2)^2-a^2 (b^2+c^2))-a (a^2-b^2-c^2) Ω) (a b c (2 a^4-(b^2-c^2)^2-a^2 (b^2+c^2))+((b^2-c^2)^2-a^2 (b^2+c^2)) Ω))
    y
    a/((b c (2 a^4-(b^2-c^2)^2-a^2 (b^2+c^2))+a (a^2-b^2-c^2) Ω) (a b c (2 a^4-(b^2-c^2)^2-a^2 (b^2+c^2))-((b^2-c^2)^2-a^2 (b^2+c^2)) Ω)),

    donde Ω=Sqrt[a^6-a^4(b^2+c^2)-a^2(b^4-3b^2c^2+c^4)+(b^2-c^2)^2(b^2+c^2)].

    Con (6,9,13)-número de búsqueda en ETC -1.52553677731012746556326650 y 3.511353767441467116215236832, respectivamente.

    ∗  ∗  ∗  ∗  ∗  ∗  ∗  ∗  ∗  ∗ 


    NOTA: (Indicación de Bernard Gibert)

    La cuártica lugar geométrico de DC(P), cuando P recorre la recta de Euler, es el complemento del conjugado isogonal de la cónica circunscrita que pasa por baricentro y el retrocentro (isotómico del ortocentro).
    ( Mostrar/Ocultar figura )
      DCconica.png
    Descargar fichero GeoGebra
      Esta cónica pasa además por X95, X253, X264, X287, X305, X306, X307, X328, X1441, X1494, X1799, X1972, X2373, X2419.
      El centro de la cónica es el punto de coordenadas baricéntricas:

    ((b^2-c^2)^2(b^2+c^2-a^2)^2 : ... : ...)

    con (6,9,13)-número de búsqueda en ETC: 4.42563725188070946770285658

      El "octavo" punto de intersección de la cónica con la cuártica es:

    X=((a^2+b^2)(a^2+c^2)(a^4-b^4+c^4) (a^6-a^4(b^2+c^2)+a^2(b^4-c^4)-(b^2-c^2)(b^2+c^2)^2) : ... : ...)

    con (6,9,13)-número de búsqueda en ETC: 2.7443387717128226959052088

  • miércoles, 11 de febrero del 2015

    Centro X5502


    (ADGEOM #2342, Dao Thanh Oai)

      Sea ABC un triángulo, su recta de Euler corta a los lados BC, CA y AB en los puntos A', B' y C', respectivamente. Las perpendiculares en A', B' y C' a BC, CA y AB, respectivamente, delimitan un triángulo A1B1C1, cuyo circuncentro es el punto X5502.

    ( Mostrar/Ocultar figura )
      ADGEOM2342.png
    Descargar fichero GeoGebra

      En coordenadas baricéntricas:

    A' = (0 : (a^2-b^2)SC : (a^2-c^2)SB).
      Perpendicular en A' a BC:

    (b^2-c^2)SB SC x + a^2(a^2-c^2)SB y - a^2(b^2-a^2)SC z =0.


    A1 = (-(a^2 - b^2) (a^2 - c^2) (a^4 - (b^2 - c^2)^2) : 2 b^2 (-a^2 + b^2) (b^2 - c^2) (-a^2 + b^2 + c^2) : 2 c^2 (-a^2 + c^2) (-b^2 + c^2) (-a^2 + b^2 + c^2)).

      Los triángulos ABC y son semejantes y perspectivos. El centro de perspectividad es X110 (foco de la parábola de Kiepert); el centro de semejanza es el otro punto de intersección de las circunferencias circunscritas a ambos triángulos, X1304:

    (a^2 (a^4 - 2 b^4 + b^2 c^2 + c^4 + a^2 (b^2 - 2 c^2)) (a^4 + b^4 + b^2 c^2 - 2 c^4 + a^2 (-2 b^2 + c^2))/((b^2 - c^2) (b^2+c^2-a^2)) : ... : ... ).

      El circuncentro de A'B'C' es X5502:
    ( a^2(a^2-b^2)(a^2-c^2) (2a^6-4a^2(b^2-c^2)^2-a^4(b^2+c^2)+3(b^2-c^2)^2(b^2+c^2)): ... : ... ).

      Las recta de Euler de los triángulos ABC y A'B'C', que son perpendiculares, se cortan en el ortocentro de A'B'C', X4240:
    ( SB SC (2 a^4-(b^2-c^2)^2-a^2 (b^2+c^2))/(b^2-c^2) : ... : ... ).

  • domingo, 8 de febrero del 2015

    Centros ortológicos y ciclológicos sobre la hipérbola de Feuerbach


    (Anopolis #2407, Antreas Hatzipolakis)
    Euler lines and Neuberg cubic
    (Bernard Gibert. K001)

      For any point P on the Neuberg cubic, K001=pK(X6,X30), recall that the Euler lines (pa), (pb), (pc) of triangles PBC, PCA, PAB concur at M on the Euler line (e) of triangle ABC.
      Conversely, for a given point M on (e), we seek points on the Neuberg cubic having this property:
     There are two such points P, Q on K001 which are the common points of the rectangular circum-hyperbola (HM) through M and the line (d) passing through X399 and the reflection M' of X3 in M. Thus, the Euler lines of the six triangles PBC, PCA, PAB, QBC, QCA, QAB concur at M on (e).
     It follows that K001 can be seen as the locus of the intersections of (HM) and (d) when M traverses (e).
    ( Mostrar/Ocultar figura )
      Anopolis2407K001.png
    Descargar fichero GeoGebra

      Si {i,{j,k}} denota los índices de centros {M,{P,Q}} que figuran actualmente en ETC (M sobre la recta de Euler de ABC, P y Q sobre la cúbica de Neuberg), obtenemos la siguiente lista:
    {2, {13, 14}}, {3, {3, 74}}, {5, {4, 1263}}, {21, {1, 3065}}, {3129, {1338, 3440}}, {3130, {1337, 3441}}

    •   Punto de Schiffler M=X21 (P=X1, Q=X3065)

      En este caso, (HM) es la hipérbola de Feuerbach.
      Sea Qa el cuarto punto de intersección de la hipérbola de Feuerbach con la circunferencia que pasa por B, C y Q=X3065. Similarmente, se definen los puntos Qb y Qc.
      Qa = (1 : b(a-c)/(a^2-b^2+c^2-a(b+2c) : c(a-b)/(a^2+b^2-c^2-a(2b+c)))

      Sea Pa el cuarto punto de intersección de la hipérbola de Feuerbach con la circunferencia que pasa por B, C y P=X1. Similarmente, se definen los puntos Pb y Pc.
      Pa = (-1:b(a-c)/(a^2-b^2+c^2+a(b-2c)): c(a-b)/(a^2+b^2-c^2+a(c-2b)))

    ( Mostrar/Ocultar figura )
      Anopolis2407Todos.png


    NOTA: Los puntos Pa, Pb, Pc, Qa, Qb, Qc corresponde, respectivamente, a los puntos O'1, O'2, O'3, O1, O2, O3, de Anopolis #2407.

      Los triángulos ABC y PaPbPc son ciclológicos.

      El centro ciclológico de ABC respecto a PaPbPc es X1320.
      El centro ciclológico de PaPbPc respecto a ABC es X1.
    Bevan point and center X(1319)

      X(1320) = isogonal conjugate of X(1319)

      Let X'Y'Z' be the pedal triangle of the Bevan point, W = X(40); then X(1319) is the point, other than W, in which the circles AWX', BWY', CWZ' concur. (Floor van Lamoen, Hyacinthos #6321, 6352).
    ( Mostrar/Ocultar figura )
      X1320.png

      Los triángulos ABC y QaQbQc son ciclológicos.

      El centro ciclológico de ABC respecto a QaQbQc es:

    Z = (a (a^9 + a^7 (-6 b^2 + 7 b c - 6 c^2) - (b - c)^4 (b + c)^3 (2 b^2 + 5 b c + 2 c^2) + a^6 (2 b^3 - 3 b^2 c - 3 b c^2 + 2 c^3) + a (b^2 - c^2)^2 (3 b^4 - b^3 c + 2 b^2 c^2 - b c^3 + 3 c^4) + a^5 (12 b^4 - 15 b^3 c + 13 b^2 c^2 - 15 b c^3 + 12 c^4) + a^4 (-6 b^5 + 3 b^4 c + 5 b^3 c^2 + 5 b^2 c^3 + 3 b c^4 - 6 c^5) + a^3 (-10 b^6 + 9 b^5 c - 3 b^4 c^2 + 5 b^3 c^3 - 3 b^2 c^4 + 9 b c^5 - 10 c^6) + a^2 (6 b^7 + 3 b^6 c - 11 b^5 c^2 + b^4 c^3 + b^3 c^4 - 11 b^2 c^5 + 3 b c^6 + 6 c^7)):...:...)

    con (6,9,13)-número de búsqueda en ETC: 3.10306953362835054

      El centro ciclológico de QaQbQc respecto a ABC es X3065.

      Los triángulos PaPbPc y QaQbQc son ciclológicos.

      El centro ciclológico de PaPbPc respecto a QaQbQc es:

    T = (a (a^9 - a^8 (b + c) - (b - c)^4 (b + c)^5 + a^7 (-4 b^2 + 2 b c - 4 c^2) + a (b - c)^2 (b + c)^4 (b^2 + c^2) + 4 a^6 (b^3 + c^3) + a^5 (6 b^4 - 2 b^3 c + 9 b^2 c^2 - 2 b c^3 + 6 c^4) + a^4 (-6 b^5 + 2 b^4 c + b^3 c^2 + b^2 c^3 + 2 b c^4 - 6 c^5) - a^3 (4 b^6 + 2 b^5 c + 5 b^4 c^2 - 2 b^3 c^3 + 5 b^2 c^4 + 2 b c^5 + 4 c^6) + a^2 (4 b^7 - 5 b^5 c^2 - 3 b^4 c^3 - 3 b^3 c^4 - 5 b^2 c^5 + 4 c^7)) : ... : ...)

    con (6,9,13)-número de búsqueda en ETC: 5.40190694693080237

      El centro ciclológico de QaQbQc respecto a PaPbPc es:

    T' = (a (a^9 + 10 a^7 b c - 3 a^8 (b + c) + (b - c)^6 (b + c)^3 - a (b - c)^4 (b + c)^2 (3 b^2 - 2 b c + 3 c^2) + a^6 (8 b^3 - 6 b^2 c - 6 b c^2 + 8 c^3) - a^5 (6 b^4 + 12 b^3 c - 13 b^2 c^2 + 12 b c^3 + 6 c^4) - a^4 (6 b^5 - 18 b^4 c + 7 b^3 c^2 + 7 b^2 c^3 - 18 b c^4 + 6 c^5) - a^2 b c (6 b^5 - 13 b^4 c + 11 b^3 c^2 + 11 b^2 c^3 - 13 b c^4 + 6 c^5) + a^3 (8 b^6 - 6 b^5 c - 9 b^4 c^2 + 26 b^3 c^3 - 9 b^2 c^4 - 6 b c^5 + 8 c^6)) : .... : ...)

    con (6,9,13)-número de búsqueda en ETC: -3.57115072621920348

    PaPbPc y QaQbQc son ortológicos.

      El centro ortológico de PaPbPc respecto a QaQbQc es:

    W = (a^13 - 2 a^12 (b + c) - (b - c)^8 (b + c)^5 + a^11 (-3 b^2 + 5 b c - 3 c^2) + 2 a (b - c)^6 (b + c)^4 (b^2 - b c + c^2) - a^8 (b + c) (3 b^2 - 2 b c + 3 c^2)^2 + a^10 (7 b^3 + 2 b^2 c + 2 b c^2 + 7 c^3) + a^2 (b - c)^4 (b + c)^3 (3 b^4 - 3 b^3 c - 5 b^2 c^2 - 3 b c^3 + 3 c^4) + a^9 (4 b^4 - 14 b^3 c + 15 b^2 c^2 - 14 b c^3 + 4 c^4) - 3 a^7 (2 b^6 - 6 b^5 c + 6 b^4 c^2 - 7 b^3 c^3 + 6 b^2 c^4 - 6 b c^5 + 2 c^6) - a^4 (b + c)^3 (4 b^6 - 16 b^5 c + 26 b^4 c^2 - 27 b^3 c^3 + 26 b^2 c^4 - 16 b c^5 + 4 c^6) - a^3 (b^2 - c^2)^2 (7 b^6 - 17 b^5 c + 11 b^4 c^2 - 6 b^3 c^3 + 11 b^2 c^4 - 17 b c^5 + 7 c^6) + a^6 (6 b^7 - 4 b^6 c + 7 b^5 c^2 + 4 b^4 c^3 + 4 b^3 c^4 + 7 b^2 c^5 - 4 b c^6 + 6 c^7) + a^5 (9 b^8 - 20 b^7 c + 3 b^6 c^2 + 3 b^5 c^3 + 6 b^4 c^4 + 3 b^3 c^5 + 3 b^2 c^6 - 20 b c^7 + 9 c^8) : ... : ...)

    con (6,9,13)-número de búsqueda en ETC: -0.115636704440395011

      El centro ortológico de QaQbQc respecto a PaPbPc es:

    W' = (a^7 + a^5 b c - 2 a^6 (b + c) - (b - c)^4 (b + c)^3 - a^2 b c (b + c)^3 + 2 a (b^2 - c^2)^2 (b^2 + c^2) - a^3 (b + c)^2 (3 b^2 - 5 b c + 3 c^2) + a^4 (3 b^3 + 2 b^2 c + 2 b c^2 + 3 c^3) : .... : ...)

    con (6,9,13)-número de búsqueda en ETC: 8.85742096870961083

    ∗  ∗  ∗  ∗  ∗  ∗  ∗  ∗  ∗  ∗ 


    Kirikami-Schiffler points and generalizations found by Peter Moses, are introduced just before X(3647)

      Suppose that X is a point and A'B'C' is a central triangle. Let LA be the line through A' parallel to the Euler line of triangle BCX, let LB be the line through B' parallel to the Euler line of CXA, and let LC be the line through C' parallel to the Euler line of AXB.
    It is well known that if X=X(1), the incenter, then the three aforementioned Euler lines concur in the Schiffler point, X(21). If their parallels, the lines LA, LB, LC concur, the point of concurrence is the Kirikami-Schiffler point of the triangle A'B'C', denoted by KS(A'B'C'). Seiichi Kirikami (February 1, 2011) found that those lines concur if A'B'C' is the reference triangle ABC and also concur if A'B'C' is the medial triangle. Peter Moses found additional cases and properties.

      Suppose that A'B'C' is the cevian triangle of a point P. Then LA, LB, LC concur if and only if P lies on the cubic K455.
    ( Mostrar/Ocultar figura )
      Anopolis2407KSceviano.png
    Descargar fichero GeoGebra

      El punto Kirikami-Schiffler del triángulo PaPbPc es:

     KS(PaPbPc) =
    (a^7 - a^6(b+c) - a^5(b^2+c^2) + a^4(b^3+b^2c+b c^2+c^3) - a^3(b^4-3b^2c^2+c^4) + a^2(b^5-b^4c-b^3c^2-b^2c^3-b c^4+c^5) + a(b^2-c^2)^2(b^2+c^2) - (b-c)^4(b+c)^3 : ... : ...),

    de coordenadas trilineales exactas en el triángulo (6-9-13) de ETC:
    (2.10910338217546, 4.44623559966046, -0.410930955784615).

      El punto Kirikami-Schiffler del triángulo QaQbQc es:

     KS(QaQbQc) =
    (a^7 + a^6(b+c) + a^5(-3b^2+4b c-3c^2) - a^4(3b^3+b^2c+b c^2+3c^3) + a^3(3b^4-4b^3c-5b^2c^2-4b c^3+3c^4) + a^2(3b^5-b^4c-3b^3c^2-3b^2c^3-b c^4+3c^5) - a(b^2-c^2)^2(b^2+c^2)-(b-c)^4(b+c)^3 : ... : ...),

    de coordenadas trilineales exactas en el triángulo (6-9-13) de ETC:
    (2.18320070886206, 2.04428297862464, 1.21776055415406).
    ( Mostrar/Ocultar figura )
      Anopolis2407KSPQ.png
    Descargar fichero GeoGebra
      Los puntos Kirikami-Schiffler KS(PaPbPc) y KS(QaQbQc) están sobre la hipérbola de Feuerbach.




    •   Circuncentro M=X3 (P=X3, Q=X74)

      En este caso, (HM) es la hipérbola de Jerabek.
      El triángulo QaQbQc degenera en el punto X74, ya que Q=X74, conjugado isogonal del punto del infinito de la recta de Euler, está en la circunferencia circunscrita.

      Sea Pa el cuarto punto de intersección de la hipérbola de Jerabek con la circunferencia que pasa por B, C y P=X3. Similarmente, se definen los puntos Pb y Pc.
      Pa = (1 : (b^2(c^2-a^2))/(a^4-b^4+c^4+a^2(b^2-2c^2)) : ((b^2-a^2)c^2)/(a^4+b^4-c^4+a^2(c^2-2b^2))).

      Los triángulos ABC y PaPbPc son ciclológicos.

      El centro ciclológico de ABC respecto a PaPbPc es X5504.
      El centro ciclológico de PaPbPc respecto a ABC es X3.
    X(5504) Kirikami Concurrent Circles image of X(3)

      Let X be a point in the plane of triangle ABC. Let H1 be the orthocenter of triangle XBC, and define H2 and H3C cyclically. Let (O1) be the circle of the points A, H2, H3, and define (O2) and (O3) cyclically. The circles (O1), (O2), (O3) concur in a point Y, the Kirikami concurrent circles image of X.


    Hyacinthos #21992, Hatzipolakis - Kirikami

      Let ABC be a triangle and X,X* two isogonal conjugate points.
    Denote: H1,H2,H3 = the orthocenters of XBC, XCA, XAB, resp. and Ha,Hb,Hc = the orthocenters of X*BC, X*CA, X*AB, resp.
    The triangles H1H2H3 and HaHbHc are cyclologic, i.e. the circumcircles of:
    (1) H1HbHc, H2HcHa, H3HaHb   (2) HaH2H3, HbH3H1, HcH1H2   are concurrent.

    If X=X(3), X*=X(4) (HaHbHc = ABC), the orthologic center (ABC-H1H2H3) is X(5504) and the orthologic center (H1H2H3-ABC) is X(265).

    ( Mostrar/Ocultar figura )
      Anopolis2407MX3.png

      Los triángulos PaPbPc y H1H2H3 son simétricos respecto al centro X125 de la hipérbola de Jerabek y, por tanto, ciclológicos.

      El centro ciclológico de PaPbPc respecto a H1H2H3 es el cuarto punto de intersección de la hipérbola de Jerabek con la circunferencia circunscrita a PaPbPc:

    U = (a^2 (a^2 - b^2 - c^2) (a^12 - (b^2 - c^2)^6 - 4 a^10 (b^2 + c^2) - 16 a^6 b^2 c^2 (b^2 + c^2) + 4 a^2 (b^2 - c^2)^4 (b^2 + c^2) + a^8 (5 b^4 + 14 b^2 c^2 + 5 c^4) + a^4 (-5 b^8 + 12 b^6 c^2 + 2 b^4 c^4 + 12 b^2 c^6 - 5 c^8)) : ... : ...)

    con (6,9,13)-número de búsqueda en ETC: 7.467418731673146855045172

      El centro ciclológico de H1H2H3 respecto a PaPbPc es el cuarto punto de intersección de la hipérbola de Jerabek con la circunferencia circunscrita a H1H2H3:

    V = ((a^2 - b^2 - c^2)/(a^10 - 3 a^8 b^2 + 2 a^6 b^4 + 2 a^4 b^6 - 3 a^2 b^8 + b^10 - 3 a^8 c^2 + 7 a^6 b^2 c^2 - 7 a^4 b^4 c^2 + 5 a^2 b^6 c^2 - 2 b^8 c^2 + 2 a^6 c^4 - 7 a^4 b^2 c^4 + b^6 c^4 + 2 a^4 c^6 + 5 a^2 b^2 c^6 + b^4 c^6 - 3 a^2 c^8 - 2 b^2 c^8 + c^10) : ... : ...)

    con (6,9,13)-número de búsqueda en ETC: -0.486875328308682812075463

      Los puntos U y V son simétricos respecto al centro de simetría, X125.

  • martes, 3 de febrero del 2015

    Una propiedad del centro X5691


    (Anopolis #2347, Antreas Hatzipolakis)
    Outer Garcia triangle
      Let ABC be a triangle. The outer Garcia triangle A'B'C' is oppositely congruent to ABC at X(10) (Paul Yiu). Peter Moses reports that
    A' = (-a : a + c : a + b), B' = (b + c : -b : b + a), C' = (c + b : c + a : -c).

    X(5691) = De Longchamps point of outer Garcia Triangle.


      Sean ABC un triángulo, I el incentro y P un punto. Denotamos por I1 la reflexión de I en AP y por IA la reflexión de I1 en AI.

      Si P(p:q:r), las coordenadas baricéntricas de IA son:
    (a^3 b c q r - a b c (q + r) (c^2 q + b^2 r) + a^2 (-c^3 q^2 + b^2 c q r + b c^2 q r - b^3 r^2) + (b^2 - c^2) (-c^3 q^2 - b^2 c q r + b c^2 q r + b^3 r^2) :
         -b^2 (-c^3 q^2 + b (-a^2 + b^2) r^2 + b^2 c r^2 + b c^2 r (2 q + r)) :
         -c^2 (-a^2 c q^2 + b c^2 q^2 + c^3 q^2 - b^3 r^2 + b^2 c q (q + 2 r)) ).

      Similarmente, se definen los puntos IB y IC.

      Los triángulos ABC y IAIBIC son ortológicos si y solo si P está en la circunferencia circunscrita o sobre una cuártica que pasa por los vértices de los triángulos ABC y circunceviano de I, por los puntos X1, X3, X36, X3307, X3308.

      Si P está en la circunferencia circunscrita su triángulo pedal degenera: sus vértices quedan sobre la recta de Simson-Wallace de P. En este caso, los puntos IA, IB y IC están alineados, sobre la recta paralela por I a la recta de Simson-Wallace de P.
    ( Mostrar/Ocultar figura )
      Anopolis2347OR.png


      X36 es el inverso de X1 en la circunferencia circunscrita. Los puntos X3307, X3308 son los conjugados isogonales de X1381, X1382, puntos en los que la recta X1X3 corta a la circunferencia circunscrita.

      Las bisectrices interiores son tangentes a la cuártica en los vértices de ABC y vuelven a cortar al circunferencia circunscrita en puntos de la curva. La recta X1X3 es tangente a la cuártica en X1.
    ( Mostrar/Ocultar figura )
      Anopolis2347cuartica.png

      Los centros de ortología de ABC respecto a IAIBIC, para P = X3, X36, X3307, X3308 son, respectivamente, Q = X102, X1, X4, X4.

      Los centros de ortología de IAIBIC respecto a ABC, para P = X3, X36 son, respectivamente, Q = X5691, X80.

      Se tiene la siguiente propiedad para el punto de De Longchamps del triángulo reflexión de ABC en el centro de Spieker:

      Let I = X(1) and O = X(3). Let I1 be the reflection of I in line AO and let IA be the reflection of I1 in line AI. Define IB and IC cyclically. Then ABC and IAIBIC are orthologic triangles, and X5691 is the ABC-orthology center of IAIBIC.



  • viernes, 30 de enero del 2015

    La cúbica K099 como lugar geométrico


    Darboux perspector cubic
    (CTC K009, Bernard Gibert)

      Denote by A', B', C' the antipodes of A, B, C on the circumcircle. For any point P, Pa is the trace of PA' on BC, Pb, Pc similarly.
      The triangles ABC and PaPbPc are perspective if and only if P lies on the Darboux cubic.
      The locus of the perspector is the Darboux perspector cubic, K099 = pK(X394, X69).

      Dados un triángulo ABC y dos puntos P y U, se denota por PaPbPc y UaUbUc los triángulo cevianos de P y U, respectivamente; y por P1P2P3 el triángulo pedal de P.
      Sobre la recta BC se toma el punto Vaa(Ua), conjugado de Ua en la involución σa de pares de puntos conjugados (B,C) y (Pa, P1). De forma similar se definen Vb y Vc

      Para cualquier punto U, los triángulos ABC y VaVbVc son perspectivos si y solo si P está sobre la cúbica de Darboux, K004=pK(X6,X2).

    ( Mostrar/Ocultar figura )
      Involucion Ceviano Pedal.png

      Si, en coordenadas baricéntricas, P=(p:q:r) y U=(u:v:w), se tiene que:
    Va = (0 : q((b^2-c^2)p+a^2(p+2q))w : r((-b^2+c^2)p+a^2(p+2r))v)

      Si P está sobre la cúbica de Darboux, el centro de perspectividad V de ABC y VaVbVc, tiene coordenadas:

    (pvw(a^4(5qr+p(q+r))+ 2a^2p(c^2(p+3q)+b^2(p+3r))+2b^2c^2(2p+q)(2p+r) - c^4(2p^2+3pq+pr+qr) - b^4(2p^2+3pr+pq+qr)) : ... : ...)


      Así, la transformación U ↦ V (cuando P está sobre la cúbica de Darboux) es una Ωo-isoconjugación con polo:

    Ωo = (p(a^4(5qr+p(q+r))+ 2a^2p(c^2(p+3q)+b^2(p+3r))+2b^2c^2(2p+q)(2p+r) - c^4(2p^2+3pq+pr+qr) - b^4(2p^2+3pr+pq+qr)) : ... : ...)

    Para P entre los puntos X1, X3, X4, X20, X40, X64, X84, X1490, X1498, X2130, X2131, X3182, X3183, X3345, X3346, X3347, X3348, X3353, X3354, X3355, X3472, X3473, X3637 sobre la cúbica de Darboux, tenemos los pares (P, Ωo):

    (X1, X57), (X3, X3), (X4, X393), (X40, X2324), (X84, X1256).



    EJEMPLO

      Para P= X3 (circuncentro), también Ωo=X3, y si U varía en la cúbica de Thomson (K002=pK(X6,X2)), el lugar geométrico del centro de perspectividad V (X3-isocunjugado de U) de los triángulos ABC y VaVbVc es "Darboux perspector cubic", K099 = pK(X394,X69).
    ( Mostrar/Ocultar figura )
      Darboux perspector cubic.png


    NOTA:
      Si Ω=(p:q:r}, P=(u:v:w) y Ωo=(α:β:γ), la imagen de la isocúbica pivotal pK(Ω,P), mediante una Ωo-isoconjugación, es la isocúbica pivotal de polo (α^2/p :β^2/q :γ^2/r) y pivote (uα/p : vβ/q : wγ/r).
      §2. The Euler line of a quadrilateral
    (Forum Geometricorum Volume 6 (2006) 289–295, Alexei Myakishev)

      Given a quadrilateral ABCD, denote by Oa and Ha the circumcenter and the orthocenter respectively of triangle BCD, and similarly, Ob, Hb for triangle ACD, Oc, Hc for triangle ABD, and Od, Hd for triangle ABC.
      Let O =OaOc ∩ ObOd, H =HaHc ∩ HbHd.
      We shall call O the quasicircumcenter and H the quasiorthocenter of the quadrilateral ABCD. Clearly, the quasicircumcenter O is the intersection of perpendicular bisectors of the diagonals of ABCD. Therefore, if the quadrilateral is cyclic, then O is the center of its circumcircle.
      The Euler line of a quadrilateral ABCD as the line containing the "quasicentroid", the quasicircumcenter, and the quasiorthocenter.
    ( Mostrar/Ocultar figura )
      EulerLineQuadrilateral.png

    QG-L4: 1st QG-Quasi Euler line
    (Encyclopedia of Quadri-Figures, Chris van Tienhoven)

      A quadrigon is a geometrical figure consisting of 4 consecutive points and 4 consecutive connecting lines. There is a cyclic order in these points and lines.
      The 1st QG-Quasi Euler line is the line through QG-P4 (1st Quasi Centroid), QG-P5 (1st Quasi Circumcenter), QG-P6 (1st Quasi Orthocenter) and QG-P7 (1st Quasi Nine-point Center).

    QG-L5: 2nd QG-Quasi Euler line

    The 2nd QG-Quasi Euler line is the line through QG-P8 (2nd Quasi Centroid), QG-P9 (2nd Quasi Circumcenter), QG-P10 (2nd Quasi Orthocenter) and QG-P11 (2nd Quasi Nine-point Center).

    QL-L7 is the Euler Line of the QL-Diagonal Triangle.

      Dados un triángulo ABC y un punto P, se denota por P* el conjugado isogonal de P y por DEF el triángulo pedal de P.
      Entonces, las "rectas de Euler", da, db, dc, de los cuadriláteros cíclicos orientados AEPF, BFPD, CDPE, respectivamente, concurren en el punto medio de PP*.

    ( Mostrar/Ocultar figura )
      Anopolis2366Go.png
      Si P=(p:q:r), el "cuasi-circuncentro" es Oa=(2p+q+r:q:r) y el baricentro Goa es:

    (a^2(c^2(a^2-c^2)q-b^4r + b^2(a^2r+c^2(4p+q+r))) : b^2(c^2(b^2-c^2)p-a^4r + a^2(b^2r+c^2(p+4q+r))) : c^2(b^2(-b^2+c^2)p-a^4q+ a^2(c^2q+b^2(p+q+4 r))))

      La ecuación de la "recta de Euler" GoaOa del cuadrilátero cíclico orientado AEPF es da:

    -b^2c^2(b^2-c^2)p(q+r) + a^4(-c^2q^2+b^2r^2) + a^2(c^4q^2-b^4r^2+b^2c^2(q-r)(p+q+r))x +
    b^2c^2(b^2-c^2)p(2p+q+r) + a^4(b^2r^2+c^2q(2p+q+2r)) - a^2(b^4r^2+c^4q(2p+q+2r) + b^2c^2(2p^2+3p q + q^2 + 5p r + 4q r+3r^2))y+
    +b^2c^2(b^2-c^2)p(2p+q+r) - a^4(c^2q^2+b^2r(2p+2q+r)) + a^2(c^4q^2+b^4r(2p+2q+r) + b^2c^2(2p^2+5p q + 3q^2 + 3p r+4q r +r^2))z=0.


      Dados un triángulo ABC y un punto P, se denota por DEF el triángulo pedal de P.
      Entonces, las "rectas de Euler", en el sentido de Alexei Myakishev, de los cuadriláteros cíclicos orientados AEPF, BFPD, CDPE, respectivamente, son concurrente si y solo si P queda en una nónica que pasa por A, B, C (puntos triples), el incentro y los exincentros.

    ( Mostrar/Ocultar figura )
      Anopolis2366Nueve.png
      Si P=(p:q:r), el "cuasi-circuncentro" es Oa=(2p+q+r:q:r) y el "quasi-baricentro" Goa es:

    (c^8 q^3 - 2 b^2 c^6 q^2 (3 p + q) + b^8 r^3 - 2 b^6 c^2 r^2 (3 p + r) + a^4 (c^4 q^3 + b^4 r^3) + a^2 (-2 c^6 q^3 + 2 b^2 c^4 q^2 (3 p + q) - 2 b^6 r^3 + 2 b^4 c^2 r^2 (3 p + r)) - 3 b^4 c^4 (q^3 + 4 q^2 r + 4 q r^2 + r^3 + 2 p (q^2 + 4 q r + r^2)) :
    -b^2 (2 c^6 q^3 + a^4 b^2 r^3 + b^6 r^3 + 2 b^4 c^2 r^2 (3 q + r) + b^2 c^4 (2 q^3 + 12 q^2 r + 6 q r^2 + r^3) - 2 a^2 (c^4 q^3 + b^4 r^3 + b^2 c^2 r^2 (3 q + r))) :
    -c^2 (a^4 c^2 q^3 + c^6 q^3 + 2 b^6 r^3 + 2 b^2 c^4 q^2 (q + 3 r) + b^4 c^2 (q^3 + 6 q^2 r + 12 q r^2 + 2 r^3) - 2 a^2 (c^4 q^3 + b^4 r^3 + b^2 c^2 q^2 (q + 3 r))))

      La ecuación de la "recta de Euler" GaOa del cuadrilátero cíclico orientado AEPF es:

    (c^6 q^4 - b^6 r^4 - b^4 c^2 r^3 (4 q + r) + b^2 c^4 q^3 (q + 4 r) + a^2 (-c^4 q^4 + b^4 r^4))x +
    (-b^6 r^4 + b^4 c^2 r^3 (2 p - 2 q + r) - c^6 q^3 (2 p + q + 2 r) - b^2 c^4 q^2 (2 p q + q^2 + 6 p r + 4 q r + 6 r^2) + a^2 (b^4 r^4 + c^4 q^3 (2 p + q + 2 r)))y +
    (c^4 (-a^2 + c^2) q^4 - b^2 c^4 q^3 (2 p + q - 2 r) + b^6 r^3 (2 p + 2 q + r) + b^4 r^2 (-a^2 r (2 p + 2 q + r) + c^2 (6 p q + 6 q^2 + 2 p r + 4 q r + r^2)))z = 0.


    NOTA: Queda estudiar otras posibles "rectas de Euler" de un cuadrilátero (ver mensajes de Seiichi Kirikami y Chris van Tienhoven, relacionados con Anopolis #2365 de asunto: "A property of X(597) and a locus").

  • martes, 27 de enero del 2015

    Séptica asociada a puntos de Kosnita


    (ADGEOM #2269, Tran Quang Hung)

      Let O be the circumcenter of triangle ABC, and Oa the circumcenter of triangle BOC. Define Ob and Oc cyclically. Then the lines AOa, BOb, COc concur in X(54)=Kosnita Point.

      Dados un triángulo ABC y un punto P, se denota por Ka, Kb, Kc los puntos de Kosnita, X54, de los triángulos PBC, PCA, PAB. Sean da, db, dc las reflexiones de las rectas PKa, PKb, PKc en BC, CA, AB, respectivamente.
      El lugar geométrico de los puntos P tales que da, db, dc son concurrentes en un punto Q es una bicircular séptica que pasa por A, B, C (puntos singulares aislados), Ha, Hb, Hc (pies de las alturas), X3, X4, X5, X80, X265, X1113, X1114, por los puntos donde las mediatriz de X3X4 corta a la circunferencia circunscrita.

    ( Mostrar/Ocultar figura )
      ADGEOM2268.png
      Si P(p:q:r), las coordenadas baricéntricas de Ka son:

    (-a^2 (a^4 p^3 - a^2 b^2 p^3 - 2 a^2 c^2 p^3 - b^2 c^2 p^3 + c^4 p^3 + 2 a^4 p^2 q - 2 a^2 b^2 p^2 q - 2 a^2 c^2 p^2 q + a^4 p q^2 - a^2 b^2 p q^2 + a^2 c^2 p q^2 + a^4 p^2 r + b^4 p^2 r - 2 a^2 c^2 p^2 r - 2 b^2 c^2 p^2 r + c^4 p^2 r + 2 a^4 p q r + 2 a^2 b^2 p q r - 2 a^2 c^2 p q r + 2 a^4 q^2 r) (a^4 p^3 - 2 a^2 b^2 p^3 + b^4 p^3 - a^2 c^2 p^3 - b^2 c^2 p^3 + a^4 p^2 q - 2 a^2 b^2 p^2 q + b^4 p^2 q - 2 b^2 c^2 p^2 q + c^4 p^2 q + 2 a^4 p^2 r - 2 a^2 b^2 p^2 r - 2 a^2 c^2 p^2 r + 2 a^4 p q r - 2 a^2 b^2 p q r + 2 a^2 c^2 p q r + a^4 p r^2 + a^2 b^2 p r^2 - a^2 c^2 p r^2 + 2 a^4 q r^2) :
    -(-a^2 b^4 p^3 + b^6 p^3 - a^2 b^2 c^2 p^3 - 2 b^4 c^2 p^3 + b^2 c^4 p^3 + a^6 p^2 q - 3 a^4 b^2 p^2 q + a^2 b^4 p^2 q + b^6 p^2 q - 3 a^4 c^2 p^2 q - 4 a^2 b^2 c^2 p^2 q - 3 b^4 c^2 p^2 q + 3 a^2 c^4 p^2 q + 3 b^2 c^4 p^2 q - c^6 p^2 q + a^6 p q^2 - 3 a^4 b^2 p q^2 + 2 a^2 b^4 p q^2 - 3 a^4 c^2 p q^2 - 4 a^2 b^2 c^2 p q^2 + 2 a^2 c^4 p q^2 - a^4 b^2 p^2 r - a^2 b^4 p^2 r + a^2 b^2 c^2 p^2 r - 4 a^4 b^2 p q r - a^6 q^2 r - a^4 b^2 q^2 r + a^4 c^2 q^2 r) (a^4 p^3 - 2 a^2 b^2 p^3 + b^4 p^3 - a^2 c^2 p^3 - b^2 c^2 p^3 + a^4 p^2 q - 2 a^2 b^2 p^2 q + b^4 p^2 q - 2 b^2 c^2 p^2 q + c^4 p^2 q + 2 a^4 p^2 r - 2 a^2 b^2 p^2 r - 2 a^2 c^2 p^2 r + 2 a^4 p q r - 2 a^2 b^2 p q r + 2 a^2 c^2 p q r + a^4 p r^2 + a^2 b^2 p r^2 - a^2 c^2 p r^2 + 2 a^4 q r^2) :
    (a^4 p^3 - a^2 b^2 p^3 - 2 a^2 c^2 p^3 - b^2 c^2 p^3 + c^4 p^3 + 2 a^4 p^2 q - 2 a^2 b^2 p^2 q - 2 a^2 c^2 p^2 q + a^4 p q^2 - a^2 b^2 p q^2 + a^2 c^2 p q^2 + a^4 p^2 r + b^4 p^2 r - 2 a^2 c^2 p^2 r - 2 b^2 c^2 p^2 r + c^4 p^2 r + 2 a^4 p q r + 2 a^2 b^2 p q r - 2 a^2 c^2 p q r + 2 a^4 q^2 r) (a^2 b^2 c^2 p^3 - b^4 c^2 p^3 + a^2 c^4 p^3 + 2 b^2 c^4 p^3 - c^6 p^3 + a^4 c^2 p^2 q - a^2 b^2 c^2 p^2 q + a^2 c^4 p^2 q - a^6 p^2 r + 3 a^4 b^2 p^2 r - 3 a^2 b^4 p^2 r + b^6 p^2 r + 3 a^4 c^2 p^2 r + 4 a^2 b^2 c^2 p^2 r - 3 b^4 c^2 p^2 r - a^2 c^4 p^2 r + 3 b^2 c^4 p^2 r - c^6 p^2 r + 4 a^4 c^2 p q r - a^6 p r^2 + 3 a^4 b^2 p r^2 - 2 a^2 b^4 p r^2 + 3 a^4 c^2 p r^2 + 4 a^2 b^2 c^2 p r^2 - 2 a^2 c^4 p r^2 + a^6 q r^2 - a^4 b^2 q r^2 + a^4 c^2 q r^2))


      La primera coordenada del punto Q, de intersección de las rectas dadbdc, es:

    a^10 (3 q^2 (q - r)^2 r^2 (q + r) + p^4 (q^3 + r^3) + 2 p q r (q^4 + q^3 r - 2 q^2 r^2 + q r^3 + r^4) + p^3 (2 q^4 + q^3 r - q^2 r^2 + q r^3 + 2 r^4) + p^2 (q^5 + 3 q^4 r - 2 q^3 r^2 - 2 q^2 r^3 + 3 q r^4 + r^5))

    - a^8(b^2 (-p^5 q^2 + 3 q^3 r^2 (2 q^2 + q r - 3 r^2) + p^4 (q^3 + 2 q^2 r + q r^2 + 3 r^3) + 2 p q r (2 q^4 + 3 q^3 r + q^2 r^2 + 2 q r^3 - 2 r^4) + p^2 q (3 q^4 + 6 q^3 r + q^2 r^2 - 3 q r^3 - r^4) + p^3 (5 q^4 + 4 q^3 r + 2 q^2 r^2 + 3 q r^3 + 3 r^4)) + c^2 (-p^5 r^2 + 3 q^2 r^3 (-3 q^2 + q r + 2 r^2) + p^4 (3 q^3 + q^2 r + 2 q r^2 + r^3) + 2 p q r (-2 q^4 + 2 q^3 r + q^2 r^2 + 3 q r^3 + 2 r^4) + p^2 r (-q^4 - 3 q^3 r + q^2 r^2 + 6 q r^3 + 3 r^4) + p^3 (3 q^4 + 3 q^3 r + 2 q^2 r^2 + 4 q r^3 + 5 r^4)))

    -a^6(b^4 (p^5 q (3 q + 2 r) + p^4 (3 q^3 - 2 q^2 r - 4 q r^2 - 4 r^3) + 3 q^2 r^2 (-2 q^3 - 2 q^2 r + q r^2 + r^3) - p q r (2 q^4 + 6 q^3 r + q^2 r^2 + 10 q r^3 - 2 r^4) - p^3 (3 q^4 + 6 q^3 r + 2 q^2 r^2 + 5 q r^3 + 3 r^4) - p^2 (3 q^5 + 4 q^4 r - 2 q^3 r^2 - 4 q^2 r^3 - 10 q r^4 + r^5)) + b^2 c^2 (6 q^3 r^3 (q + r) - 2 p^4 (q + r)^3 + p^5 (3 q^2 + q r + 3 r^2) - p^3 (4 q^4 + 9 q^3 r + 12 q^2 r^2 + 9 q r^3 + 4 r^4) + p q r (6 q^4 + 5 q^3 r - 23 q^2 r^2 + 5 q r^3 + 6 r^4) + p^2 (q^5 + q^4 r + 8 q^3 r^2 + 8 q^2 r^3 + q r^4 + r^5)) + c^4 (p^5 r (2 q + 3 r) - p^4 (4 q^3 + 4 q^2 r + 2 q r^2 - 3 r^3) + 3 q^2 r^2 (q^3 + q^2 r - 2 q r^2 - 2 r^3) + p q r (2 q^4 - 10 q^3 r - q^2 r^2 - 6 q r^3 - 2 r^4) - p^3 (3 q^4 + 5 q^3 r + 2 q^2 r^2 + 6 q r^3 + 3 r^4) - p^2 (q^5 - 10 q^4 r - 4 q^3 r^2 - 2 q^2 r^3 + 4 q r^4 + 3 r^5)))

    - a^4(b^4 c^2 (-3 q^4 r^2 (q + 3 r) - p^5 (4 q^2 + 7 q r + 4 r^2) + p^4 q (q^2 + 3 q r + 7 r^2) + p^2 r (-3 q^4 - 3 q^3 r - 21 q^2 r^2 - q r^3 + r^4) + p^3 (5 q^4 + 8 q^3 r + 5 q^2 r^2 + 13 q r^3 + r^4) - p q r (4 q^4 + 6 q^3 r - 5 q r^3 + 2 r^4)) + b^2 c^4 (-3 q^2 r^4 (3 q + r) + p^4 r (7 q^2 + 3 q r + r^2) - p^5 (4 q^2 + 7 q r + 4 r^2) + p^2 q (q^4 - q^3 r - 21 q^2 r^2 - 3 q r^3 - 3 r^4) - p q r (2 q^4 - 5 q^3 r + 6 q r^3 + 4 r^4) + p^3 (q^4 + 13 q^3 r + 5 q^2 r^2 + 8 q r^3 + 5 r^4)) + b^6 (-p^5 (3 q^2 + 4 q r + r^2) + p q r^3 (7 q^2 + 4 q r + 2 r^2) + q^2 r^2 (4 q^3 + 8 q^2 r + q r^2 - r^3) + p^4 (-5 q^3 - 2 q^2 r + 8 q r^2 + 4 r^3) + p^3 (-q^4 + 4 q^3 r + 5 q^2 r^2 + 4 r^4) + p^2 (q^5 + 2 q^4 r - 8 q^3 r^2 - 3 q^2 r^3 - 10 q r^4 - r^5)) + c^6 (-p^5 (q^2 + 4 q r + 3 r^2) + p q^3 r (2 q^2 + 4 q r + 7 r^2) + p^4 (4 q^3 + 8 q^2 r - 2 q r^2 - 5 r^3) + q^2 r^2 (-q^3 + q^2 r + 8 q r^2 + 4 r^3) + p^3 (4 q^4 + 5 q^2 r^2 + 4 q r^3 - r^4) - p^2 (q^5 + 10 q^4 r + 3 q^3 r^2 + 8 q^2 r^3 - 2 q r^4 - r^5)))

    - a^2(c^8 (p^5 (q + r)^2 - p q^2 r^2 (5 q^2 + q r - 3 r^2) - q^2 r^3 (4 q^2 + 5 q r + r^2) + p^4 (-5 q^3 - 6 q^2 r + 2 q r^2 + 2 r^3) + p^2 q (2 q^4 + 4 q^3 r + 10 q^2 r^2 + 4 q r^3 - r^4) + p^3 (q^4 + 2 q^3 r - 7 q^2 r^2 - q r^3 + r^4)) + b^8 (p^5 (q + r)^2 + p q^2 r^2 (3 q^2 - q r - 5 r^2) - q^3 r^2 (q^2 + 5 q r + 4 r^2) + p^4 (2 q^3 + 2 q^2 r - 6 q r^2 - 5 r^3) + p^3 (q^4 - q^3 r - 7 q^2 r^2 + 2 q r^3 + r^4) + p^2 r (-q^4 + 4 q^3 r + 10 q^2 r^2 + 4 q r^3 + 2 r^4)) + b^2 c^6 (-2 q^3 r^2 (q^2 - 4 q r - 5 r^2) + p^5 (5 q^2 + 5 q r + r^2) + p^4 (q^3 - 5 q^2 r - 4 q r^2 - 2 r^3) + p q r (-2 q^4 + 7 q^3 r + 2 q^2 r^2 - q r^3 + 2 r^4) - p^3 (4 q^4 + 2 q^3 r - 4 q^2 r^2 + 11 q r^3 + 4 r^4) - p^2 (3 q^5 + 3 q^4 r - q^3 r^2 - 7 q^2 r^3 + q r^4 + r^5)) + b^4 c^4 (-2 p q^2 r^2 (2 q^2 + q r + 2 r^2) + p^5 (2 q^2 + 9 q r + 2 r^2) + 3 q^2 r^2 (q^3 - 3 q^2 r - 3 q r^2 + r^3) + p^4 (2 q^3 - 3 q^2 r - 3 q r^2 + 2 r^3) + p^3 (4 q^4 + 4 q^3 r - 6 q^2 r^2 + 4 q r^3 + 4 r^4) + p^2 (2 q^5 + 7 q^4 r - 4 q^3 r^2 - 4 q^2 r^3 + 7 q r^4 + 2 r^5)) + b^6 c^2 (2 q^2 r^3 (5 q^2 + 4 q r - r^2) + p^5 (q^2 + 5 q r + 5 r^2) + p^4 (-2 q^3 - 4 q^2 r - 5 q r^2 + r^3) + p q r (2 q^4 - q^3 r + 2 q^2 r^2 + 7 q r^3 - 2 r^4) - p^3 (4 q^4 + 11 q^3 r - 4 q^2 r^2 + 2 q r^3 + 4 r^4) - p^2 (q^5 + q^4 r - 7 q^3 r^2 - q^2 r^3 + 3 q r^4 + 3 r^5)))

    -(-c^2 q + b^2 (p + q)) (q + r) (b^2 r - c^2 (p + r)) (b^6 (p + q) (p - r) (p - q - r) r + c^6 (p - q) q (p - q - r) (p + r) + b^4 c^2 (p (2 q - r) r^2 + p^3 (q + r) + q r (q^2 - q r - 2 r^2) + p^2 (q^2 - 2 q r - r^2)) + b^2 c^4 (-p q^2 (q - 2 r) + p^3 (q + r) + p^2 (-q^2 - 2 q r + r^2) + q r (-2 q^2 - q r + r^2)))

      Algunos pares (P,Q):   (X(3),X(3)), {X(4),X(186)), (X(80),X(3))

    NOTA: La información sobre los puntos de la séptica en la mediatriz de OH y comunes con la isocúbica pK(X6,X382) en la recta del infinito, son aportados por Bernard Gibert.

  • domingo, 25 de enero del 2015

    Triángulos ortíacos y rectas de Euler


    (TTW #r2068, Joaquín Castellsaguer)

      Sean AaAbAc, BbBcBa, CcCaCb los triángulos ortíacos de ABC. Sean A1, A2, A3 los puntos medios de AaA, AaAb, AaAc y definimos cíclicamente B1, B2, B3, C1, C2, C3.
      Entonces:
      1) Las rectas de Euler de A1A2A3, B1B2B3 y C1C2C3 concurren en un punto W.
      2) Sus paralelas por A, B, C concurren en X74, conjugado isogonal del punto del infinito de la recta de Euler de ABC.
      3) Sus paralelas por Aa, Bb, Cc concurren en X1982, "Hatzipolakis Reflection Point".
      4) Los tres puntos de concurrencia están alineados con el simediano X6 y se cumple W=X74+3X1986.

    ( Mostrar/Ocultar figura )
      r2068.png
    Descargar fichero GeoGebra

      Los vértices del triángulo órtico son Aa(0:SC:SB), Bb(SC:0:SA), Cc(SB:SA:0).
      La proyección ortogonal de Aa sobre AC es Ab(SC^2:0:S^2).
      La proyección ortogonal de Aa sobre AB es Ac(SB^2:S^2:0).
      Los puntos medios A1, A2, A3 de AaA, AaAb, AaAc son:
    A1(a^2:SC:SB), A2(SC^2:b^2SC:2S^2-SASC), A3(SB^2:2S^2-SASB:c^2SB).

      1)   La recta de Euler de A1A2A3 es:
    3 (a - b - c) (b - c) (a + b - c) (a - b + c) (b + c) (a + b + c) (a^2 - b^2 - c^2)x + a^2 (b^2 - c^2)^2 (9 b^2 - c^2) - 3 (b^2 - c^2)^3 (b^2 + c^2) + a^6 (3 b^2 + 5 c^2) + a^4 (-9 b^4 + 8 b^2 c^2 - 7 c^4)y + a^2 (b^2 - 9 c^2) (b^2 - c^2)^2 - 3 (b^2 - c^2)^3 (b^2 + c^2) - a^6 (5 b^2 + 3 c^2) + a^4 (7 b^4 - 8 b^2 c^2 + 9 c^4)=0.

      Permutando cíclicamente , se obtiene las ecuaciones de las rectas de Euler de los triángulos B1B2B3 y C1C2C3, las cuales concurren en el punto W, de primera coordenada baricéntrica:

    a^2(3a^12(b^2+c^2) - 4a^10(3b^4+b^2c^2+3c^4) + a^8(15b^6+4b^4c^2+4b^2c^4+15c^6) - 8a^6(3b^6c^2-4b^4c^4+3b^2c^6) - a^4(b^2-c^2)^2(15b^6-13b^4c^2-13b^2c^4+15c^6) + 4a^2(b^2-c^2)^4(3b^4+5b^2c^2+3c^4) - 3(b^2-c^2)^4(b^6+2b^4c^2+2b^2c^4+c^6) ),

    con coordenadas trilineales exactas en el triángulo (6,9,13) de ETC: {5.07545904659784472, 5.40986924589424550, -2.44714917137218073}


      2)   Las paralelas por A, B, C a las rectas de Euler de A1A2A3, B1B2B3 y C1C2C3 , son:

    c^2 (S^2 - 3 SA SC)y -b^2 (S^2 - 3 SA SB)z =0,
    -c^2 (S^2 - 3 SB SC)x + a^2 (S^2 - 3 SC SA)z=0,
    b^2 (S^2 - 3 SC SB)x -a^2 (S^2 - 3 SC SA)y =0,

    que concurren en X74.


      3)   Las paralelas por Aa, Bb, Cc a las rectas de Euler de A1A2A3, B1B2B3 y C1C2C3 concurren el punto X1986, de primera coordenada:

    a^2SB SC(S^2-3 SA^2)(a^2 (SB SC- SA^2)+SA (SB - SC)^2 ).

    Paralela por Aa a la recta de Euler de A1A2A3:

    (a-b-c)(b-c)(a+b-c)(a-b+c)(b+c)(a+b+c)(a^2-b^2-c^2)x + (a^2-b^2+c^2)(a^4(b^2+c^2)+(b^2-c^2)^2(b^2+c^2)- 2a^2(b^4-b^2c^2+c^4))y -(a^2+b^2-c^2)(a^4(b^2+c^2)+(b^2-c^2)^2(b^2+c^2)- 2a^2(b^4-b^2c^2+c^4))z = 0.

  • viernes, 16 de enero del 2015

    Cúbica pK(G/P,G) y cúbica central asociada

    a Aye, por su "cumple"



    ( International Journal of Computer-Generated Mathematics Volume 10(2015)1, Sava Grozdev and Deko Dekov)

      Given points U and P in the plane of ▲ABC . Let ▲UaUbUc be the anticevian triangle of U. Denote by Ra , Rb and Rc the reflections of Ua , Ub and Uc in P. If the lines ARa , BRb and CRc concur in a point, we say that the Prasolov anticevian product is defined. In this case the intersection point of the lines is the Prasolov anticevian product of points U and P.
    ( Mostrar/Ocultar figura )
      JCGM2015p001.png
    Problem. Prove that if U is the Incenter of ▲ABC and P is the Spieker center of ▲ABC , then the Prasolov anticevian product is defined and the Prasolov anticevian product of points X(1) and X(10) is X(4).

      El lugar geométrico de los puntos U tales que el triángulo reflexión RaRbRc, del triángulo anticeviano UaUbUc de U respecto al centro de Spieker, es perspectivo con ABC es la cúbica K345=pK(X37,X2). El centro de perspectividad V queda en K033=pK(X37,X8), "Spieker central cubic".

    ( Mostrar/Ocultar figura )
      JCGM2015p001_K345.png
    Descargar fichero GeoGebra


    CASO GENERAL

      Sean dos puntos U=(u:v:w) y P=(p:q:r) dados por sus coordenadas baricéntricas, UaUbUc el triángulo anticeviano de U y RaRbRc el triángulo cuyos vértices son las reflexiones de los correspondientes vértices de UaUbUc en P. La condición algebraica para que las rectas ARa, BRb y CRc sean concurrentes es:
      La primera coordenada del punto de concurrencia es:
    (q+r)^2u(3u-v-w) + p^2(u^2-6(v-w)^2-u(v+w)) - 2 p(r(2u^2-3u v+3v^2+u w-2v w-w^2) + q(2u^2+u v-v^2-3u w-2v w+3w^2))

      Si el punto P es fijo, el lugar geométrico de los puntos U tales que las rectas ARa, BRb y CRc son concurrentes, en un punto V, es la isocúbica pivotal C(P)=pK(X2/P,X2).
      El punto V queda sobre la isocúbica central pK(X2/P,hG,4(P)), de centro P.

    X2/P es el cociente ceviano del baricentro y P.
    ( Mostrar/Ocultar figura )
      JCGM2015p001_pK.png

    EJEMPLOS (ver §3.1 Central pK isocubics. J.P.Ehrmann and B.Gibert):

    P Isocúbica de polo X2/P
    y pivote X2
    Isocúbica de centro P
    pK(X2/P,hG,4(P))
    pKc(Q)
    Q = aP =ta(X2/P)
    X3 K002, pK(X6,X2) K004, pK(X6,X20) K004, pKc(X4)
    [a^2(2a^2-b^2-c^2)(a^4-b^4+b^2c^2-c^4)] K043, pK(X187, X2) K042, pK(X187,aX67) K042, pKc(X67)
    X6 K168, pK(X3, X2) pK(X3, X193) pKc(X69)
    X206 K177, pK(X32,X2) K161, pK(X32,X5596) K161, pKc(X66)
    X523 K237, pK(X115,X2) pK(X115,X148) pKc(X523)
    [a(a^2-bc)(a^3-b^3+abc-c^3)] K251, pK(X238,X2) pK(X238, [3a^3-b^3-abc-c^3]) pKc([a^6-2a^4bc+a^2b^2c^2-(b^3-c^3)^2])
    [(-a^6+a^2b^2c^2)(a^6-b^6+a^2b^2c^2-c^6)] K252, pK(X1691,X2) pK(X1691, [-3a^12+ 2a^8b^2c^2+ 2a^6(b^6+c^6) + a^4b^4c^4 - 2a^2b^2c^2(b^6+c^6)+ (b^6-c^6)^2 ]) pKc([a^12 - 2a^8b^2c^2+ a^4b^4c^4 - (b^6-c^6)^2])
    X960 K253, pK(X2092,X2) pK(X2092, [a^3 (b^2+3 b c+c^2)+a^2 (b^3+b^2 c+b c^2+c^3)-a b c (b^2+b c+c^2)-b^2 c^2 (b+c)]) pKc(aX960)
    [a^2(a^4-b^4+4b^2c^2-c^4) (a^2-2(b^2+c^2))] K284, pK(X574,X2) pK(X574,[-3a^8+8a^6(b^2+c^2) + 2a^4(b^4-12b^2c^2+c^4) - 8a^2(b^6-2b^4c^2-2b^2c^4+c^6)+ (b^4-c^4)^2 ]) pKc(X5486)
    [a^3(a^2+2bc+a(b+c)) (-a^3(a^2+2bc+a(b+c)) + c^3(c(b+c)+a(2b+c)) + b^3(b(b+c)+a(b+2c)))] K321, pK(X5019,X2) pK(X5019,?) pKc([1/((a^5-b^5+2a^3bc-b^4c-bc^4-c^5+a^4(b+c)- a(b^4+2b^3c+2bc^3+c^4)))])
    [a^2(S+Sqrt[3]SA])(b^2(Sqrt[3]SB+S) + c^2(Sqrt[3]SC+S) - a^2(Sqrt[3]SA+S))] K341a, pK(X15,X2) pK(X15,?) pKc(X2992)
    [a^2(S-Sqrt[3]SA])(b^2(Sqrt[3]SB-S) + c^2(Sqrt[3]SC-S) - a^2(Sqrt[3]SA-S))] K341b, pK(X16,X2) pK(X16,?) pKc(X2993)
    X10 K345, pK(X37,X2) K033, pK(X37,X8) K033, pKc(X1)
    [a^4(b^4+c^4-a^2(b^2+c^2))^2] K357, pK(X511,X2) pK(X551,[a^2(a^2b^2-b^4+a^2c^2-c^4)]) pKc(X290)
    X9 K363 , pK(X1,X2) K202, pK(X1,X144) K202, pKc(X7)
    X214 K453 , pK(X44,X2) pK(X44,[-3a^4+2a^3(b+c) + a^2(2b^2-3bc+2c^2) + a(-2b^3+b^2c+bc^2-2c^3)+ (b^2-c^2)^2]) pKc(X80)
    X3163 K472 , pK(X30,X2) pK(X30,[-5a^8+5a^6(b^2+c^2)+a^4(6b^4-17b^2c^2+6c^4)- 7a^2(b^2-c^2)^2(b^2+c^2) + (b^2-c^2)^2(b^4+7b^2c^2+c^4)]) pKc(X1494)
    [(4 a^4 - 2 (b^2 - c^2)^2 + a^2 (b^2 + c^2)) (-8 a^4 + 2 b^4 + 2 b^2 c^2 + 2 c^4 + 2 (b^2 - c^2)^2 - a^2 (b^2 + c^2) + a^2 (4 b^2 + c^2) + a^2 (b^2 + 4 c^2))] K485, pK([4a^4+a^2(b^2+c^2)-2(b^2-c^2)^2],X2) pK([4a^4+a^2(b^2+c^2)-2(b^2-c^2)^2],[-20 a^8 + 2 a^6 (b^2 + c^2) + 2 (b^2 - c^2)^2 (2 b^4 + 11 b^2 c^2 + 2 c^4) + a^4 (24 b^4 - 29 b^2 c^2 + 24 c^4) + a^2 (-10 b^6 + 13 b^4 c^2 + 13 b^2 c^4 - 10 c^6)]) pKc([1/(4 a^4 - 2 b^4 + b^2 c^2 - 2 c^4 - 2 a^2 (b^2 + c^2))])
    X113 K489 , pK(X3003,X2) K255 pK(X3003,X146) K255 , pKc(X74)
    X5 K612, pK(X216,X2) K044 pK(X216,X4) K044 , pKc(X3)
    X3647 K637, pK(X1100,X2) pK(X1100,X3648) pKc(X79)
    X1249 K663, pK(X4,X2) pK(X4,[-7a^8 + 4a^6(b^2+c^2)+ 14a^4(b^2-c^2)^2 - 12a^2(b^2-c^2)^2(b^2+c^2)+ (b^2-c^2)^2(b^4+14b^2c^2+c^4)]) pKc(X253)
    [(-(b^2 - c^2)^2 + a^2 (b^2 + c^2)) (a^8 + 2 b^2 c^2 (b^2 - c^2)^2 - 3 a^6 (b^2 + c^2) - a^2 (b^2 - c^2)^2 (b^2 + c^2) + 3 a^4 (b^4 + c^4))/(a^2 - b^2 - c^2)] K671, pK(X53,X2) pK(X53,[3 b^2 c^2 (b^2 - c^2)^6 + a^14 (b^2 + c^2) - a^12 (2 b^4 + 9 b^2 c^2 + 2 c^4) + a^8 (b^2 - c^2)^2 (20 b^4 + 37 b^2 c^2 + 20 c^4) + a^10 (-5 b^6 + 11 b^4 c^2 + 11 b^2 c^4 - 5 c^6) - a^2 (b^2 - c^2)^4 (3 b^6 + 7 b^4 c^2 + 7 b^2 c^4 + 3 c^6) - 5 a^6 (b^2 - c^2)^2 (5 b^6 + 7 b^4 c^2 + 7 b^2 c^4 + 5 c^6) + a^4 (b^2 - c^2)^2 (14 b^8 + 5 b^6 c^2 + 26 b^4 c^4 + 5 b^2 c^6 + 14 c^8)]) pKc([1/(a^8 + 2 b^2 c^2 (b^2 - c^2)^2 - 3 a^6 (b^2 + c^2) - a^2 (b^2 - c^2)^2 (b^2 + c^2) + 3 a^4 (b^4 + c^4))])
    X618 pK(X396,X2) K046-a, pK(X396,X616) K046-a, pKc(X13)
    X619 pK(X395,X2) K046-b, pK(X395,X617) K046-b, pKc(X14)
    X141 pK(X39,X2) K140, pK(X39,X69) K140, pKc(X6)
    X1 pK(X9,X2) K201, pK(X9,X145) K201, pKc(X8)
    X1125 pK(X1213,X2) pK(X1213,X1) pKc(X10)



    ∗  ∗  ∗  ∗  ∗  ∗  ∗  ∗  ∗  ∗ 



      Si el punto U es fijo, el lugar geométrico de los puntos P tales que las rectas ARa, BRb y CRc son concurrentes en un punto Q, es una cónica c(U) circunscrita al triángulo medial.

    ( Mostrar/Ocultar figura )
      JCGM2015p001conica.png

      El punto Q queda en la cónica, anticomplementaria de c(U):

    u(v+w-u)(v-w)yz + v(w+u-v)(w-u)zx + w(u+v-w)(u-v)xy = 0,

    circunscrita a ABC, que pasa por U y por su anticomplemento aU. Las rectas PQ, cuando P varía, pasan por G/U, cociente ceviano del baricentro y U (centro de perspectividad de los triángulos medial y UaUbUc, centro de la cónica circunscrita de perspector U).

    NOTA sobre cuadrivértices y cuadriláteros:


      Los puntos U, Ua, Ub y Uc forman un cuadrivértice, de triángulo diagonal ABC. Las rectas UG/U, UaG/Ua, UbG/Ub, UcG/Uc concurren en el cuadrado baricéntrico de U, que es el punto QA-P16 de "Encyclopedia Of Quadri-Figures (EQF)". Ver también QL-P13
      Si QA-P1 es el baricentro del cuadrivértice UUaUbUc, PA-Q16 es el complemento del conjugado isotómico del anticomplemento de QA-P1.
    ( Mostrar/Ocultar figura )
      QA-P16.png

    La cónica c(U) es una hipérbola rectangular si y sólo si U está en la cúbica de Thomson, K002.

    En particular, se obtienen las hipérbolas rectangulares el triángulo medial: c(X2) es la hipérbola de Kiepert; c(X1)=c(X9) es la hipérbola de Feuerbach; c(X3)=c(X6) es la hipérbola de Jerabek.
    ( Mostrar/Ocultar figura )
      JCGM2015p001 Hip Eq.png


    La cónica c(U) es una parábola si y sólo si U está en la cúbica de Allardice A1(G), K219.

    ( Mostrar/Ocultar figura )
      JCGM2015p001Parabolas.png



  • jueves, 1 de enero del 2015

    Cónicas circunscritas y bisectrices asociadas


      Dado un triángulo ABC y un punto P, las tangentes en los vértices de ABC a la cónica circunscrita de perspector P cortan a los lados opuesto en los punto D, E y F (alineados en la tripolar de P, que coincide con la polar de P respecto a la cónica).
      Las bisectrices de los ángulos ADB, BEC y CFA forman un triángulo perspectivo con ABC, con centro de perspectividad Q.


    ( Mostrar/Ocultar figura )
      X256-conica.png
    Descargar fichero GeoGebra

      Si las coordenadas baricéntricas de P son (u:v:w):

    Q = ( a / (a v w + u Sqrt[a^2 v w+(c^2v-b^2w)(v-w)]) : .. : .. )

    • Si P =X1 (incentro), Q = X(1128) = segundo punto de De Villiers = punto de Jacobi J(A-π)/4 (§16.4.19 PDF):

    (sen A/(1 +2sen(A/2)) : sen B/(1 +2sen(B/2)) : sen C/(1 +2sen(C/2)) ).

    ( Mostrar/Ocultar figura )
      X256-X1.png
    Descargar fichero GeoGebra
    ETC X(1128)
      Let Ia, Ib ,Ic be the excenters of ABC, and let Xa, Xb, Xc be the respective incenters of triangles IaBC, IbCA, IcAB, respectively. The triangle XaXbXc is perspective to ABC, and the perspector is X(1128). (Darij Grinberg, 8/22/02).


    • Si P =X2 (baricentro), Q = X(2)
    • Si P =X6 (simediano), Q = X(256) = primer punto de Sharygin:

    (a/(a^2 + bc) : b/(b^2 + ca) : c /(c^2 + ab) )


    ( Mostrar/Ocultar figura )
      X256-X6.png
    Descargar fichero GeoGebra
    ETC X(1281)
      The Sharygin points are described in
    Darij Grinberg, Sharygin Points Report, Hyacinthos #6293 (1/8/03) and #6315 (1/10/03)
    The first of ten sections is an Introduction quoted, in part, here:

    We will treat two remarkable triangles: the triangle bounded by the perpendicular bisectors of the internal angle bisectors of a triangle ABC, and the triangle bounded by the perpendicular bisectors of the external angle bisectors of triangle ABC. These two triangles and the triangle ABC are three perspective triangles, having a common perspectrix: the Lemoine axis of ABC. The mutual perspectors of the three triangles will be called the first, second and third Sharygin points of ABC (after a problem of Igor Sharygin - see Section 10).

    The report introduces fifteen Sharygin points, of which the 1st, 2nd, 4th, and 6th are X(256), X(291), X(846), and X(1054), respectively. X(1281) is the 3rd Sharygin point.

      Otra propiedad geométrica de X256 se describe en: Puntos de Sharygin, pág. 6-7.
    Al final de este artículo se transcribe: "Darij Grinberg.- Sharygin Points Report" (which is not currently at the Files Directory of Hyacinthos as SHARYGIN.ZIP).

    • Si P =X145 (anticomplemento del punto de Nagel), Q = X(1)
    ( Mostrar/Ocultar figura )
      X256-X145.png
    Descargar fichero GeoGebra
    La cónica circunscrita de perspector X145, pasa por X190 (Yff Parabolic Point) y su centro es

    (15a^2 - 14a(b+c) + 3(b+c)^2 : 15b^2 - 14b(c+a) + 3(c+a)^2 : 15c^2 - 14c(a+b) + 3(a+b)^2),

    con número de búsqueda en ETC: {2.15130173930895, -0.956134106359534, 3.30969498355223}.
Página Personal